Sunteți pe pagina 1din 276

19CS106 Discrete Mathematical Structures Unit wise Question Bank

Unit –I
1. Let P: I am in Bangalore. Q: I love cricket. Then q -> p (q implies p) is?
a) If I love cricket then I am in Bangalore
b) If I am in Bangalore then I love cricket
c) I am not in Bangalore
d) I love cricket
2. Let P: We give a nice overall squad performance, Q: We will win the match.
Then the symbolic form of “We will win the match if and only if we give a nice overall squad
performance.“ is?
a) P v Q
b) Q ∧ P
c) Q<->P
d) ~P v Q
3. Let P, Q, R be true, false and true respectively, which of the following is true?
a) P∧Q∧R
b) P∧~Q∧~
R c) Q-
>(P∧R)
d) P->(Q∧R)
4. Consider the following statements.
A: Raju should exercise.
B: Raju is not a decent table tennis player.
C: Raju wants to play good table tennis.
The symbolic form of “Raju is not a decent table tennis player and if he wants to play good table
tennis then he should exercise.” is?
a) A->B->C
b) B∧(C->A)
c) C->B∧A
d) B<->A∧C
5. The statement (~P<->Q)∧~Q is true
when? a) P: True Q: False
b) P: True Q: True
c) P: False Q: True
d) P: False Q: False
6. Which of the following is De-Morgan’s law?
a) P ∧ (Q v R) Ξ (P ∧ Q) v (P ∧ R)
b) ~(P ∧ R) Ξ ~P v ~R, ~(P v R) Ξ ~P ∧ ~R
c) P v ~P Ξ True, P ∧ ~P Ξ False
d) None of the mentioned
7. Which of the following satisfies commutative law?
a) ∧
b) v
c) ↔
d) All of the mentioned
8. The contrapositive of p → q is the proposition of
a) ¬p → ¬q
b) ¬q → ¬p
c) q → p
d) ¬q → p
9. What are the contrapositive of the conditional statement “I come to class whenever there is going
to be a test.”
a) “If I come to class, then there will be a test.”
b) “If I do not come to class, then there will not be a test.”
c) “If there is not going to be a test, then I don’t come to class.”
d) “If there is going to be a test, then I don’t come to class.”
10. P ∨ q is logically equivalent to
a) ¬q → ¬p
b) q → p
c) ¬p → ¬q
d) ¬p → q
11. Which of the following statement is correct?
a) p ∨ q ≡ q ∨ p
b) ¬(p ∧ q) ≡ ¬p ∨ ¬q
c) (p ∨ q) ∨ r ≡ p ∨ (q ∨
r) d) All of mentioned
12. Let domain of m includes all students, P (m) be the statement “m spends more than 2 hours
in playing polo”. Express ∀m ¬P (m) quantification in English.
a) A student is there who spends more than 2 hours in playing polo
b) There is a student who does not spend more than 2 hours in playing polo
c) All students spends more than 2 hours in playing polo
d) No student spends more than 2 hours in playing polo
13. Translate ∀x∃y(x < y) in English, considering domain as a real number for both the
variable. a) For all real number x there exists a real number y such that x is less than y
b) For every real number y there exists a real number x such that x is less than y
c) For some real number x there exists a real number y such that x is less than y
d) For each and every real number x and y such that x is less than y
14. Which of the following statement is the negation of the
statement? “2 is even and –3 is negative”?
a) 2 is even and –3 is not negative.
b) 2 is odd and –3 is not negative.
c) 2 is even or –3 is not negative.
d) 2 is odd or –3 is not negative.
15. Express, “The difference of a real number and itself is zero” using required operators.
a) ∀x(x − x! = 0)
b) ∀x(x − x = 0)
c) ∀x∀y(x − y = 0)
d) ∃x(x − x = 0)
16. Find a counterexample of ∀x∀y(xy > y), where the domain for all variables consists of
all integers.
a) x = -1, y = 17
b) x = -2 y = 8
c) Both x = -1, y = 17 and x = -2 y = 8
d) Does not have any counter example
17. Which rule of inference is used in each of these arguments, “If it is Wednesday, then the
Smartmart will be crowded. It is Wednesday. Thus, the Smartmart is crowded.”
a) Modus tollens
b) Modus ponens
c) Disjunctive syllogism
d) Simplification
18. What rule of inference is used in this argument?
“If I go for a balanced diet, then I will be fit. If I will be fit, then I will remain healthy. Therefore, if I
go for a balanced diet, then I will remain healthy.”
a) Modus tollens
b) Modus ponens
c) Disjunctive syllogism
d) Hypothetical syllogism
19. The premises (p ∧ q) ∨ r and r → s implies which of the conclusion?
a) p ∨ r
b) p ∨ s
c) q ∨ s
d) q ∨ r

20. “Parul is out for a trip or it is not snowing” and “It is snowing or Raju is playing chess” imply
that
a) Parul is out for trip
b) Raju is playing chess
c) Parul is out for a trip and Raju is playing
chess d) Parul is out for a trip or Raju is playing
chess

21. Which of the following can only be used in disproving the statements?
a) Direct proof
b) Contrapositive proofs
c) Counter Example
d) Mathematical Induction

22. In proving √5 as irrational, we begin with assumption √5 is rational in which type of proof?
a) Direct proof
b) Proof by Contradiction
c) Vacuous proof
d) Mathematical Induction

23. A proof covering all the possible cases, such type of proofs are known as
a) Direct proof
b) Proof by Contradiction
c) Vacuous proof
d) Exhaustive proof

24. Identify the valid conclusion from the premises Pv Q, Q → R, P → M and ~ M


a)P ^ (R v R)
b)P ^ (P ^ R)
c)R ^ (P v Q)
d)Q ^ (P v R)

25. Which of the arguments is not valid in proving sum of two odd number is not odd.
a) 3 + 3 = 6, hence true for all
b) 2n +1 + 2m +1 = 2(n+m+1) hence true for all
c) All of the mentioned
d) None of the mentioned
26. A theorem used to prove other theorems is known as _
a) Lemma
b) Corollary
c) Conjecture
d) None of the mentioned
27. The compound statement A-> (A->B) is false, and then the truth values of A, B are respectively
a) T, T
b) F,
T c) T,
F
d) F, F
28. The compound propositions p and q are called logically equivalent if is a tautology.
a) p ↔ q
b) p → q
c) ¬ (p ∨ q)
d) ¬p ∨ ¬q
29. The statement,” Every comedian is funny” where C(x) is “x is a comedian” and F (x) is “x is
funny” and the domain consists of all people, is represented using logical connectives, predicates and
quantifiers as
a) ∃x(C(x) ∧ F (x))
b) ∀x(C(x) ∧ F (x))
c) ∃x(C(x) → F (x))
d) ∀x(C(x) → F (x))
30. What rules of inference are used in this argument?
“Jay is an awesome student. Jay is also a good dancer. Therefore, Jay is an awesome student and a
good dancer.”
a) Conjunction
b) Modus ponens
c) Disjunctive syllogism
d) Simplification

Unit –II

1. What is the Cartesian product of A = {1, 2} and B = {a, b}?


a) {(1, a), (1, b), (2, a), (b, b)}
b) {(1, 1), (2, 2), (a, a), (b, b)}
c) {(1, a), (2, a), (1, b), (2, b)}
d) {(1, 1), (a, a), (2, a), (1, b)}
2. What is the cardinality of the set of odd positive integers less than 10?
a) 10
b) 5
c) 3
d) 20
3. Which of the following two sets are equal?
a) A = {1, 2} and B = {1}
b) A = {1, 2} and B = {1, 2, 3}
c) A = {1, 2, 3} and B = {2, 1, 3}
d) A = {1, 2, 4} and B = {1, 2, 3}
4. The members of the set S = {x | x is the square of an integer and x < 100} is
a) {0, 2, 4, 5, 9, 58, 49, 56, 99, 12}
b) {0, 1, 4, 9, 16, 25, 36, 49, 64, 81}
c) {1, 4, 9, 16, 25, 36, 64, 81, 85, 99}
d) {0, 1, 4, 9, 16, 25, 36, 49, 64, 121}
5. The intersection of the sets {1, 2, 5} and {1, 2, 6} is the set
a) {1, 2}
b) {5, 6}
c) {2, 5}
d) {1, 6}
6. If A is {{Φ}, {Φ, {Φ}}}, then the power set of A has how many element?
a) 2
b) 4
c) 6
d) 8
7. Which sets are not empty?
a) {x: x is a even prime greater than 3}
b) {x : x is a multiple of 2 and is odd}
c) {x: x is an even number and x+3 is even}
d) { x: x is a prime number less than 5 and is odd}

8. If set C is {1, 2, 3, 4} and C – D = Φ then set D can be


a) {1, 2, 4, 5}
b) {1, 2, 3}
c) {1, 2, 3, 4, 5}
d) None of the mentioned
9. Which of the following statement regarding sets is false?
a) A ∩ A = A
b) A U A = A
c) A – (B ∩ C) = (A – B) U (A –C)
d) (A U B)’ = A’ U B’
10. Let set A = {1, 2} and C be {3, 4} then A X B (Cartesian product of set A and B) is?
a) {1, 2, 3, 4}
b) {(1, 3),(2, 4)}
c) {(1, 3), (2, 4), (1, 4), (2, 3)}
d) {(3, 1), (4, 1)}
11. If the number of subsets of a set are 4 then the number of elements in that sets are
a) 1
b) 2
c) 3
d) 4
12. Let the set be A= {a, b, c, {a,b}} then which of the following is false?
a) {a, b} Є A
b) a Є A
c) {a} Є A
d) b, c ЄA
13. The value of ⌊1/2.⌊5/2⌋ ⌋ is
a) 1
b) 2
c) 3
d) 0.5
14. The domain of the function that assign to each pair of integers the maximum of these two
integers is
a) N
b) Z
c) Z +
d) Z+ X Z+
15. The inverse of function f(x) = x3 + 2 is
a) f -1 (y) = (y – 2) 1/2
b) f -1 (y) = (y – 2) 1/3
c) f -1 (y) = (y) 1/3
d) f -1 (y) = (y – 2)

16. Let f and g be the function from the set of integers to itself, defined by f(x) = 2x + 1 and g(x) =
3x + 4. Then the composition of f and g is
a) 6x + 9
b) 6x + 7
c) 6x + 6
d) 6x + 8
17. What is the domain of a function?
a) the maximal set of numbers for which a function is defined
b) the maximal set of numbers which a function can take values
c) it is a set of natural numbers for which a function is defined
d) none of the mentioned
18. Codomain is the subset of range.
a) True
b) False
19. A mapping f : X → Y is one one if
a) f(x1) ≠ f(x2) for all x1, x2 in X
b) If f(x1) = f(x2) then x1 = x2 for all x1, x2 in X
c) f(x1) = f(x2) for all x1, x2 in X
d) None of the mentioned
20. Floor (2.4) + Ceil (2.9) is equal to
a) 4
b) 6
c) 5
d) none of the mentioned
21. A ceil function map a real number to
a) smallest previous integer
b) greatest previous integer
c) smallest following integer
d) none of the mentioned
22. For an inverse to exist it is necessary that a function should be
a) injection
b) bijection
c) surjection
d) none of the mentioned
23. A function f(x) is defined from A to B then f -1 is defined
a) from A to B
b) from B to A
c) depends on the inverse of function
d) none of the mentioned
24. In the given AP series find the number of terms?

5, 8, 11, 14, 17, 20.........50.

a)11
b)13
c)15
d) None of the mentioned

25. If in the truth table the answer column has the truth values both TRUE and FALSE then it is said
to be .

A. tautology

B. contradiction

C. contingency

D. equivalence relation

26. A = {1, 3, 5, 7, 9} is a .

A. null set

B. finite set
C. singleton set

D. infinite set

27. If cardinality of (A U B) = cardinality of A+ cardinality of B. This means

a) A is a subset of B

b) B is a subset of A

c) A and B are disjoint

d) None of the mentioned

28. For the sequence 0, 1, 2, 3 an is

a) ⌈n/2⌉+

⌊n/2⌋ b)

⌈n/2⌉+⌈n/2⌉

c) ⌊n/2⌋+⌊n/2⌋

d) ⌊n/2⌋

29. A drawer contains 12 red and 12 blue socks, all unmatched. A person takes socks out at random
in the dark. How many socks must he take out to be sure that he has at least two blue socks?

a) 18

b) 35

c) 28

d) 14

30. A bag contains 25 balls such as 10 balls are red, 7 are white and 8 are blue. What is the minimum
number of balls that must be picked up from the bag blindfolded (without replacing any of it) to be
assured of picking at least one ball of each colour?

a) 10

b) 18
c) 63
d) 35

Unit –III
1. There are 70 patients admitted in a hospital in which 29 are diagnosed with typhoid, 32 with
malaria, and 14 with both typhoid and malaria. Find the number of patients diagnosed with
typhoid or malaria or both.

a) 39

b) 17

c) 47
d) 53

2. The numbers between 1 and 520, including both, are divisible by 2 or 6 is

a) 349

b) 54

c) 213

d) 303

3. What is the generating function for the sequence 1, 6, 16, 216,

….? a) (1+6x)/x3

b) 1/(1−6x)

c) 1/(1−4x)

d) 1-6x2

4. What is the generating function for generating series 1, 2, 3, 4, 5,…


? a) 2/(1−3x)
b) 1/(1+x)
c) 1/(1−x)2
d) 1/(1−x2)
5. Consider the recurrence relation a1=4, an=5n+an-1. The value of a64 is
a) 10399
b) 23760
c) 75100
d) 53700
6. Determine the solution of the recurrence relation Fn=20Fn-1 − 25Fn-2 where F0=4 and F1=14.
a) an = 14*5n-1
b) an = 7/2*2n−1/2*6n
c) an = 7/2*2n−3/4*6n+1
d) an = 3*2n−1/2*3n
7. Find the value of a4 for the recurrence relation an=2an-1+3, with a0=6.
a) 320
b) 221
c) 141
d) 65
8. What is the solution to the recurrence relation an=5an-1+6an-2?
a) 2n2
b) 6n
c) (3/2)n
d) n!*3
9. The sum of all integers from 1 to 520 that are multiples of 4 or
5? a) 187
b) 208
c) 421
d) 52
10. What is the recurrence relation for the sequence 1, 3, 7, 15, 31, 63,…?
a) an = 3an-1−2an+2
b) an = 3an-1−2an-2
c) an = 3an-1−2an-1
d) an = 3an-1−2an-3

11. Determine the value of a2 for the recurrence relation an = 17an-1 + 30n with a0=3.
a) 4387
b) 5484
c) 238
d) 1437
12. The solution to the recurrence relation an=an-1+2n, with initial term a0=2 are
a) 4n+7
b) 2(1+n)
c) 3n2
d) 5*(n+1)/2
13. Which of the following is a correct representation of inclusion exclusion principle (|
A,B| represents intersection of sets A,B)?
a) |A U B|=|A|+|B|-|A,B|
b) |A,B|=|A|+|B|-|A U B|
c) |A U B|=|A|+|B|+|A,B|
d) |A,B|=|A|+|B|+|A U B|
14. Which of the following is not an application of inclusion-exclusion principle?
a) Counting intersections
b) Graph coloring
c) Matching of bipartite graphs
d) Maximum flow problem
15. Who invented the concept of inclusion-exclusion principle?
a) Abraham de Moivre
b) Daniel Silva
c) J.J. Sylvester
d) Sieve
16. With reference to the given Venn diagram, what is the formula for computing |
AUBUC| (where |x, y| represents intersection of sets x and y)?

a) |A U B U C|=|A|+|B|+|C|-|A,B|-|A,C|-|B,C|+|A, B,C|
b) |A, B,C|=|A|+|B|+|C|-|A U B|-|A U C|-|B U C|+|A U B U C|
c) |A, B,C|=|A|+|B|+|C|+|A,B|-|A,C|+|B,C|+|A U B U C|
d) |A U B U C|=|A|+|B|+|C| + |A,B| + |A,C| + |B,C|+|A, B,C|

17. Determine the solution for the recurrence relation an = 6an-1−8an-2 provided initial conditions
a0=3 and a1=5.
a) an = 4 * 2n – 3n
b) an = 3 * 7n – 5*3n
c) an = 5 * 7n
d) an = 3! * 5n
18. Determine the solution for the recurrence relation bn=8bn-1−12bn-2 with b0=3 and b1=4.
a) 7/2*2n−1/2*6n
b) 2/3*7n-5*4n
c) 4!*6n
d) 2/8n
19. If Sn=4Sn-1+12n, where S0=6 and S1=7, find the solution for the recurrence relation.
a) an=7(2n)−29/6n6n
b) an=6(6n)+6/7n6n
c) an=6(3n+1)−5n
d) an=nn−2/6n6n

20. What is the sequence depicted by the generating series 4 + 15x2 + 10x3 + 25x5 +
16x6+⋯? a) 10, 4, 0, 16, 25, …
b) 0, 4, 15, 10, 16, 25,…
c) 4, 0, 15, 10, 25, 16,…
d) 4, 10, 15, 25,…
21. What is the generating function for the generating sequence A = 1, 9, 25, 49,
…? a) 1+(A-x2)
b) (1-A)-1/x
c) (1-A)+1/x2
d) (A-x)/x3
22. What will be the sequence generated by the generating function 4x/(1-x)2?
a) 12, 16, 20, 24,…
b) 1, 3, 5, 7, 9,…
c) 0, 4, 8, 12, 16, 20,…
d) 0, 1, 1, 3, 5, 8, 13,…

23. What is the recurrence relation for the sequence 1, 3, 7, 15, 31, 63,…?
a) an = 3an-1−2an+2
b) an = 3an-1−2an-2
c) an = 3an-1−2an-1

d) an = 3an-1−2an-3

24. In class, students want to join sports. 15 people will join football, 24 people will
join basketball, and 7 people will join both. How many people are there in the class?
a) 19
b) 82
c) 64
d) 30

25. At a software company, skilled workers have been hired for a project. Out of 75 candidates,
48 of them were software engineer; 35 of them were hardware engineer; 42 of them were
network engineer; 18 of them had skills in all three jobs and all of them had skills in at least one
of these jobs. How many candidates were hired who were skilled in exactly 2 jobs?
a) 69
b) 14
c) 32
d) 8

26. A recurrence relation is expressed in terms of ………………………

A.Previous terms.

B. present existing terms.

C. Previous and Present existing terms.


D. No Previous and No Present terms

27. Let {an} be a sequence that satisfies the recurrence relation an= an-1+ an-2 for n=2,3,4…..and

suppose that a0=3 and a1=5. What are a2 and a3?

A. a2=2, a3=3
B. a2=2, a3=-3
C. a2=-2, a3=3
D. a2=-2, a3=-3

28. A recurrence relation of the form C0 an+C1 an-1+C2 an-2.+.......+Ck an-k=f(n).if f(n)=0 then the
recurrence relation is called as

A.Linear homogeneous recurrence relation

B.Linear non homogeneous recurrence relation

C.Non Linear homogeneous recurrence relation.

D. Non Linear non homogeneous recurrence relation.

29. if r1 and r2 are the real and distinct roots of the recurrence relation of the
form an=A.an-1+B.an-2.then the solution of recurrence relation
is------------------------

A.an=A.r1 n+B.r n

B. an=(A+B)r1n

C. an=(A+B)r2n

D. an= rn(A a cos n θ + B sin n θ )

30. If r1 and r2 are the real and equal roots of the recurrence relation of the form an=A.an-

1+B.an-2.then the solution of recurrence relation is------------------------

A.an=A.r1 n+B.r n

B. an=(A+Bn)rn

C. an=(A+B)rn

D. an= rn(A a cos n θ + B sin n θ )

Unit –IV
1. The binary relation {(1,1), (2,1), (2,2), (2,3), (2,4), (3,1), (3,2)} on the set {1, 2, 3} is
a) reflective, symmetric and transitive
b) irreflexive, symmetric and transitive
c) neither reflective, nor irreflexive but transitive
d) irreflexive and antisymmetric

2. Determine the characteristics of the relation aRb if a2 = b2.


a) Transitive and symmetric
b) Reflexive and asymmetry
c) Trichotomy, antisymmetry, and irreflexive
d) Symmetric, Reflexive, and transitive
3. Let a set S = {2, 4, 8, 16, 32} and <= be the partial order defined by S <= R if a divides b.
Number of edges in the Hasse diagram of is
a) 6
b) 5
c) 9
d) 4

4. The less-than relation, <, on a set of real numbers is

a) not a partial ordering because it is not asymmetric and irreflexive equals antisymmetric

b) a partial ordering since it is asymmetric and reflexive

c) a partial ordering since it is antisymmetric and reflexive

d) not a partial ordering because it is not anti-symmetric and reflexive

5. Suppose a relation R = {(3, 3), (5, 5), (5, 3), (5, 5), (6, 6)} on S = {3, 5, 6}. Here R is
known as

a) equivalence relation

b) reflexive relation

c) symmetric relation

d) transitive relation

6. Which of the following relations is the reflexive relation over the set {1, 2, 3,

4}? a) {(0,0), (1,1), (2,2), (2,3)}

b) {(1,1), (1,2), (2,2), (3,3), (4,3), (4,4)}

c) {,(1,1), (1,2), (2,1), (2,3), (3,4)}

d) {(0,1), (1,1), (2,3), (2,2), (3,4), (3,1)

7. number of reflexive relations are there on a set of 11 distinct elements.

a) 2110

b) 3121

c) 290
d) 2132

8. If a set A has 8 elements and a set B has 10 elements, how many relations are there from A
to B?

a) 290

b) 380

c) 164

d) 280

9. Amongst the properties {reflexivity, symmetry, antisymmetric, transitivity} the


relation R={(a,b) ∈ N2 | a!= b} satisfies property.
a) Symmetric
b) transitivity
c) antisymmetric
d) reflexivity

10. .If every two elements of a poset are comparable then the poset is called
a) sub ordered poset
b) totally ordered poset
c) sub lattice
d) semi group

11. The transitive closure of the relation {(0,1), (1,2), (2,2), (3,4), (5,3), (5,4)} on the set {1, 2, 3,
4, 5} is
a) {(0,1), (1,2), (2,2), (3,4)}
b) {(0,0), (1,1), (2,2), (3,3), (4,4), (5,5)}
c) {(0,1), (1,1), (2,2), (5,3), (5,4)}
d) {(0,1), (0,2), (1,2), (2,2), (3,4), (5,3), (5,4)}

12. Let R be an equivalence relation on a set S. Then R need NOT satisfy the property:
a) Reflexive

b) Symmetric

c)Anti-symmetric

d)Transitive

13. A relation is reflexive if in the graphical representation of relation, every node has a
loop. a) True
b) False
14. Let A = {1, 2, 3, 4} and R be a relation in A given by R = {(1, 1), (2, 2), (3, 3), (4, 4), (1,
2), (2, 1), (3, 1), (1, 3) }. Then R is

(a) reflexive

(b) Symmetric

(c) Transitive

(d) An equivalent relation

15. Suppose a relation R = {(3, 3), (5, 5), (5, 3),(3,5) (5, 5), (6, 6)} on S = {3, 5, 6}. Here R is
known as
a) equivalence relation
b) reflexive relation
c) symmetric relation
d) transitive relation

16. The relation ≤ is a partial order if it is


a) reflexive, antisymmetric and transitive
b) reflexive, symmetric
c) asymmetric, transitive
d) irreflexive and transitive

17. A Poset in which every pair of elements has both a least upper bound and a greatest lower
bound is termed as

a) Sub lattice

b) Lattice

c) Trail

d) Walk

18. If a relation is reflexive, then all the diagonal entries in the relation matrix
must be .

A. 0

B. 1

C. 2

D. None of the Mentioned


19. How many binary relations are there on a set S with 9 distinct elements?

a) 290

b) 2100

c) 281

d) 260

20. Let A = {1, 2, 3} and B = {u, v, w}. Then which of the following is NOT a relation from
A to B?

a) {(1, v)}

b) {(1, u), (3, 2), (2, w)}

c) {(3, v)}

d) {(1, u), (1, v), (1, w)}

21. Hasse diagram are drawn for

a) Partially ordered sets

b) Lattices

c) Boolean algebra

d) None of the mentioned

22. The number of relations from A = {a,b,c} to B = {1,2} are _.

A. 6

B. 8

C. 32

D. 64

23. Let R = {(3, 3), (6, 6), (9, 9), (12,12), (3,6), (6,3), (3, 9), (9, 3), (9, 12),(12,9)} be a relation

on the set A = {3, 6, 9, 12}. The relation is

A. reflexive and transitive

B. reflexive and symmetric


C. symmetric and transitive

D. equivalence relation

24. Let R={(1,b),(3,d),(2,b)} and S={(b,4),(2,5),(d,a)} be a relation then R composition

S= .

A. {(1,b),(3,d),(2,b)}

B. {(1,4),(3,a),(2,4)}

C. {(4,b),(2,5),(3,a)}

D. {(1,d),(3,b),(2,c)}

25. Let R={(1, 3), (4, 2), (2, 2), (3, 3), (1, 1),(4,4)} be a relation on the set A={1, 2, 3, 4}. The

relation R is .

A. transitive

B. reflexive

C. not symmetric

D. function

26. If every aRb implies bRa then a relation R will be..............relation.


A) Reflexive
B) symmetric
C) Asymmetric
D) Anti symmetric
27. Determine the characteristics of the relation aRb if a2 = b2.
a) Transitive and symmetric
b) Reflexive and asymmetry
c) Trichotomy, antisymmetry, and irreflexive
d) Symmetric, Reflexive, and transitive
28. Suppose a relation R = {(4, 4), (5, 5), (5, 4), (5, 5), (6, 6)} on S = {4, 5, 6}. Here R is
known as
a) equivalence relation
b) reflexive relation
c) symmetric relation
d) transitive relation
29. If R = ((1, 1), (3, 1), (2, 3), (4, 2)), then which of the following represents R2, where R2 is R
composite R?
A. ((1, 1), (2, 1), (4, 3), (3, 1))
B. ((1, 1), (3, 1), (2, 3), (4, 2))
C. 1(1, 3), (3, 3), (3, 4), (3, 2))
D. f(1, 1), (9, 1), (4, 9), (16, 4))

30. Which of the following relations is the anti-symmetric relation over the set {1, 2, 3, 4}?
a) {(0,0), (1,1), (2,2), (2,3)}
b) {(1,1), (1,2), (2,1), (3,3), (4,3), (4,4)}
c) {,(1,1), (1,2), (2,1), (2,3), (3,4)}
d) {(0,1), (1,1), (2,3), (2,2), (3,2), (3,1)}

Unit –V Graph Theory

1. In the given graph identify the cut vertices.

A) B and E

B) C and D

C) A and E

D) C and B

2. A graph is bipartite if and only if its chromatic number is .


A. 1

B. 2

C. odd

D. even

3. The given Graph is regular. State whether True or False

A) True

B) False

4.A graph is planar if and only if it does not contain .

A. sub graphs homeomorphic to k3 & k3,3

B. sub graphs isomorphic to k5 or k3,3

C. sub graphs isomorphic to k3 & k3,3

D. sub graphs homeomorphic to k5 or k3,3

5.A connected planar graph having 6 vertices, 7 edges contains regions.

A) 15

B) 3

C) 1

D) 11

6. Kn denotes graph.

A. regular

B. simple
C. complete

D. null

7. Hamilton cycle is a cycle that contains every of G.

A. path

B. cycle

C. vertex

D. edge

8.A graph with all vertices having equal degree is known as a

A) Multi Graph

B ) Regular Graph

C) Simple Graph

D) Complete Graph

9. Which of the following ways can be used to represent a graph?

A) Adjacency List and Adjacency Matrix

B) Incidence Matrix

C) Adjacency List, Adjacency Matrix as well as Incidence Matrix

D) No way to represent

10. Minimum number of unique colours required for vertex colouring of a graph is called?

A) Vertex matching

B) Chromatic index

C) Chromatic number

D) Color number

11. The length of Hamiltonian Path in a connected graph of n vertices is

a) n–1
b) n

c) n+1

d) n/2

12). A graph with no edges is known as empty graph. Empty graph is also known as... ?

a. Trivial graph

b. Regular graph

c. Bipartite graph

d. None of the mentioned

13. In a graph if e=(u, v) means

A. u is adjacent to v but v is not adjacent to u

B. e begins at u and ends at v

C. u is processor and v is successor

D. both b and c

14. The maximum degree of any vertex in a simple graph with n vertices

is A. n–1

B. n+1

C.2n–1

D. n

15. Which of these adjacency matrices represents a simple

graph? A) [ [1, 0, 0], [0, 1, 0], [0, 1, 1] ]

B) [ [1, 1, 1], [1, 1, 1], [1, 1, 1] ]

C) [ [0, 0, 1], [0, 0, 0], [0, 0, 1] ]


D) [ [0, 0, 1], [1, 0, 1], [1, 0, 0] ]

16.In a graph if e=[u, v], Then u and v are called

A. Endpoints of e

B. Adjacent nodes

C. Neighbors

D. All of above

17. The problem of finding a path in a graph that visits every vertex exactly once is

called? A) Hamiltonian path problem

B) Hamiltonian cycle problem

C) Subset sum problem

D) Turnpike reconstruction problem

18. What would be the number of zeros in the adjacency matrix of the given graph?

a) 10
b) 6
c) 16
d) 0

19. Dijkstras Algorithm will work for both negative and positive

weights? a)True

b) False

20. A graph with one vertex and no edges is:

a)multigraph

b) Digraph
c) Isolated graph

d) Trivial graph

21. A complete graph of n vertices should have edges.

a) n-1

b) n

c) n(n-1)/2

d) n(n+1)/2

22. A Euler graph is one in which

a) Only two vertices are of odd degree and rests are even

b) Only two vertices are of even degree and rests are odd

c) All the vertices are of odd degree

d) All the vertices are of even degree

23. Which one of the following is TRUE for any simple connected undirected graph with
more than 2 vertices?
a) No two vertices have the same degree.
b) At least two vertices have the same degree.
c) At least three vertices have the same degree.
d) All vertices have the same degree.

24. A Graph is a collection of

a) Rows & columns

b) Vertices & edges

c) Equations

d) None of the mentioned

25. The total number of degrees in a graph is 20. How many edges does it

have? a)40

b) 20

c) 10
d) 5

26. The degree of any vertex of graph is.....?

a. The number of edges incident with vertex

b. Number of vertex in a graph

c. Number of vertices adjacent to that vertex

d. Number of edges in a graph

27. The maximum degree of any vertex in a simple graph with n vertices is

a. n–1

b. n+1

c. 2n–1

d. n

28. Choose the most appropriate definition of plane graph

a) A graph drawn in a plane in such a way that any pair of edges meet only at their end vertices

b) A graph drawn in a plane in such a way that if the vertex set of graph can be partitioned into
two non - empty disjoint subset X and Y in such a way that each edge of G has one end in X
and one end in Y

c) A simple graph which is Isomorphic to Hamiltonian graph

d) None of the mentioned

29 Suppose v is an isolated vertex in a graph, then the degree of v is

a) 0

b) 1

c) 2

d) 3

30. For the following Graphs which statement is correct?


a) K4 is planar while Q3 is not
b) Both K4 and Q3 are planar
c) Q3 is planar while K4 is not
d) Neither K4 nor Q3 are planar
A relation is reflexive if in the graphical representation of relation, every node
has a loop.
Select one:
a.False

b.True

The correct answer is: True

Question 2
which of these adjacency matrices represents a simple graph?
Select one:
a. [ [1, 0, 0], [0, 1, 0], [0, 1, 1] ]

b. [ [0, 0, 1], [1, 0, 1], [1, 0, 0] ]

c. [ [0, 0, 1], [0, 0, 0], [0, 0, 1] ]

d. [ [1, 1, 1], [1, 1, 1], [1, 1, 1] ]

The correct answer is: [ [0, 0, 1], [1, 0, 1], [1, 0, 0] ]

Question 3
The problem of finding a path in a graph that visits every vertex exactly
once is called?
Select one:
a.Hamiltonian path problem

b.Turnpike reconstruction problem

c. Hamiltonian cycle problem

d.Subset sum problem

The correct answer is: Hamiltonian path problem

Question 4
In a graph if e=[u, v], Then u and v are called
Select one:
a.Endpoints of e

b.Adjacent nodes

c. Neighbors
d.All of

above

Feedback

The correct answer is: All of above

Question 5
Which of the following properties does a simple graph not hold?
Select one:
a.Must be unweighted

b.Must be connected

c. Must have no multiple edges

d.Must have no loops or multiple

edges Feedback

The correct answer is: Must be connected

Question 6
The relation = is a partial order if it is
Select one:
a.Asymmetric, transitive

b.Reflexive, symmetric

c. Irreflexive and transitive

d.Reflexive, ant symmetric and

transitive Feedback

The correct answer is: Reflexive, ant symmetric and transitive

Question 7
The transitive closure of the relation {(0,1), (1,2), (2,2), (3,4), (5,3), (5,4)} on the
set {1, 2,
3, 4, 5} is
Select one:
a. {(0,1), (1,1), (2,2), (5,3), (5,4)}

b. {(0,1), (1,2), (2,2), (3,4)}

c. {(0,1), (0,2), (1,2), (2,2), (3,4), (5,3), (5,4)}


d. {(0,0), (1,1), (2,2), (3,3), (4,4), (5,5)}

The correct answer is: {(0,1), (0,2), (1,2), (2,2), (3,4), (5,3), (5,4)}

Question 8
If a set A has 8 elements and a set B has 10 elements, how many relations are
there from A to B?
Select one:
a. 380

b. 290

c. 280

d. 164

The correct answer is: 280

Question 9
Suppose a relation R = {(3, 3), (5, 5), (5, 3), (5, 5), (6, 6)} on S = {3, 5, 6}. Here
R is known as
Select one:
a.Equivalence relation

b.Symmetric relation

c. Transitive relation

d.Reflexive relation

The correct answer is: Equivalence relation

Question 10
Minimum number of unique colours required for vertex colouring of a graph
is called?
Select one:
a.Color number

b.Chromatic number

c. Vertex matching

d.Chromatic index

The correct answer is: Chromatic number


Question 11
Which of the following is an equivalence relation on R, for a, b ? Z?
Select one:
a. (a-B) ? Z

b. (2c3)/3 ? Z

c. (ab+cD)/2 ? Z

d. (a2+C) ? Z

The correct answer is: (a2+C) ? Z

Question 12
A graph with no edges is known as empty graph. Empty graph is also known
as... ?
Select one:
a.Trivial graph

b.None of these

c. Regular graph

d.Bipartite graph

The correct answer is: Trivial graph

Question 13
In the given graph identify the cut vertices.

Select one:
a.A and E

b.C and B

c. B and E

d.C and D

The correct answer is: C and B

Question 14
In a graph if e=(u, v) means
Select one:
a.u is adjacent to v but v is not adjacent to u

b.both b and c

c. u is processor and v is successor

d.e begins at u and ends at v

The correct answer is: both b and c

Question 15
A connected planar graph having 6 vertices, 7 edges contains regions.
Select one:
a.15

b.1

c. 3

d.11

The correct answer is: 3

Question 16
Let R be an equivalence relation on a set S. Then R need NOT satisfy the
property:
Select one:
a.Reflexive

b.Transitive

c. Anti-symmetric

d.Symmetric

The correct answer is: Anti-symmetric

Question 17
The maximum degree of any vertex in a simple graph with n vertices is
Select one:
a.n

b.n+1
Question 18
A graph with all vertices having equal degree is known as a
Select one:
a.Regular Graph

b.Simple Graph

c. Complete Graph

d.Multi Graph

The correct answer is: Regular Graph

Question 19
Which of the following ways can be used to represent a graph?
Select one:
a.Incidence Matrix

b.Adjacency List and Adjacency Matrix

c. Adjacency List, Adjacency Matrix as well as Incidence Matrix

d.No way to represent

The correct answer is: Adjacency List, Adjacency Matrix as well as Incidence Matrix

Question 20
In a simple graph, the number of edges is equal to twice the sum of the
degrees of the vertices.
Select one:
a.False

b.True

The correct answer is: False

Question 21
The length of Hamiltonian Path in a connected graph of n vertices is

a.n/2

b.n

c. n+1
Question 22
A Poset in which every pair of elements has both a least upper bound and a
greatest lower bound is termed as
Select one:
a.Lattice

b.Sub lattice

c. Walk

d.Trail

The correct answer is: Lattice

Question 23
If a simple graph G, contains n vertices and m edges, the number of edges
in the Graph G'(Complement of G) is
Select one:
a. ( n*n-n-2*m)/2

b. (n*n-n-2*m)/2

c. (n*n+n+2*m)/2

d. (n*n-n+2*m)/2

The correct answer is: (n*n-n-2*m)/2

Question 24
The given Graph is regular.

Select one:
a.True

b.False

The correct answer is: True

Question 25
How many binary relations are there on a set S with 9 distinct elements?
Select one:
a. 260
b. 2100

c. 281

d. 290

The correct answer is: 281

Question 26
The less-than relation, <, on a set of real numbers is
Select one:
a.A partial ordering since it is antisymmetric and reflexive

b.Not a partial ordering because it is not antisymmetric and reflexive

c. a partial ordering since it is asymmetric and reflexive

d.not a partial ordering because it is not asymmetric and irreflexive equals


antisymmetric

The correct answer is: not a partial ordering because it is not asymmetric and irreflexive
equals antisymmetric

Question 27
The binary relation {(1,1), (2,1), (2,2), (2,3), (2,4), (3,1), (3,2)} on the set {1, 2,
3} is
Select one:
a.Irreflexive, symmetric and transitive

b.Neither reflexive, nor irreflexive but transitive

c. Reflexive, symmetric and transitive

d.Irreflexive and ant symmetric

The correct answer is: Neither reflexive, nor irreflexive but transitive

Question 28
The number of symmetric relations on a set with 15 distinct elements is
Select one:
a. 250

b. 2196

c. 278

d. 2320
The correct answer is: 2196

Question 29
What is the number of edges present in a complete graph having n vertices?
Select one:
a.n

b.Information given is insufficient

c. (n*(n-1))/2

d. (n*(n+1))/2

The correct answer is: (n*(n-1))/2

Question 30
Let A = {1, 2, 3} and B = {u, v, w}. Then which of the following is NOT a
relation from A to B?
Select one:
a. {}

b. {(1, u), (1, v), (1, w)}

c. {(3, v)}

d. {(1, u), (3, 2), (2, w)}

The correct answer is: {(1, u), (3, 2), (2, w)}
The function f(x)=x+1 from the set of integers to itself is onto. Is it True or False?
Select one:
True

False
The correct answer is 'True'.

Question 2
State whether the given statement is true or false
The range of function f(x) = sin(x) is (-∞,
∞).
Select one:
True

False

The correct answer is 'False'.

Question 3
State whether the given statement is true or false
Onto function are known as injection.
Select one:
True

False

The correct answer is 'False'.

Question 4
The shaded area of figure is best described by

Select one:
a.A‘ (Complement of A)

b.A U B -B

c. A ∩ B
d.B

The correct answer is: A U B -B

Question 5
Let the sequence be 1, 3, 5, 7, 9……… then this sequence is
Select one:
a.An airthmetic sequence

b.A geometic progression

c. A harmonic sequence

d.None of the mentioned.

The correct answer is: An airthmetic sequence

Question 6
If f is a function defined from R to R , is given by f(x) = 3x – 5 then f –1(x) is
given by:
Select one:
a. 1/(3x-5)

b. (x+5)/3

c. does not exist since it is not a bijection

d. none of the mentioned

The correct answer is: (x+5)/3

Question 7
A is an ordered collection of objects.
Select one:
a.Function

b.Proposition

c. Relation

d.Set

The correct answer is: Set

Question 8
A floor function map a real number to :
Select one:
a.smallest previous integer

b.greatest previous integer

c. smallest following integer

d.none of the mentioned

The correct answer is: greatest previous integer

Question 9
A function f(x) is defined from A to B then f -1 is defined :
Select one:
a.from A to B

b.From B to A

c. depends on the inverse of function

d.None of the mentioned

The correct answer is: From B to A

Question 10
A mapping f : X -> Y is one one if :
Select one:
a.f(x1) ≠ f(x2) for all x1, x2 in X.

b. If f(x1) = f(x2) then x1 = x2 for all x1, x2 in X.

c. f(x1) = f(x2) for all x1, x2 in X.

d.None of the above

The correct answer is: If f(x1) = f(x2) then x1 = x2 for all x1, x2 in X.

Question 11
An injection is a function which is :
Select one:
a.many-one

b.one-one

c. on to

d.None of the above

The correct answer is: one-one


Question 12
Every set is a of itself
Select one:
a.None of these

b.Proper Subset

c. Improper Subset

d.Complement

The correct answer is: Improper Subset

Question 13
Floor(2.4) + Ceil(2.9) is equal to :
Select one:
a.5

b.none of the mentioned

c. 4

d.6

The correct answer is: 5

Question 14
For an inverse to exist it is necessary that a function should be :
Select one:
a.surjection

b.injection

c. bijection

d.none of the mentioned

The correct answer is: bijection

Question 15
If A is {{Φ}, {Φ, {Φ}}, then the power set of A has how many element?
Select one:
a.2

b.4
c. 6

d.8

The correct answer is: 4

Question 16
If A is not equal to B, then the Cartesian product ?
Select one:
a.A x B not equal B x A

b.A x B equal B x A

c. Not possible

d.None of these.

The correct answer is: A x B not equal B x A

Question 17
If n(A X B) = n(B X A) = 36 then which of the following may hold true?
Select one:
a. n(A)=2, n(B)=18

b. n(A)=9, n(B)=4

c. n(A)=6, n(b)=6

d. None of the mentioned

The correct answer is: n(A)=6, n(b)=6

Question 18
If n(A)=20 and n(B)=30 and n(A U B) = 40 then n(A n B) is
Select one:
a.40

b.10

c. 30

d.20

The correct answer is: 10

Question 19
If set A has 4 elements and B has 3 elements then set n(A X B) is
Select one:
a.12

b.24

c. 7

d.14

The correct answer is: 12

Question 20
If the number of subsets of a set are 4 then the number of elements in that
sets are
Select one:
a.2

b.3

c. 1

d.4

The correct answer is: 2

Question 21
Power set of empty set has exactly subset.
Select one:
a.Two

b.Three

c. Zero

d.One

The correct answer is: One

Question 22
The Cartesian Product B x A is equal to the Cartesian product A x B. Is it
True or False?
Select one:
a.True

b.False

The correct answer is: False


Question 23
The difference of {1, 2, 3} and {1, 2, 5} is the set
Select one:
a. {1}

b. {3}

c. {2}

d. {5}

The correct answer is: {3}

Question 24
The number of elements in the Power set P(S) of the set S = [ [ Φ] , 1, [ 2, 3 ]]
is
Select one:
a.6

b.2

c. 4

d.8

The correct answer is: 8

Question 25
The number of proper subsets of the set {1, 2, 3} is.
Select one:
a.8

b.7

c. 6

d.5

The correct answer is: 6

Question 26
The series a,(a+b)/2, b is in
Select one:
a.HP

b.AP
c. NONE

d.GP

The correct answer is: AP

Question 27
The value of ⌊1/2.⌊5/2⌋ ⌋ is

Select one:
A.2

B. 3

C. 0.5

D. 1

The correct answer is: 1

Question 28
What is the Cardinality of the Power set of the set {0, 1, 2}.
Select one:
a.6

b.8

c. 7

d.9

The correct answer is: 8

Question 29
Which of the following statements regarding sets is false ?
Select one:
a.A X B ? B X A

b.n(A X B) = n(A) * n(B)

c. All of the mentioned

d.A X B = B X A

The correct answer is: A X B = B X A


Question 30
Which sets are not empty?
Select one:
a.{x : x is a multiple of 2 and is odd}

b.{x: x is a even prime greater than 3}

c. { x: x is a prime number less than 5 and is odd}

d.{x: x is an even number and x+3 is even}

The correct answer is: { x: x is a prime number less than 5 and is odd}
Determine the solution for the recurrence relation a n = 6an-1−8an-2 provided initial
conditions a0=3 and a1=5.
Select one:
a. an = 4 * 2n – 3n

b. an = 3 * 7n – 5*3n

c. an = 5 * 7n

d. an = 3! * 5n
The correct answer is: an = 3 * 7n – 5*3n

Question 2

Determine the solution for the recurrence relation b n=8bn-1−12bn-2 with b0=3 and b1=4.
Select one:
a. 2/3*7n-5*4n

b. 7/2*2n−1/2*6n

c. 4!*6n

d. 2/8n
The correct answer is: 7/2*2n−1/2*6n

Question 3
Determine the value of a2 for the recurrence relation a n = 17an-1 + 30n with a0=3.
Select one:
a. 1437

b. 238

c. 5484

d. 4387
The correct answer is: 1437

Question 4
Find the sequence generated by 1/1−x2−x4.,assume that 1, 1, 2, 3, 5, 8,… has
generating function 1/1−x−x2.
Select one:
a. 1, 1, 2, 2, 4, 6, 8,…..

b. 1, 4, 3, 5, 7,…..
c. 0, 1, 2, 3, 5, 8,…

d. 0, 0, 1, 1, 2, 3, 5, 8,...…
The correct answer is: 0, 0, 1, 1, 2, 3, 5, 8,...…

Question 5
The numbers between 1 and 520, including both, are divisible by 2 or 6 is
Select one:
a. 349

b. 54

c. 213

d. 303
The correct answer is: 303

Question 6
The solution to the recurrence relation an=an-1+2n, with initial term a0=2 are

Select one:
a. 5*(n+1)/2

b. 3n2

c. 2(1+n)

d. 4n+7
The correct answer is: 2(1+n)

Question 7
The sum of all integers from 1 to 520 that are multiples of 4 or 5?
Select one:
a. 187

b. 208

c. 421

d. 52.
The correct answer is: 208

Question 8
There are 70 patients admitted in a hospital in which 29 are diagnosed with typhoid,
32 with malaria, and 14 with both typhoid and malaria. Find the number of patients
diagnosed with typhoid or malaria or both.
Select one:
a. 39

b. 17

c. 47

d. 53
The correct answer is: 47

Question 9
Using the inclusion-exclusion principle, find the number of integers from a set of 1-
100 that are not divisible by 2, 3 and 5
Select one:
a. 33

b. 26

c. 22

d. 25
The correct answer is: 26

Question 10
What is the generating function for generating series 1, 2, 3, 4, 5,… ?
Select one:
a. 2/(1−3x)

b. 1/(1+x)

c. 1/(1−x) 2

d. 1(1−x2)
The correct answer is: 1/(1−x) 2

Question 11
What is the generating function for the sequence 1, 6, 16, 216,….?
Select one:
a. (1+6x)/x3

b. 1(1−6x)

c. 1(1−4x)
d. 1-6x2
The correct answer is: 1(1−6x)

Question 12
What is the recurrence relation for 1, 7, 31, 127, 499?
Select one:
a. bn+1=5bn-1+3

b. bn=4bn+7!

c. bn=4bn-1+3

d. bn=bn-1+1
The correct answer is: bn=4bn-1+3

Question 13
What is the recurrence relation for the sequence 1, 3, 7, 15, 31, 63,…?
Select one:
a. an = 3an-1−2an+2

b. an = 3an-1−2an-2

c. an = 3an-1−2an-1

d. an = 3an-1−2an-3
The correct answer is: an = 3an-1−2an-2

Question 14
What is the sequence depicted by the generating series 4 + 15x 2 + 10x3 + 25x5 +
16x6+⋯?
Select one:
a. 0, 4, 15, 10, 16, 25,…

b. 10, 4, 0, 16, 25, …

c. 4, 0, 15, 10, 25, 16,…

d. 4, 10, 15, 25,…


The correct answer is: 4, 0, 15, 10, 25, 16,…

Question 15
What is the solution to the recurrence relation an=5an-1+6an-2?
Select one:
a. 2n2
b. 6n

c. (3/2)n

d. n!*3
The correct answer is: 6n

Question 16
What will be the sequence generated by the generating function 4x/(1-x) 2?
Select one:
a. 12, 16, 20, 24,…

b. 1, 3, 5, 7, 9,…

c. 0, 4, 8, 12, 16, 20,…

d. 0, 1, 1, 3, 5, 8, 13,…
The correct answer is: 0, 4, 8, 12, 16, 20,…

Question 17
Which of the following is a correct representation of inclusion exclusion principle (|
A,B| represents intersection of sets A,B)?
Select one:
a. |A U B|=|A|+|B|-|A,B|

b. |A,B|=|A|+|B|+|A U B|

c. |A U B|=|A|+|B|+|A,B|

d. |A,B|=|A|+|B|-|A U B|
The correct answer is: |A U B|=|A|+|B|-|A,B|

Question 18
Which of the following is not an application of inclusion-exclusion principle?
Select one:
a. Maximum flow problem

b. Graph coloring

c. Counting intersections

d. Matching of bipartite graphs


The correct answer is: Maximum flow problem

Question 19
Which one of the following problem types does inclusion-exclusion principle belong
to?
Select one:
a. Combinatorial problems

b. Graph problems

c. Numerical problems

d. String processing problems


The correct answer is: Combinatorial problems

Question 20
With reference to the given Venn diagram, what is the formula for computing
|AUBUC| (where |x, y| represents intersection of sets x and y)?

Select one:
a. |A U B U C|=|A|+|B|+|C| + |A,B| + |A,C| + |B,C|+|A, B,C|

b. |A U B U C|=|A|+|B|+|C|-|A,B|-|A,C|-|B,C|+|A, B,C|

c. |A, B,C|=|A|+|B|+|C|-|A U B|-|A U C|-|B U C|+|A U B U C|

d. |A, B,C|=|A|+|B|+|C|+|A,B|-|A,C|+|B,C|+|A U B U C|
The correct answer is: |A U B U C|=|A|+|B|+|C|-|A,B|-|A,C|-|B,C|+|A, B,C|
Discrete Mathematical Structures ONLINE
EXAMIANTION' QUESTIONS
1. In the given graph identify the cut vertices.

A) Band E
B) Cand D
C) A and E
D) C and B

ANSWER: D
2. What is the num.ber of edges present in a complete graph having n vertices?
A) (n*(n+1))/2
B) (n*(n-1))/2
C) n
D) Information given is insufficient
ANSWER: B
3. The given Graph is regular.

A) True
B) False
ANSWER: A
4. In a simple graph, the number of edges is equa! to twice the sum of the degrees of the
vertices.
A) True
B) False

ANSWER: B
5. A connected planar graph having 6 vertices, 7 edges contains regions.
A) 15
B) 3
C) 1
D) 11
ANSWER: B
6. If a simple graph G, contains n vertices and m edges, the number of edges in the Graph
G'(Complement of G) is
A) (n*n-n-2*rn)/2
B) (n*n+n+2*m)/2
C) ( n*n-n-2*m)/2
D) (n*n-n+2”m)/2

ANSWER: A
7. Which of the following properties does a simple graph not hold?
A) Must be connected
B) Must be unweighted
C) Must have no loops or multiple edges
D) Must have no multiple
edges ANSWER: A

8. A graph with all vertices having equal degree is known as a


A) Multi Graph
B ) Regular Graph
C) Simple Graph
D) Complete G.raph

ANSWER: B

Scanned with CamScanner


9. Which of the following ways can be used to represent a graph?
A) Adjacency List and Adjacency Matrix
B) Incidence Matrix
C) Adjacency List, Adjacency Matrix as well as Incidence Matrix
D) No way to

represent ANSWER: C

10. Minimum number of unique colours required for vertex colouring of a graph is called?
A) Vertex matching
B) Chromatic index
C) Chromatic number
D) Color number
ANSWER : C

11. The length of Hamiltonian Path in a connected graph of n vertices is


A) n—1
B) n
C) n-r1
D) n/2

ANSWER: A
121. A graph with no edges is known as empty graph. Empty graph is also known as... ?
A. Trivial graph
B. Regular graph
C. Bipartite graph
D. None of
these View
Answer
ANSWER: A
13. In a graph if e=(u, v) means
A. u is adjacent to v but v is not adjacent to u
B. e begins at u and ends at v
C. u is processor and v is successor
D. both b and
c ANSWER: D
14. The maximum degree of any vertex in a simple graph with n vertices is
A. A n—1
B. B n+1
C. C 2n—1
D. D n
View Answer
ANSWER: A
15. which of these adjacency matrices represents a simple
graph? A) [ [1, 0, 0], [0, 1, 0], [0, 1, 1] ]
B) [ [1, 1, I{, [1, 1, 1], [1, 1, 1] ]
C) [ [0, 0, 1], [0, 0, 0], [0, 0, 1] ]
D) [ [0, 0, 1], [1, 0, 1], [1, 0,
0] ] ANSWER: D

16. In a graph if e=[u, v], Then u and v are called


A. Endpoints of e
B. Adjacent nodes
C.Neighbors
D. All of

above

ANSWER: D
Scanned with CamScanner
17. The binary relation ((1,1), (2,1), (2,2), (2,3), (2,4), (3,1), (3,2)) on the set (1, 2, 3} is
A) Reflexive, symmetric and transitive
B) Irreflexive, symmetric and transitive
C) Neither reflexive, nor irreflexive but transitive
D ) Irreflexive and ant symmetric
ANSWER : C
18. The relation = is a partial order if it is
A) Reflexive, ant symmetric and transitive
B) Reflexive, symmetric
C) Asymmetric, transitive
D) Irreflexive and transitive
ANSWER: A
19. A Poset in which every pair of elements has both a least upper bound and a greatest lower
bound is termed as
A) Sub lattice
B) Lattice
C) Trail
D) Walk
ANSWER: B
20. Let R be an equivalence relation on a set S. Then R need NOT satisfy the property:
A) Reflexive
B) Symmetric
C) Anti-symmetric
D) Transitive
ANSWER: C
21. How many binary relations are there on a set S with 9 disti'nct
elements? A) 290
B) 2100
C) 281
D) 260
ANSWER: C

22. The number of symmetric relations on a set with 15 distinct elements is


A) 2196
B) 250
C) 2320
D) 278
ANSWER: A
23. If a set A has 8 elements and a set B has 10 elements, how many relations are there
from A to B?
A) 290
B) 380
C) 164
D) 280
ANSWER :
D
24. A relation is reflexive if in the graphical representation of relation, every node has a loop.
A) True
B) False
ANSWER: A
25. Let A = {1, 2, 3} and B = (u, v, w}. Then which of the following is NOT a relation from A to B?
A) {}
B) {(1, u), (3, 2), (2, w)}
C) ((3, v))
D) ((1, u), (1, v), (1,

w)) ANSWER: B
Scanned with CamScanner
26. The transitive closure of the relation ((0,1), (1,2), (2,2), (3,4), (5,3), (5,4)} on the set (1, 2, 3, 4,
5) is
A) ((0,1), (1,2), (2,2), (3,4))
B) {(0,0), (1,1), (2,2), (3,3), (4,4), (5,5))
C) ((0,1), (1,1), (2,2), (5,3), (5,4))
D) ((0,1), (0,2), (1,2), (2,2), (3,4), (5,3),
(5,4)} ANSWER : D

27. The less-than relation, <, on a set of real numbers is


A) not a partial ordering because it is not asymmetric and irreflexive equals antisymmetric
B) a partial ordering since it is asymmetric and reflexive
C) A partial ordering since it is antisymmetric and reflexive
D) Not a partial ordering because it is not antisymmetric and reflexive
ANSWER: A

28. Suppose a relation R = ((3, 3), (5, 5), (5, 3), (5, 5), (6, 6)) on S = (3, 5, 6). Here R is known as

A) Equivalence relation
B) Reflexive relation
C) Symmetric relation
D) Transitive relation
ANSWER: A
29. Which of the following is an equivalence relation on R, for a, b ?
Z? A ) (a-B) ? Z
B) (a2+C) ? Z
C) (ab+cD)/2 ? Z
D) (2c3)/3 ? Z
ANSWER: B
30. The problem of finding a path in a graph that visits every vertex exactly once is called?
A) Hamiltonian path problem
B) Hamiltonian cycle problem
C) Subset sum problem
D) Turnpike reconstruction problem

ANSWER : A

Scanned with CamScanner


Scanned with CamScanner
Scanned with CamScanner
Question 1

A → (A ∨ q) is a

Select one:

a. Tautology

b. None of the mentioned

c. Contradiction

d. Contingency

The correct answer is:

Tautology Question 2

Express, “The difference of a real number and itself is zero” using required operators.

Select one:

a. ∀x(x − x! = 0)

b. ∀x(x − x = 0)

c. ∀x∀y(x − y =

0)

d. ∃x(x − x = 0)

The correct answer is: ∀x(x − x =

0) Question 3

Let P, Q, R be true, false, false, respectively, which of the following

is true Select one:

a. P∧(Q∧~R)

b. (P->Q)∧~R

c. Q<->(P∧R)

d. P<->(QvR)

The correct answer is: Q<->(P∧R)

Question 4
Let P: We should be honest., Q: We should be dedicated .,R: We should be

overconfident. Then ‘We should be honest or dedicated but not overconfident.’ is

best represented by: Select one:

a. ~P V ~Q V R

b. P ∧ ~Q ∧ R

c. P V Q ∧ R

d. P V Q ∧ ~R

The correct answer is: P V Q ∧

~R Question 5

Let T (x, y) mean that student x likes dish y, where the domain for x consists of all students at
your school and the domain for y consists of all dishes. Express ¬T (Amit, South Indian) by a simple
English sentence.

Select one:

a. All students does not like South Indian dishes

b. Amit does not like South Indian people.

c. Amit does not like South Indian dishes.

d. Amit does not like some dishes.

The correct answer is: Amit does not like some dishes.

Question 6

p → q is logically equivalent

to: Select one:

a. ¬p ∨ ¬q

b. p ∨ ¬q

c. ¬p ∨ q

d. ¬p ∧ q

The correct answer is: ¬p ∨ q


Question 7

The inverse of p → q is the

proposition: Select one:

a. ¬q → p

b. ¬p → ¬q

c. ¬q → ¬p

d. q → p

The correct answer is: ¬p →

¬q Question 8

Translate ∀x∃y(x < y) in English, considering domain as real number for both the

variable. Select one:

a. For all real number x there exists a real number y such that x is less than y

b. For every real number y there exists a real number x such that x is less than y

c. For some real number x there exists a real number y such that x is less than y

d. For each and every real number x and y such that x is less than y

The correct answer is: For all real number x there exists a real number y such that x is

less than y Question 9

What are the contrapositive of the conditional statement “I come to class whenever there is
going to be a test.""

Select one:

a. “If I come to class, then there will be a test.”

b. “If there is going to be a test, then I don’t come to class.”

c. “If there is not going to be a test, then I don’t come to class.”

d. “If I do not come to class, then there will not be a test.”

The correct answer is: “If I do not come to class, then there will not be a test.”
Question 10

What is the negation of the statement A->(B v(or)

C)? Select one:

a. A ∧ ~B ∧ ~C

b. A->B->C

c. ~A ∧ B v C

d. None of the mentioned

The correct answer is: A ∧ ~B ∧

~C Question 11

When to proof P→Q true, we proof P false, that type of proof is

known as Select one:

a. Direct Proofs

b. Contrapositive proofs

c. Vaccous Proofs

d. Mathematical Induction

The correct answer is: Vaccous Proofs

Question 12

Which rule of inference is used in each of these arguments, “If it hailstoday, the local office will
be closed. The local office is not closed today. Thus, it did not hailed today.”

Select one:

a. Modus tollens

b. Conjunction

c. Hypothetical syllogism

d. Simplification

The correct answer is: Modus

tollens Question 13
Which rule of inference is used,”Bhavika will work in an enterprise this summer. Therefore, this
summer Bhavika will work in an enterprise or he will go to beach.”

Select one:

a. Simplification

b. conjunction

C. addition

d. Disjunctive syllogism

The correct answer is:

addition Question 14

Determine the truth value of statement ∃n (4n = 3n) if the domain consists of all

integers. Select one:

True

False

The correct answer is

'True'. Question 15

Which of the arguments is not valid in proving sum of two odd number is

not odd. Select one:

a. 3 + 3 = 6 ,hence true for all

b. 2n +1 + 2m +1 = 2(n+m+1) hence true for all

c. All of the Mentioned

d. None of the mentioned

The correct answer is: 3 + 3 = 6 ,hence true

for all Question 16

“Parul is out for a trip or it is not snowing” and “It is snowing or Raju is playing chess”

imply that Select one:

a. Parul is out for trip.


b. Raju is playing chess

c. Parul is out for a trip and Raju is playing chess.

d. Parul is out for a trip or Raju is playing chess.

The correct answer is: Parul is out for a trip or Raju is playing chess.

Question 17

(P->Q)-> (^Q) is .

Select one:

a. well formed formula

b. Contradiction

c. not a well formed formula

d. tautology

The correct answer is: not a well formed

formula Question 18

¬ (p ↔ q) is logically equivalent

to: Select one:

a. q↔p

b. ¬q↔¬p

c. ¬p↔¬q

d. p↔¬q

The correct answer is: p↔¬q

Question 19

A proof covering all the possible cases, such type of proofs are known

as Select one:

a. Vacuous proof

b. Proof by Contradiction
c. Exhaustive proof

d. Direct proof

The correct answer is: Exhaustive

proof Question 20

Consider the following statements

A: Raju should exercise.

B: Raju is not a decent table tennis

player. C: Raju wants to play good

table tennis.

The symbolic form of “Raju is not a decent table tennis player and if he wants to play good
table tennis then he should exercise.” is

Select one:

a. A->B->C

b. B∧(C->A)

c. C->B∧A

d. B<->A∧C

The correct answer is: B∧(C->A)

Question 21

Consider the following two statements.

S1: If a candidate is known to be corrupt, then he will not be

elected. S2: If a candidate is kind, he will be elected.

Which one of the following statements follows from S1 and S2 as per sound inference rules of logic?

Select one:

a. If a person is known to be corrupt he is kind

b. if a person is not known to be corrupt he is not kind

c. If a person is kind he is not known to be corrupt


d. if a person is not kind he is not known to be corrupt

The correct answer is: If a person is kind he is not known to be

corrupt Question 22

If there are n distinct components in a statement then there are combinations of values in
the truth table.

Select one:

a. n+1

b. n+2

c. n

d. 2^n

The correct answer is:

2^n Question 23

In Propositional Logic, given P and P → Q, we can infer

. Select one:

a. ~ P ∧ Q

b. ~ Q

c. Q

d. P ∧ Q

The correct answer is:

Q Question 24

In proving √5 as irrational, we begin with assumption √5 is rational in which type of

proof? Select one:

a. Proof by Contradiction

b. Mathematical Induction

c. Direct proof

d. Vacuous proof
The correct answer is: Proof by

Contradiction Question 25

Let domain of m includes all students , P (m) be the statement “m spends more than 2 hours
in playing polo”. Express ∀m ¬P (m) quantification in English.

Select one:

a. A student is there who spends more than 2 hours in playing polo

b. There is a student who does not spend more than 2 hours in playing polo

c. All students spends more than 2 hours in playing polo

d. No student spends more than 2 hours in playing polo

The correct answer is: No student spends more than 2 hours in playing polo

Question 26

Let L(x, y) be the statement “x loves y,” where the domain for both x and y consists of all people in
the world.

Use quantifiers to express , “Joy is loved by

everyone.” Select one:

a. ∀x L(x, Joy)

b. ∀y L(Joy,y)

c. ∃y∀x L(x, y)

d. ∃x ¬L(Joy, x).

The correct answer is: ∀x L(x,

Joy) Question 27

Suppose the predicate F(x, y, t) is used to represent the statement that person x can fool person y
at
time t. which one of the statements below expresses best the meaning of the formula

∀x∃y∃t(¬F(x, y, t))? Select one:

a. Every one can fool some person at some time

b. No one can foll every one all the time

c. Every one can not fool some one all the time
d. No one can fool some one at some time

The correct answer is: No one can foll every one all the

time Question 28

The premises (p ∧ q) ∨ r and r → s imply which of the

conclusion? Select one:

a. p ∨ r

b. q ∨ s

c. q ∨ r

d. p ∨ s

The correct answer is: p

∨ s Question 29

Use quantifiers and predicates with more than one variable to express, “There is a pupil in this
lecture who has taken at least one course in Discrete Maths.”

Select one:

a. ∀x∀yP(x, y), where P (x, y) is “x has taken y,” the domain for x consists of all pupil in this
class, and the domain for y consists of all Discrete Maths lectures

b. ∃x∀yP(x, y), where P (x, y) is “x has taken y,” the domain for x consists of all pupil in this
class, and the domain for y consists of all Discrete Maths lectures

c. ∃x∃yP (x, y), where P (x, y) is “x has taken y,” the domain for x consists of all pupil in this
class, and the domain for y consists of all Discrete Maths lectures

∃x∃yP (x, y), where P (x, y) is “x has taken y,” the domain for x consists of all Discrete Maths
lectures, and the domain for y consists of all pupil in this class

d. ∃x∀yP(x, y), where P (x, y) is “x has taken y,” the domain for x consists of all pupil in this
class, and the domain for y consists of all Discrete Maths lectures
The correct answer is: ∃x∃yP (x, y), where P (x, y) is “x has taken y,” the domain for x consists of all
pupil in this class, and the domain for y consists of all Discrete Maths lectures

Question 30

What rules of inference are used in this argument?

“All students in this science class has taken a course in physics” and “Marry is a student in this class”
imply the conclusion “Marry has taken a course in physics.”

Select one:

a. Universal instantiation

b. Universal generalization

c. Existential instantiation

d. Existential generalization

The correct answer is: Universal instantiation


1. Which of the following statement is a proposition?
a) Get me a glass of milkshake
b) God bless you!
c) What is the time now?
d) The only odd prime number is 2 View
Answer

Answer: d
Explanation: Only this statement has got the truth value which is
false.
2. The truth value of ‘4+3=7 or 5 is not prime’.
a) False
b) True View
Answer

Answer: b
Explanation: Compound statement with ‘or’ is true when either of
the statement is true. Here the first part of the statement is
true, hence the whole is true.
3. Which of the following option is true?
a) If the Sun is a planet, elephants will fly
b) 3 +2 = 8 if 5-2 = 7
c) 1 > 3 and 3 is a positive integer
d) -2 > 3 or 3 is a negative integer View Answer

Answer: a
Explanation: Hypothesis is false, thus the whole statement is true.
4. What is the value of x after this statement, assuming the initial value of x is 5?
‘If x equals to one then x=x+2 else x=0’.
a) 1
b) 3
c) 0
d) 2
View Answer

Answer: c
Explanation: If condition is false so value decided according to else
condition.
5. Let P: I am in Bangalore.; Q: I love cricket.; then q -> p(q implies
p) is?
a) If I love cricket then I am in Bangalore
b) If I am in Bangalore then I love cricket
c) I am not in Bangalore
d) I love cricket View
Answer

Answer: a
Explanation: Q is hypothesis and P is conclusion. So the compound
statement will be if hypothesis then conclusion.
6. Let P: If Sahil bowls, Saurabh hits a century.; Q: If Raju bowls, Sahil gets out on first ball.
Now if P is true and Q is false then which of the following can be true?
a) Raju bowled and Sahil got out on first ball
b) Raju did not bowled
c) Sahil bowled and Saurabh hits a century
d) Sahil bowled and Saurabh got out View
Answer

Answer: c
Explanation: Either hypothesis should be false or both (hypothesis
and conclusion) should be true.
7. The truth value ‘9 is prime then 3 is even’.
a) False
b) True View
Answer
Answer: b
Explanation: The first part of the statement is false, hence whole
is true.
8. Let P: I am in Delhi.; Q: Delhi is clean.; then q ^ p(q and p) is?
a) Delhi is clean and I am in Delhi
b) Delhi is not clean or I am in Delhi
c) I am in Delhi and Delhi is not clean
d) Delhi is clean but I am in Mumbai View Answer

Answer: a
Explanation: Connector should be ‘and’, that is q and p.
9. Let P: This is a great website, Q: You should not come back here. Then ‘This is a great
website and you should come back here.’ is best represented by?
a) ~P V ~Q
b) P ∧ ~Q
c) P V Q
d) P ∧ Q
View Answer

Answer: b
Explanation: The second part of the statement is negated, hence
negation operator is used.
10. Let P: We should be honest., Q: We should be dedicated., R: We should be overconfident.
Then ‘We should be honest or dedicated but not overconfident.’ is best represented by?
a) ~P V ~Q V R
b) P ∧ ~Q ∧ R
c) P V Q ∧ R
d) P V Q ∧ ~R
View Answer

Answer: d
Explanation: The third part of the statement is negated, hence
negation operator is used, for (‘or’ –V) is used and for(’but’-
∧).

This set of Discrete Mathematics Multiple Choice Questions &


Answers (MCQs) focuses on “Logic and Bit Operations”.

1. Which of the following bits is the negation of the bits “010110”? a) 111001
b) 001001
c) 101001
d) 111111
View Answer

Answer: c
Explanation: Flip each of the bit to get the negation of the
required string.
2. Which of the following option is suitable, if A is “10110110”, B is”11100000” and C
is”10100000”?
a) C=A or B
b) C=~A
c) C=~B
d) C=A and B
View Answer

Answer: d
Explanation: Output of and is 1 when both other inputs are one.
3. How many bits string of length 4 are possible such that they contain
2 ones and 2 zeroes?
a) 4
b) 2
c) 5
d) 6
View Answer

Answer: d
Explanation: The strings are {0011, 0110, 1001, 1100, 1010 and 0101}.
4. If a bit string contains {0, 1} only, having length 5 has no more than 2 ones in it. Then how
many such bit strings are possible?
a) 14
b) 12
c) 15
d) 16
View Answer

Answer: d
Explanation: The total strings are 1(having no one in it) +5(having
1 one in it) +10 (having 2 ones in it) = 16.
5. If A is “001100” and B is “010101” then what is the value of A (Ex-or) B?
a) 000000
b) 111111
c) 001101
d) 011001
View Answer

Answer: d
Explanation: In Ex-or if both the inputs are same then output is 0
otherwise 1.
6. The Ex-nor of this string “01010101” with “11111111” is? a) 10101010
b) 00110100
c) 01010101
d) 10101001
View Answer

Answer: c
Explanation: In Ex-nor if both the inputs are same then output is 1
otherwise 0.
7. What is the one’s complement of this string “01010100”? a) 10101010
b) 00110101
c) 10101011
d) 10101001
View Answer

Answer: c
Explanation: Negate every bit in one’s complement.
8. What is the 2’s complement of this string “01010100”? a) 10101010
b) 00110100
c) 10101100
d) 10101001
View Answer

Answer: c
Explanation: In two’s complement negate every bit from left until
the first one from right is encountered.
9. If in a bits string of {0,1}, of length 4, such that no two ones are together. Then the total
number of such possible strings are?
a) 1
b) 5
c) 7
d) 4
View Answer

Answer: c
Explanation: Strings can be {1001, 1010, 0101, 1000, 0100, 0010, 0001}.
10. Let A: “010101”, B=?, If { A (Ex-or) B } is a resultant string of all ones then which of the
following statement regarding B is correct?
a) B is negation of A
b) B is 101010
c) {A (and) B} is a resultant string having all zeroes
d) All of the mentioned View
Answer

Answer: d
Explanation: In Ex-or both if both the inputs are the same then
output is 0 otherwise 1.

This set of Discrete Mathematics Interview Questions and Answers


for freshers focuses on “Logics – Implication and Double
Implications”.

1. Let P and Q be statements, then P<->Q is logically equivalent to

a) P<->~Q
b) ~P<->Q
c) ~P<->~Q
d) None of the mentioned
View Answer
Answer: c
Explanation: Both of them have same truth table, Hence they are equal.
2. What is the negation of the statement A->(B v(or) C)?
a) A ∧ ~B ∧ ~C
b) A->B->C
c) ~A ∧ B v C
d) None of the mentioned View
Answer

Answer: a
Explanation: A->P is logically equivalent to ~A v P.
3. The compound statement A-> (A->B) is false, then the truth values of A, B are respectively

a) T, T
b) F, T
c) T, F
d) F, F View
Answer

4. The statement which is logically equivalent to A∧ (and) B is?


a) A->B
b) ~A ∧ ~ B
c) A ∧ ~B
d) ~(A->~B)
View Answer

Answer: d
Explanation: The truth table of both statements are same.
5. Let P: We give a nice overall squad performance, Q: We will win the match.
Then the symbolic form of “We will win the match if and only if we
give a nice overall squad performance.“ is?
a) P v Q
b) Q ∧ P
c) Q<->P
d) ~P v Q
View Answer

Answer: c
Explanation: If and only if statements are bi-conditionals.
6. Let P, Q, R be true, false true, respectively, which of the following is true?
a) P∧Q∧R
b) P∧~Q∧~R
c) Q->(P∧R)
d) P->(Q∧R)
View Answer

Answer: c
Explanation: Hypothesis is false, hence statement is true.
7. “Match will be played only if it is not a humid day.” The negation of this statement is?
a) Match will be played but it is a humid day
b) Match will be played or it is a humid day
c) All of the mentioned statement are correct
d) None of the mentioned View
Answer

Answer: a
Explanation: Negation of P->Q is P∧~Q.
8. Consider the following statements. A: Raju should
exercise.
B: Raju is not a decent table tennis player.
C: Raju wants to play good table tennis.
The symbolic form of “Raju is not a decent table tennis player and
if he wants to play good table tennis then he should exercise.”
is?
a) A->B->C
b) B∧(C->A)
c) C->B∧A
d) B<->A∧C
View Answer

Answer: b
Explanation: For conditionals statement (if then), implications
are used.
9. The statement (~P<->Q)∧~Q is true when?
a) P: True Q: False
b) P: True Q: True
c) P: False Q: True
d) P: False Q: False View
Answer

Answer: a
Explanation: For a bi-conditional to be true both inputs should be
same.
10. Let P, Q, R be true, false, false, respectively, which of the following is true?
a) P∧(Q∧~R)
b) (P->Q)∧~R
c) Q<->(P∧R)
d) P<->(QvR)
View Answer

Answer: c
Explanation: For a bi-conditional to be true both inputs should be
the same.
This set of Discrete Mathematics Multiple Choice Questions &
Answers (MCQs) focuses on “Logics and Proofs – De-Morgan’s Laws”.

1. Which of the following statements is the negation of the statements “4 is odd or -9 is


positive”?
a) 4 is even or -9 is not negative
b) 4 is odd or -9 is not negative
c) 4 is even and -9 is negative
d) 4 is odd and -9 is not negative View Answer

Answer: c
Explanation: Using De Morgan’s Law ~(A V B) ↔ ~A ∧ ~B.
2. Which of the following represents: ~A (negation of A) if A stands for “I like badminton but
hate maths”?
a) I hate badminton and maths
b) I do not like badminton or maths
c) I dislike badminton but love maths
d) I hate badminton or like maths View
Answer

Answer: d
Explanation: De Morgan’s Law ~ (A ∧ B) ↔ ~A V ~B.
3. The compound statement A v ~(A ∧ B).
a) True
b) False View
Answer

Answer: a
Explanation: Applying De-Morgan’s law we get A v ~ A Ξ Tautology.
4. Which of the following is De-Morgan’s law?
a) P ∧ (Q v R) Ξ (P ∧ Q) v (P ∧ R)
b) ~(P ∧ R) Ξ ~P v ~R, ~(P v R) Ξ ~P ∧ ~R
c) P v ~P Ξ True, P ∧ ~P Ξ False
d) None of the mentioned View
Answer

Answer: b
Explanation: Definition of De–Morgan’s Law.
5. What is the dual of (A ∧ B) v (C ∧ D)?
a) (A V B) v (C v D)
b) (A V B) ^ (C v D)
c) (A V B) v (C ∧ D)
d) (A ∧ B) v (C v D) View
Answer

Answer: b
Explanation: In dual ∧ is replaced by v and vice – versa.
6. ~ A v ~ B is logically equivalent to?
a) ~ A → ~ B
b) ~ A ∧ ~ B
c) A → ~B
d) B V A
View Answer

Answer: c
Explanation: By identity A → B Ξ ~A V B.
7. Negation of statement (A ∧ B) → (B ∧ C) is
a) (A ∧ B) →(~B ∧ ~C)
b) ~(A ∧ B) v ( B v C)
c) ~(A →B) →(~B ∧ C)
d) None of the mentioned
View Answer

Answer: a
Explanation: ~(A →B) Ξ A ∧ ~B using this we can easily fetch the answer.
8. Which of the following satisfies commutative law?
a) ∧
b) v
c) ↔
d) All of the mentioned View
Answer

Answer: d
Explanation: All of them satisfies commutative law.
9. If the truth value of A v B is true, then truth value of ~A ∧ B can be
a) True if A is false
b) False if A is false
c) False if B is true and A is false
d) None of the mentioned View
Answer

Answer: a
Explanation: If A is false then both the condition are obeyed.
10. If P is always against the testimony of Q, then the compound statement P→(P v ~Q) is a

a) Tautology
b) Contradiction
c) Contingency
d) None of the mentioned View
Answer

Answer: a
Explanation: Since either hypothesis is false or both (hypothesis
as well as conclusion) are true.

1. A compound proposition that is always is called a


tautology.
a) True
b) False View
Answer

Answer: a
Explanation: Tautology is always true.
2. A compound proposition that is always is called a
contradiction.
a) True
b) False View
Answer

Answer: b
Explanation: Contradiction is always false.
3. If A is any statement, then which of the following is a tautology?
a) A ∧ F
b) A ∨ F
c) A ∨ ¬A
d) A ∧ T
View Answer

Answer: c
Explanation: A ∨ ¬A is always true.
4. If A is any statement, then which of the following is not a contradiction?
a) A ∧ ¬A
b) A ∨ F
c) A ∧ F
d) None of mentioned View
Answer

Answer: b
Explanation: A ∨ F is not always false.
5. A compound proposition that is neither a tautology nor a contradiction is called a

a) Contingency
b) Equivalence
c) Condition
d) Inference
View Answer

Answer: a
Explanation: Definition of contingency.
6. ¬ (A ∨ q) ∧ (A ∧ q) is a
a) Tautology
b) Contradiction
c) Contingency
d) None of the mentioned View
Answer
Answer: b
Explanation: ≡ (¬A ∧ ¬q) ∧ (A ∧ q)
≡ (¬A ∧ A) ∧ (¬q ∧ q)
≡ F ∧ F ≡ F.
7. (A ∨ ¬A) ∨ (q ∨ T) is a
a) Tautology
b) Contradiction
c) Contingency
d) None of the mentioned View
Answer

Answer: a
Explanation: ≡ (A ∨ ¬A) ∨ (q ∨ T)
≡ T ∨ T ≡ T.
8. A ∧ ¬(A ∨ (A ∧ T)) is always
a) True
b) False View
Answer

Answer: b
Explanation: ≡ A ∧ ¬ (A ∨ (A ∧ T))
≡ A ∧ ¬(A ∨ A)
≡ A ∧ ¬A ≡ F.
9. (A ∨ F) ∨ (A ∨ T) is always
a) True
b) False View
Answer

Answer: a
Explanation: ≡ (A ∨ F) ∨ (A ∨ T)
≡ A ∨ T ≡ T.
10. A → (A ∨ q) is a
a) Tautology
b) Contradiction
c) Contingency
d) None of the mentioned View
Answer

Answer: a
Explanation: ≡ A → (A ∨ q)
≡ ¬A ∨ (A ∨ q)
≡ (A ∨ ¬A) ∨ q
≡ T ∨ q ≡ T.

This set of Discrete Mathematics Multiple Choice Questions &


Answers (MCQs) focuses on “Logics – Types of Statements”.

1. The contrapositive of p → q is the proposition of


a) ¬p → ¬q
b) ¬q → ¬p
c) q → p
d) ¬q → p
View Answer

Answer: b
Explanation: Definition of contrapositive.
2. The inverse of p → q is the proposition of
a) ¬p → ¬q
b) ¬q → ¬p
c) q → p
d) ¬q → p
View Answer

Answer: a
Explanation: Definition of inverse.
3. The converse of p → q is the proposition of
a) ¬p → ¬q
b) ¬q → ¬p
c) q → p
d) ¬q → p
View Answer

Answer: c
Explanation: Definition of converse.
4. What is the contrapositive of the conditional statement? “The home team misses whenever it
is drizzling?”
a) If it is drizzling, then home team misses
b) If the home team misses, then it is drizzling
c) If it is not drizzling, then the home team does not misses
d) If the home team wins, then it is not drizzling View Answer

Answer: d
Explanation: q whenever p contrapositive is ¬q → ¬p.
5. What is the converse of the conditional statement “If it ices today, I will play ice hockey
tomorrow.”
a) “I will play ice hockey tomorrow only if it ices today.”
b) “If I do not play ice hockey tomorrow, then it will not have iced today.”
c) “If it does not ice today, then I will not play ice hockey tomorrow.”
d) “I will not play ice hockey tomorrow only if it ices today.” View Answer

Answer: a
Explanation: If p, then q has converse q → p.
6. What are the contrapositive of the conditional statement “I come to class whenever there is
going to be a test.”
a) “If I come to class, then there will be a test.”
b) “If I do not come to class, then there will not be a test.”
c) “If there is not going to be a test, then I don’t come to class.”
d) “If there is going to be a test, then I don’t come to class.” View Answer

Answer: b
Explanation: q whenever p, has contrapositive ¬q → ¬p.
7. What are the inverse of the conditional statement “ A positive integer is a composite only if it
has divisors other than 1 and itself.”
a) “A positive integer is a composite if it has divisors other than
1 and itself.”
b) “If a positive integer has no divisors other than 1 and itself, then it is not composite.”
c) “If a positive integer is not composite, then it has no divisors other than 1 and itself.”
d) None of the mentioned View
Answer

Answer: c
Explanation: p only if q has inverse ¬p → ¬q.
8. What are the converse of the conditional statement “When Raj stay up late, it is necessary
that Raj sleep until noon.”
a) “If Raj stay up late, then Raj sleep until noon.”
b) “If Raj does not stay up late, then Raj does not sleep until noon.”
c) “If Raj does not sleep until noon, then Raj does not stay up late.”
d) “If Raj sleep until noon, then Raj stay up late.” View Answer

Answer: d
Explanation: Necessary condition for p is q has converse q → p.
9. What are the contrapositive of the conditional statement “Medha will find a decent job when
she labour hard.”?
a) “If Medha labour hard, then she will find a decent job.”
b) “If Medha will not find a decent job, then she not labour hard.”
c) “If Medha will find a decent job, then she labour hard.”
d) “If Medha not labour hard, then she will not find a decent job.” View Answer

Answer: b
Explanation: The statement q when p has its contrapositive as ¬q →
¬p.
10. What are the inverse of the conditional statement “If you make your notes, it will be a
convenient in exams.”
a) “If you make notes, then it will be a convenient in exams.”
b) “If you do not make notes, then it will not be a convenient in exams.”
c) “If it will not be a convenient in exams, then you did not make your notes.”
d) “If it will be a convenient in exams, then you make your notes View Answer

Answer: b
Explanation: If p then q has inverse ¬p → ¬q.
1. The compound propositions p and q are called logically equivalent if is a tautology.
a) p ↔ q
b) p → q
c) ¬ (p ∨ q)
d) ¬p ∨ ¬q
View Answer

Answer: a
Explanation: Definition of logical equivalence.
2. p → q is logically equivalent to
a) ¬p ∨ ¬q
b) p ∨ ¬q
c) ¬p ∨ q
d) ¬p ∧ q
View Answer

Answer: c
Explanation: (p → q) ↔ (¬p ∨ q) is tautology.
3. p ∨ q is logically equivalent to
a) ¬q → ¬p
b) q → p
c) ¬p → ¬q
d) ¬p → q
View Answer

Answer: d
Explanation: (p ∨ q) ↔ (¬p → q) is tautology.
4. ¬ (p ↔ q) is logically equivalent to
a) q↔p
b) p↔¬q
c) ¬p↔¬q
d) ¬q↔¬p
View Answer

Answer: b
Explanation: ¬(p↔q)↔(p↔¬q) is tautology.
5. p ∧ q is logically equivalent to
a) ¬ (p → ¬q)
b) (p → ¬q)
c) (¬p → ¬q)
d) (¬p → q)
View Answer

Answer: a
Explanation: (p ∧ q) ↔ (¬(p → ¬q)) is tautology.
6. Which of the following statement is correct?
a) p ∨ q ≡ q ∨ p
b) ¬(p ∧ q) ≡ ¬p ∨ ¬q
c) (p ∨ q) ∨ r ≡ p ∨ (q ∨ r)
d) All of mentioned
View Answer

Answer: d
Explanation: Verify using truth table, all are correct.
7. p ↔ q is logically equivalent to
a) (p → q) → (q → p)
b) (p → q) ∨ (q → p)
c) (p → q) ∧ (q → p)
d) (p ∧ q) → (q ∧ p)
View Answer

Answer: c
Explanation: (p ↔ q) ↔ ((p → q) ∧ (q → p)) is tautology.
8. (p → q) ∧ (p → r) is logically equivalent to
a) p → (q ∧ r)
b) p → (q ∨ r)
c) p ∧ (q ∨ r)
d) p ∨ (q ∧ r) View
Answer

Answer: a
Explanation: ((p → q) ∧ (p → r)) ↔ (p → (q ∧ r)) is tautology.
9. (p → r) ∨ (q → r) is logically equivalent to
a) (p ∧ q) ∨ r
b) (p ∨ q) → r
c) (p ∧ q) → r
d) (p → q) → r
View Answer

Answer: c
Explanation: ((p → r) ∨ (q → r)) ↔ ((p ∧ q) → r) is tautology.
10. ¬ (p ↔ q) is logically equivalent to
a) p ↔ ¬q
b) ¬p ↔ q
c) ¬p ↔ ¬q
d) ¬q ↔ ¬p
View Answer

Answer: a
Explanation: (¬ (p ↔ q)) ↔ (p ↔ ¬q) is tautology.

1. Let P (x) denote the statement “x >7.” Which of these have truth value true?
a) P (0)
b) P (4)
c) P (6)
d) P (9) View
Answer

Answer: d
Explanation: Put x=9, 9>7 which is true.
2. Let Q(x) be the statement “x < 5.” What is the truth value of the quantification ∀xQ(x),
having domains as real numbers.
a) True
b) False View
Answer

Answer: b
Explanation: Q(x) is not true for every real number x, because,
for instance, Q(6) is false. That is, x = 6 is a counterexample
for the statement ∀xQ(x). This is false.
3. Determine the truth value of ∀n(n + 1 > n) if the domain consists of all real numbers.
a) True
b) False View
Answer

Answer: a
Explanation: There are no elements in the domain for which the
statement is false.
4. Let P(x) denote the statement “x = x + 7.” What is the truth value of the quantification
∃xP(x), where the domain consists of all real numbers?
a) True
b) False View
Answer

Answer: b
Explanation: Because P(x) is false for every real number x, the
existential quantification of Q(x), which is ∃xP(x), is false.
5. Let R (x) denote the statement “x > 2.” What is the truth value of the quantification ∃xR(x),
having domain as real numbers?
a) True
b) False View
Answer

Answer: a
Explanation: Because “x > 2” is sometimes true—for instance, when x
= 3–the existential quantification of R(x), which is ∃xR(x), is true.
6. The statement,” Every comedian is funny” where C(x) is “x is a comedian” and F (x) is “x is
funny” and the domain consists of all people.
a) ∃x(C(x) ∧ F (x))
b) ∀x(C(x) ∧ F (x))
c) ∃x(C(x) → F (x))
d) ∀x(C(x) → F (x))
View Answer

Answer: d
Explanation: For every person x, if comedian then x is funny.
7. The statement, “At least one of your friends is perfect”. Let P
(x) be “x is perfect” and let F (x) be “x is your friend” and let
the domain be all people.
a) ∀x (F (x) → P (x))
b) ∀x (F (x) ∧ P (x))
c) ∃x (F (x) ∧ P (x))
d) ∃x (F (x) → P (x))
View Answer

Answer: c
Explanation: For some x, x is friend and funny.
8. ”Everyone wants to learn cosmology.” This argument may be true for which domains?
a) All students in your cosmology class
b) All the cosmology learning students in the world
c) Both of the mentioned
d) None of the mentioned View
Answer

Answer: c
Explanation: Domain may be limited to your class or may be whole
world both are good as it satisfies universal quantifier.
9. Let domain of m includes all students, P (m) be the statement “m spends more than 2 hours in
playing polo”. Express ∀m ¬P (m) quantification in English.
a) A student is there who spends more than 2 hours in playing polo
b) There is a student who does not spend more than 2 hours in playing polo
c) All students spends more than 2 hours in playing polo
d) No student spends more than 2 hours in playing polo View Answer

Answer: d
Explanation: There is no student who spends more than 2 hours in
playing polo.
10. Determine the truth value of statement ∃n (4n = 3n) if the domain consists of all integers.
a) True
b) False View
Answer

Answer: a
Explanation: For n=0, 4n=3n hence, it is true.

This set of Discrete Mathematics Multiple Choice Questions &


Answers (MCQs) focuses on “Logics – Nested Quantifiers”.
1. Let Q(x, y) denote “M + A = 0.” What is the truth value of the quantifications ∃A∀M Q(M,
A).
a) True
b) False View
Answer

Answer: b
Explanation: For each A there exist only one M, because there is
no real number A such that M + A = 0 for all real numbers M.
2. Translate ∀x∃y(x < y) in English, considering domain as a real number for both the variable.
a) For all real number x there exists a real number y such that x is less than y
b) For every real number y there exists a real number x such that x is less than y
c) For some real number x there exists a real number y such that x is less than y
d) For each and every real number x and y such that x is less than y
View Answer

Answer: a
Explanation: Statement is x is less than y. Quantifier used are
for each x, there exists a y.
3. “The product of two negative real numbers is not negative.” Is given by?
a) ∃x ∀y ((x < 0) ∧ (y < 0) → (xy > 0))
b) ∃x ∃y ((x < 0) ∧ (y < 0) ∧ (xy > 0))
c) ∀x ∃y ((x < 0) ∧ (y < 0) ∧ (xy > 0))
d) ∀x ∀y ((x < 0) ∧ (y < 0) → (xy > 0)) View
Answer

Answer: d
Explanation: For every negative real number x and y, the product
of these integer is positive.
4. Let Q(x, y) be the statement “x + y = x − y.” If the domain for both variables consists of all
integers, what is the truth value of
∃xQ(x, 4).
a) True
b) False View
Answer

Answer: b
Explanation: There exist no integer for which x+4=x-4.
5. Let L(x, y) be the statement “x loves y,” where the domain for both x and y consists of all
people in the world. Use quantifiers to express, “Joy is loved by everyone.”
a) ∀x L(x, Joy)
b) ∀y L(Joy,y)
c) ∃y∀x L(x, y)
d) ∃x ¬L(Joy, x) View
Answer

Answer: a
Explanation: Joy is loved by all the people in the world.
6. Let T (x, y) mean that student x likes dish y, where the domain for x consists of all students
at your school and the domain for y consists of all dishes. Express ¬T (Amit, South Indian) by a
simple English sentence.
a) All students does not like South Indian dishes.
b) Amit does not like South Indian people.
c) Amit does not like South Indian dishes.
d) Amit does not like some dishes. View Answer

Answer: d
Explanation: Negation of the statement Amit like South Indian dishes.
7. Express, “The difference of a real number and itself is zero” using required operators.
a) ∀x(x − x! = 0)
b) ∀x(x − x = 0)
c) ∀x∀y(x − y = 0)
d) ∃x(x − x = 0)
View Answer

Answer: b
Explanation: For every real number x, difference with itself is
always zero.
8. Use quantifiers and predicates with more than one variable to express, “There is a pupil in
this lecture who has taken at least one course in Discrete Maths.”
a) ∃x∃yP (x, y), where P (x, y) is “x has taken y,” the domain for x consists of all pupil in this
class, and the domain for y consists of all Discrete Maths lectures
b) ∃x∃yP (x, y), where P (x, y) is “x has taken y,” the domain for x consists of all Discrete
Maths lectures, and the domain for y consists of all pupil in this class
c) ∀x∀yP(x, y), where P (x, y) is “x has taken y,” the domain for x consists of all pupil in this
class, and the domain for y consists of all Discrete Maths lectures
d) ∃x∀yP(x, y), where P (x, y) is “x has taken y,” the domain for x consists of all pupil in this
class, and the domain for y consists of all Discrete Maths lectures
View Answer

Answer: a
Explanation: For some x pupil, there exists a course in Discrete
Maths such that x has taken y.
9. Determine the truth value of ∃n∃m(n + m = 5 ∧ n − m = 2) if the domain for all variables
consists of all integers.
a) True
b) False View
Answer

Answer: b
Explanation: The equation does not satisfy any value of m and n in
the domain consist of integers.
10. Find a counterexample of ∀x∀y(xy > y), where the domain for all variables consists of all
integers.
a) x = -1, y = 17
b) x = -2 y = 8
c) Both x = -1, y = 17 and x = -2 y = 8
d) Does not have any counter example View
Answer

Answer: c
Explanation: Putting x=-1, y=17; -17>17 which is wrong. Putting
x=-2, y=8; -16>8 which is wrong.

1. Which rule of inference is used in each of these arguments, “If it is Wednesday, then the
Smartmart will be crowded. It is Wednesday. Thus, the Smartmart is crowded.”
a) Modus tollens
b) Modus ponens
c) Disjunctive syllogism
d) Simplification View
Answer

Answer: b
Explanation: (M ∧ (M → N)) → N is Modus ponens.
2. Which rule of inference is used in each of these arguments, “If it hailstoday, the local office
will be closed. The local office is not closed today. Thus, it did not hailed today.”
a) Modus tollens
b) Conjunction
c) Hypothetical syllogism
d) Simplification View
Answer

Answer: a
Explanation: (¬N ∧ (M → N)) → ¬M is Modus tollens.
3. Which rule of inference is used, ”Bhavika will work in an enterprise this summer. Therefore,
this summer Bhavika will work in an enterprise or he will go to beach.”
a) Simplification
b) Conjunction
c) Addition
d) Disjunctive syllogism View
Answer
Answer: c
Explanation: p → (p ∨ q) argument is ‘Addition’.
4. What rule of inference is used here?
“It is cloudy and drizzling now. Therefore, it is cloudy now.”
a) Addition
b) Simplification
c) Resolution
d) Conjunction
View Answer

Answer: b
Explanation: (p ∧ q) → p argument is Simplification.
5. What rule of inference is used in this argument?
“If I go for a balanced diet, then I will be fit. If I will be
fit, then I will remain healthy. Therefore, if I go for a
balanced diet, then I will remain healthy.”
a) Modus tollens
b) Modus ponens
c) Disjunctive syllogism
d) Hypothetical syllogism View
Answer

Answer: d
Explanation: ((p → q) ∧ (q → r)) → (p → r) argument is
‘Hypothetical syllogism’.
6. What rules of inference are used in this argument?
“All students in this science class has taken a course in
physics” and “Marry is a student in this class” imply the
conclusion “Marry has taken a course in physics.”
a) Universal instantiation
b) Universal generalization
c) Existential instantiation
d) Existential generalization View
Answer

Answer: a
Explanation: ∀xP (x), ∴ P (c) Universal instantiation.
7. What rules of inference are used in this argument?
“It is either colder than Himalaya today or the pollution is
harmful. It is hotter than Himalaya today. Therefore, the
pollution is harmful.”
a) Conjunction
b) Modus ponens
c) Disjunctive syllogism
d) Hypothetical syllogism View
Answer

Answer: c
Explanation: ((p ∨ q) ∧ ¬p) → q argument is Disjunctive syllogism.
8. The premises (p ∧ q) ∨ r and r → s implies which of the conclusion?
a) p ∨ r
b) p ∨ s
c) q ∨ s
d) q ∨ r View
Answer

Answer: b
Explanation: The premises (p ∧ q) ∨ r as two clauses, p ∨ r and q
∨ r. We can also replace r → s by the equivalent clause ¬r ∨ s.
using the two clauses p ∨ r and ¬r ∨ s, we can use resolution to
conclude p ∨ s.
9. What rules of inference are used in this argument?
“Jay is an awesome student. Jay is also a good dancer. Therefore,
Jay is an awesome student and a good dancer.”
a) Conjunction
b) Modus ponens
c) Disjunctive syllogism
d) Simplification View
Answer

Answer: a
Explanation: ((p) ∧ (q)) → (p ∧ q) argument is conjunction.
10. “Parul is out for a trip or it is not snowing” and “It is snowing or Raju is playing chess”
imply that
a) Parul is out for trip
b) Raju is playing chess
c) Parul is out for a trip and Raju is playing chess
d) Parul is out for a trip or Raju is playing chess View Answer

Answer: d
Explanation: Let p be “It is snowing,” q be “Parul is out for a
trip,” and r the proposition “Raju is playing chess.” The
hypotheses as ¬p
∨ q and p ∨ r, respectively. Using resolution, the proposition q ∨
r is, “Parul is out for a trip or Raju is playing chess.”

1. Let the statement be “If n is not an odd integer then square of n is not odd.”, then if P(n) is “n
is an not an odd integer” and Q(n) is “(square of n) is not odd.” For direct proof we should prove
a) ∀nP ((n) → Q(n))
b) ∃ nP ((n) → Q(n))
c) ∀n~(P ((n)) → Q(n))
d) ∀nP ((n) → ~(Q(n)))
View Answer

Answer: a
Explanation: Definition of direct proof.
2. Which of the following can only be used in disproving the statements?
a) Direct proof
b) Contrapositive proofs
c) Counter Example
d) Mathematical Induction View
Answer

Answer: c
Explanation: Counter examples cannot be used to prove results.
3. Let the statement be “If n is not an odd integer then sum of n with some not odd number will
not be odd.”, then if P(n) is “n is an not an odd integer” and Q(n) is “sum of n with some not odd
number will not be odd.” A proof by contraposition will be
a) ∀nP ((n) → Q(n))
b) ∃ nP ((n) → Q(n))
c) ∀n~(P ((n)) → Q(n))
d) ∀n(~Q ((n)) → ~(P(n)))
View Answer

Answer: d
Explanation: Definition of proof by contraposition.
4. When to proof P→Q true, we proof P false, that type of proof is known as
a) Direct proof
b) Contrapositive proofs
c) Vacuous proof
d) Mathematical Induction View
Answer

Answer: c
Explanation: Definition of vacuous proof.
5. In proving √5 as irrational, we begin with assumption √5 is rational in which type of proof?
a) Direct proof
b) Proof by Contradiction
c) Vacuous proof
d) Mathematical Induction View
Answer

Answer: b
Explanation: Definition of proof by contradiction.
6. A proof covering all the possible cases, such type of proofs are known as
a) Direct proof
b) Proof by Contradiction
c) Vacuous proof
d) Exhaustive proof View
Answer

Answer: d
Explanation: Definition of exhaustive proof.
7. Which of the arguments is not valid in proving sum of two odd number is not odd.
a) 3 + 3 = 6, hence true for all
b) 2n +1 + 2m +1 = 2(n+m+1) hence true for all
c) All of the mentioned
d) None of the mentioned View
Answer

Answer: a
Explanation: Some examples are not valid in proving results.
8. A proof broken into distinct cases, where these cases cover all prospects, such proofs are
known as
a) Direct proof
b) Contrapositive proofs
c) Vacuous proof
d) Proof by cases View
Answer

Answer: c
Explanation: Definition of proof by cases.
9. A proof that p → q is true based on the fact that q is true, such proofs are known as
a) Direct proof
b) Contrapositive proofs
c) Trivial proof
d) Proof by cases View
Answer

Answer: c
Explanation: Definition of trivial proof.
10. A theorem used to prove other theorems is known as
a) Lemma
b) Corollary
c) Conjecture
d) None of the mentioned View
Answer

Answer: a
Explanation: Definition of lemma.

a) P^
(R v
R)
b) P^
(P
^
R)
c) R^
(P
v
Q)
d) Q ^
(P
v
R)

1. {x: x is an integer neither positive nor negative} is


a) Empty set
b) Non-empty set
c) Finite set
d) Non- empty and Finite set View
Answer

Answer: d
Explanation: Set = {0} non-empty and finite set.
2. {x: x is a real number between 1 and 2} is an
a) Infinite set
b) Finite set
c) Empty set
d) None of the mentioned View
Answer

Answer: a
Explanation: It is an infinite set as there are infinitely many
real number between any two different real numbers.
3. Write set {1, 5, 15, 25,…} in set-builder form.
a) {x: either x=1 or x=5n, where n is a real number}
b) {x: either x=1 or x=5n, where n is a integer}
c) {x: either x=1 or x=5n, where n is an odd natural number}
d) {x: x=5n, where n is a natural number} View Answer

Answer: c
Explanation: Set should include 1 or an odd multiple of 5.
4. Express {x: x= n/ (n+1), n is a natural number less than 7} in roster form.
a) {1⁄2, 2⁄3, 4⁄5, 6⁄7}
b) {1⁄2, 2⁄3, 3⁄4, 4⁄5, 5⁄6, 6⁄7, 7⁄8}
c) {1⁄2, 2⁄3, 3⁄4, 4⁄5, 5⁄6, 6⁄7}
d) Infinite set
View Answer

Answer: c
Explanation: n/(n+1) = 1/(1+1) = 1⁄2 and n>7.
5. Number of power set of {a, b}, where a and b are distinct elements.
a) 3
b) 4
c) 2
d) 5
View Answer

Answer: b
Explanation: Power set of {a, b} = {∅ , {a, b}, {a}, {b}}.
6. Which of the following is subset of set {1, 2, 3, 4}? a) {1, 2}
b) {1, 2, 3}
c) {1}
d) All of the mentioned
View Answer

Answer: d
Explanation: There are total 16 subsets.
7. A = {∅ ,{∅ },2,{2,∅ },3}, which of the following is true?
a) {{∅ ,{∅ }} ∈ A
b) {2} ∈ A
c) ∅ ⊂ A
d) 3 ⊂ A
View Answer

Answer: c
Explanation: Empty set is a subset of every set.
8. Subset of the set A= { } is?
a) A
b) {}
c) ∅
d) All of the mentioned

View Answer

Answer: d
Explanation: Every set is subset of itself and Empty set is subset
of each set.
9. {x: x ∈ N and x is prime} then it is
a) Infinite set
b) Finite set
c) Empty set
d) Not a set
View Answer

Answer: a
Explanation: There is no extreme prime, number of primes is infinite.
10. Convert set {x: x is a positive prime number which divides 72} in roster form.
a) {2, 3, 5}
b) {2, 3, 6}
c) {2, 3}
d) {∅ }
View Answer

Answer: c
Explanation: 2 and 3 are the divisors of 72 which are prime.

MCQs
Test & Rank
Internship
Training
Discrete Mathematics Questions and Answers – Sets
« PrevNext »
This set of Discrete Mathematics Multiple Choice Questions &
Answers (MCQs) focuses on “Sets”.

1. A is an ordered collection of objects.


a) Relation
b) Function
c) Set
d) Proposition
View Answer

Answer: c
Explanation: By the definition of set.
2. The set O of odd positive integers less than 10 can be expressed by
a) {1, 2, 3}
b) {1, 3, 5, 7, 9}
c) {1, 2, 5, 9}
d) {1, 5, 7, 9, 11}

View Answer

Answer: b
Explanation: Odd numbers less than 10 is {1, 3, 5, 7, 9}.
3. Power set of empty set has exactly subset.
a) One
b) Two
c) Zero
d) Three View
Answer

Answer: a
Explanation: Power set of null set has exactly one subset which is
empty set.
4. What is the Cartesian product of A = {1, 2} and B = {a, b}? a) {(1, a), (1, b), (2, a), (b,
b)}
b) {(1, 1), (2, 2), (a, a), (b, b)}
c) {(1, a), (2, a), (1, b), (2, b)}
d) {(1, 1), (a, a), (2, a), (1, b)}
View Answer

Answer: c
Explanation: A subset R of the Cartesian product A x B is a
relation from the set A to the set B.
5. The Cartesian Product B x A is equal to the Cartesian product A x B.
a) True
b) False View
Answer

Answer: b
Explanation: Let A = {1, 2} and B = {a, b}. The Cartesian product
A x B = {(1, a), (1, b), (2, a), (2, b)} and the Cartesian
product B
x A = {(a, 1), (a, 2), (b, 1), (b, 2)}. This is not equal to A x B.
6. What is the cardinality of the set of odd positive integers less than 10?
a) 10
b) 5
c) 3
d) 20
View Answer

Answer: b
Explanation: Set S of odd positive an odd integer less than 10 is
{1, 3, 5, 7, 9}. Then, Cardinality of set S = |S| which is 5.
7. Which of the following two sets are equal? a) A = {1, 2} and
B = {1}
b) A = {1, 2} and B = {1, 2, 3}
c) A = {1, 2, 3} and B = {2, 1, 3}
d) A = {1, 2, 4} and B = {1, 2, 3}
View Answer

Answer: c
Explanation: Two set are equal if and only if they have the same
elements.
8. The set of positive integers is
a) Infinite
b) Finite
c) Subset
d) Empty
View Answer

Answer: a
Explanation: The set of positive integers is not finite.
9. What is the Cardinality of the Power set of the set {0, 1, 2}?
a) 8
b) 6
c) 7
d) 9
View Answer

Answer: a
Explanation: Power set P ({0, 1, 2}) is the set of all subsets of
{0, 1, 2}. Hence, P({0, 1, 2}) = {null, {0}, {1}, {2}, {0, 1},
{0,2}, {1,
2}, {0, 1, 2}}.
10. The members of the set S = {x | x is the square of an integer and x < 100} is
a) {0, 2, 4, 5, 9, 58, 49, 56, 99, 12}
b) {0, 1, 4, 9, 16, 25, 36, 49, 64, 81}
c) {1, 4, 9, 16, 25, 36, 64, 81, 85, 99}
d) {0, 1, 4, 9, 16, 25, 36, 49, 64, 121}
View Answer

Answer: b
Explanation: The set S consists of the square of an integer less
than 10

This set of Discrete Mathematics Multiple Choice Questions &


Answers (MCQs) focuses on “Set Operations – 1”.

1. The union of the sets {1, 2, 5} and {1, 2, 6} is the set a) {1, 2, 6, 1}
b) {1, 2, 5, 6}
c) {1, 2, 1, 2}
d) {1, 5, 6, 3}
View Answer

Answer: b
Explanation: The union of the sets A and B, is the set that
contains those elements that are either in A or in B.
2. The intersection of the sets {1, 2, 5} and {1, 2, 6} is the set

a) {1, 2}
b) {5, 6}
c) {2, 5}
d) {1, 6}
View Answer

Answer: a
Explanation: The intersection of the sets A and B, is the set
containing those elements that are in both A and B.
3. Two sets are called disjoint if there is the empty set.
a) Union
b) Difference
c) Intersection
d) Complement
View Answer

Answer: c
Explanation: By the definition of the disjoint set.
4. Which of the following two sets are disjoint? a) {1, 3, 5} and {1, 3,
6}
b) {1, 2, 3} and {1, 2, 3}
c) {1, 3, 5} and {2, 3, 4}
d) {1, 3, 5} and {2, 4, 6}
View Answer

Answer: d
Explanation: Two sets are disjoint if the intersection of two sets
is the empty set.
5. The difference of {1, 2, 3} and {1, 2, 5} is the set
a) {1}
b) {5}
c) {3}
d) {2}
View Answer

Answer: c
Explanation: The difference of the sets A and B denoted by A-B, is
the set containing those elements that are in A not in B.
6. The complement of the set A is
a) A – B
b) U – A
c) A – U
d) B – A View
Answer

Answer: b
Explanation: The complement of the set A is the complement of A
with respect to U.
7. The bit string for the set {2, 4, 6, 8, 10} (with universal set of natural numbers less than or
equal to 10) is
a) 0101010101
b) 1010101010
c) 1010010101
d) 0010010101
View Answer

Answer: a
Explanation: The bit string for the set has a one bit in second,
fourth, sixth, eighth, tenth positions, and a zero elsewhere.
8. Let Ai = {i, i+1, i+2, …..}. Then set {n, n+1, n+2, n+3, …..}
is the of the set Ai.
a) Union
b) Intersection
c) Set Difference
d) Disjoint
View Answer

Answer: b
Explanation: By the definition of the generalized intersection of
the set.
9. The bit strings for the sets are 1111100000 and 1010101010. The union of these sets is
a) 1010100000
b) 1010101101
c) 1111111100
d) 1111101010
View Answer

Answer: d
Explanation: The bit string for the union is the bitwise OR of the
bit strings.
10. The set difference of the set A with null set is
a) A
b) null
c) U
d) B
View Answer

Answer: a
Explanation: The set difference of the set A by the null set
denoted by A – {null} is A.

This set of Discrete Mathematics Interview Questions and Answers


focuses on “Set Operations – 2”.

1. Let the set A is {1, 2, 3} and B is {2, 3, 4}. Then the number of elements in A U B is?
a) 4
b) 5
c) 6
d) 7
View Answer

Answer: a
Explanation: AUB is {1, 2, 3, 4}.
2. Let the set A is {1, 2, 3} and B is { 2, 3, 4}. Then number of elements in A ∩ B is?
a) 1
b) 2
c) 3
d) 4
View Answer

Answer: b
Explanation: A ∩ B is {2, 3}.
3. Let the set A is {1, 2, 3} and B is {2, 3, 4}. Then the set A – B is?
a) {1, -4}
b) {1, 2, 3}
c) {1}
d) {2, 3}
View Answer

Answer: c
Explanation: In A – B the common elements get cancelled.
4. In which of the following sets A – B is equal to B – A? a) A = {1, 2, 3}, B = {2,
3, 4}
b) A = {1, 2, 3}, B = {1, 2, 3, 4}
c) A = {1, 2, 3}, B = {2, 3, 1}
d) A = {1, 2, 3, 4, 5, 6}, B = {2, 3, 4, 5, 1}
View Answer

Answer: c
Explanation: A- B= B-A = Empty set.
5. Let A be set of all prime numbers, B be the set of all even prime numbers, C be the set of all
odd prime numbers, then which of the following is true?
a) A ≡ B U C
b) B is a singleton set.
c) A ≡ C U {2}
d) All of the mentioned View
Answer

Answer: d
Explanation: 2 is the only even prime number.
6. If A has 4 elements B has 8 elements then the minimum and maximum number of elements
in A U B are
a) 4, 8
b) 8, 12
c) 4, 12
d) None of the mentioned
View Answer

Answer: b
Explanation: Minimum would be when 4 elements are same as in 8,
maximum would be when all are distinct.
7. If A is {{Φ}, {Φ, {Φ}}}, then the power set of A has how many element?
a) 2
b) 4
c) 6
d) 8
View Answer

Answer: b
Explanation: The set A has got 2 elements so n(P(A))=4.
8. Two sets A and B contains a and b elements respectively. If power set of A contains 16 more
elements than that of B, value of ‘b’ and ‘a’ are
a) 4, 5
b) 6, 7
c) 2, 3
d) None of the mentioned
View Answer

Answer: a
Explanation: 32-16=16, hence a=5, b=4.
9. Let A be {1, 2, 3, 4}, U be set of all natural numbers, then U-A’(complement of A) is
given by set.
a) {1, 2, 3, 4, 5, 6, ….}
b) {5, 6, 7, 8, 9, ……}
c) {1, 2, 3, 4}
d) All of the mentioned
View Answer

Answer: c
Explanation: U – A’ ≡ A.
10. Which sets are not empty?
a) {x: x is a even prime greater than 3}
b) {x : x is a multiple of 2 and is odd}
c) {x: x is an even number and x+3 is even}
d) { x: x is a prime number less than 5 and is odd} View Answer

Answer: d
Explanation: Because the set is {3}.

1. The shaded area of figure is best described by? discrete-mathematics-


questions-answers-venn-diagram-q1
a) A ∩ B
b) A U B
c) A
d) B
View Answer

Answer: a
Explanation: The region is A intersection B.
2. The shaded area of figure is best described by? discrete-mathematics-
questions-answers-venn-diagram-q2
a) A‘ (Complement of A)
b) A U B -B
c) A ∩ B
d) B
View Answer

Answer: b
Explanation: The region is complement of B.
3. If n(A)=20 and n(B)=30 and n(A U B) = 40 then n(A ∩ B) is?
a) 20
b) 30
c) 40
d) 10
View Answer

Answer: d
Explanation: n(A U B) = n(A) + n(B) – n(A ∩ B).
4. The shaded area of figure is best described by? discrete-mathematics-
questions-answers-venn-diagram-q4
a) A‘ (Complement of A)
b) B – (A ∩ B) – (C ∩ B)
c) A ∩ C ∩ B
d) B’ (Complement of B) View
Answer

Answer: b
Explanation: The region is difference B with A and C.
5. The relation between sets A, B, C as shown by venn diagram is

a) A is subset of B and B is subset of C


b) C is not a subset of A and A is subset of B
c) C is subset of B and B is subset of A
d) None of the mentioned View
Answer

Answer: c
Explanation: As set C is totally inside set B, set B is totally
inside set A.
6. Let A: All badminton player are good sportsperson. B: All person who
plays cricket are good sportsperson.
Let X denotes set of all badminton players, Y of all cricket
players, Z of all good sportsperson. Then which of the following
statements is correct?
a) Z contains both X and Y
b) Z contains X and Y is outside
c) X contains Y and Z
d) None of the mentioned View
Answer

Answer: a
Explanation: X and Y are subset of Z.
7. If n(A)=10, n(B)=30,n(C)=50 and if set A, B, C are pairwise disjoint then which of the
following is correct?
a) n(A U B)=0
b) n(B U C)=0
c) n(A U B U C)=90
d) All of the mentioned View
Answer

Answer: d
Explanation: All the statements are true based on definition.
8. In the given figure the if n(A)=20,n(U)=50,n(C)=10 and n(A∩B)=5 then n(B)=?
discrete-mathematics-questions-answers-venn-diagram-q8
a) 35
b) 20
c) 30
d) 10
View Answer

Answer: a
Explanation: Here n(B)= n(U) – n(A) + n(A∩B).
9. Let the students who likes table tennis be 12, the ones who like lawn tennis 10, those who
like only table tennis are 6, then number of students who likes only lawn tennis are, assuming
there are total of 16 students.
a) 16
b) 8
c) 4
d) 10
View Answer

Answer: c
Explanation: The students who only plays lawn tennis will be a
total lawn tennis player – those who play both the sports.
10. The shaded area of figure is best described by? discrete-mathematics-
questions-answers-venn-diagram-q10
a) A‘ (Complement of A)
b) A U B – (A ∩ B)
c) A – B
d) B
View Answer

Answer: b
Explanation: The region is complement of( A intersection B).

1. Let C and D be two sets then which of the following statements are true?
i) C U D = D U C
ii) C∩D=D∩C
a) Both of the statements
b) Only i statement
c) Only ii statement
d) None of the statements View
Answer

Answer: a
Explanation: Commutative laws hold good in sets.
2. If set C is {1, 2, 3, 4} and C – D = Φ then set D can be
a) {1, 2, 4, 5}
b) {1, 2, 3}
c) {1, 2, 3, 4, 5}
d) None of the mentioned
View Answer

Answer: c
Explanation: C ∩ D should be equivalent to C for C – D = Φ.
3. Let C and D be two sets then C – D is equivalent to
a) C’ ∩ D
b) C‘∩ D’
c) C ∩ D’
d) None of the mentioned View
Answer

Answer: c
Explanation: Set C-D will be having those elements which are in C
but not in D.
4. For two sets C and D the set (C – D) ∩ D will be
a) C
b) D
c) Φ
d) None of the mentioned View
Answer

Answer: c
Explanation: C-D ≡ C ∩ D’, D ∩ D’ ≡ Φ.
5. Which of the following statement regarding sets is false?
a) A ∩ A = A
b) A U A = A
c) A – (B ∩ C) = (A – B) U (A –C)
d) (A U B)’ = A’ U B’
View Answer

Answer: d
Explanation: (A U B)’ = A’ ∩ B’.
6. Let C = {1,2,3,4} and D = {1, 2, 3, 4} then which of the following hold not true in this case?
a) C – D = D – C
b) C U D = C ∩ D
c) C ∩ D = C – D
d) C – D = Φ
View Answer

Answer: c
Explanation: C ∩ D = {1, 2, 3, 4} ≠ Φ.
7. If C’ U (D ∩ E’) is equivalent to
a) (C ∩ (D U E))’
b) (C ∩( D∩ E’))’
c) (C ∩( D’ U E))’
d) (C U ( D ∩ E’)’
View Answer

Answer: c
Explanation: (C’)’≡ C, (C∩ D)’ ≡ C’ U D’.
8. Let Universal set U is {1, 2, 3, 4, 5, 6, 7, 8}, (Complement of
A) A’ is {2, 5, 6, 7}, A ∩ B is {1, 3, 4} then the set B’ will
surely have of which of the element?
a) 8
b) 7
c) 1
d) 3
View Answer

Answer: a
Explanation: The set A is {1,3,4,8} and thus surely B does not have
8 in it. Since 8 does not belong to A ∩ B. For other element like
7 we can’t be sure.
9. Let a set be A then A ∩ φ and A U φ are
a) φ, φ
b) φ, A
c) A, φ
d) None of the mentioned View
Answer

Answer: b
Explanation: By Domination Laws on sets.
10. If in sets A, B, C, the set B ∩ C consists of 8 elements, set A
∩ B consists of 7 elements and set C ∩ A consists of 7 elements
then the minimum element in set A U B U C will be?
a) 8
b) 14
c) 22
d) 15
View Answer

Answer: a
Explanation: For minimum elements set B and C have 8 elements
each and all of the elements are same, Also set A should have 7
elements which are already present in B and C. Thus A U B U C ≡ A
≡ B.
1. Let set A = {1, 2} and C be {3, 4} then A X B (Cartesian product of set A and B) is?
a) {1, 2, 3, 4}
b) {(1, 3),(2, 4)}
c) {(1, 3), (2, 4), (1, 4), (2, 3)}
d) {(3, 1), (4, 1)}
View Answer

Answer: c
Explanation: In set A X B : {(c , d) |c ∈ A and d ∈ B}.
2. If set A has 4 elements and B has 3 elements then set n(A X B) is?
a) 12
b) 14
c) 24
d) 7
View Answer

Answer: a
Explanation: The total elements in n(A X B) = n(A) * n(B).
3. If set A has 3 elements then number of elements in A X A X A are

a) 9
b) 27
c) 6
d) 19
View Answer

Answer: b
Explanation: n(A X A X A) = n(A)* n(A)* n(A).
4. Which of the following statements regarding sets is false?
a) A X B = B X A
b) A X B ≠ B X A
c) n(A X B) = n(A) * n(B)
d) All of the mentioned View
Answer

Answer: a
Explanation: The Cartesian product of sets is not commutative.
5. If n(A X B) = n(B X A) = 36 then which of the following may hold true?
a) n(A)=2, n(B)=18
b) n(A)=9, n(B)=4
c) n(A)=6, n(b)=6
d) None of the mentioned
View Answer

Answer: c
Explanation: n(A) should be equal to n(B) for n(A X B) = n(B x A).
6. If C = {1} then C X (C X C) = (C X C) X C the given statement is true or false.
a) True
b) False View
Answer

Answer: b
Explanation: The Cartesian product is not associative, (C × C) × C
= { ((1, 1), 1) } ≠ { (1,(1, 1)) } = C × (C × C).
7. Let the sets be A, B, C, D then (A ∩ B) X (C ∩ D) is equivalent to
a) (A X C) ∩ (B X D)
b) (A X D) U (B X C)
c) (A X C) U ( B X D)
d) None of the mentioned View
Answer

Answer: a
Explanation: (A ∩ B) X (C ∩ D) = (A X C) ∩ (B X D) but in case of
unions this is not true.
8. If A ⊆ B then A X C ⊆ B X C the given statement is true or false.
a) True
b) False View
Answer

Answer: a
Explanation: Let an arbitrary element x ∈ A and y ∈ C, then x ∈ B
(subset property), (x,y) ∈ AX C also (x,y) ∈ B X C. This implies
A X C ⊆ B X C.
9. If set A and B have 3 and 4 elements respectively then the number of subsets of set (A X B)
is?
a) 1024
b) 2048
c) 512
d) 4096
View Answer

Answer: d
Explanation: The A X B has 12 elements, then the number of the
subset are 2 12 = 4096.
10. If set A X B=B X A then which of the following sets may satisfy? a) A={1, 2, 3}, B={1, 2,
3, 4}
b) A={1, 2}, B={2, 1}
c) A={1, 2, 3}, B={2, 3, 4}
d) None of the mentioned
View Answer

Answer: b
Explanation: For set A X B = B X A, this is possible only when
set A = B.

1. If a set contains 3 elements then the number of subsets is?


a) 6
b) 3
c) 12
d) 8
View Answer

Answer: d
Explanation: For elements with n elements the number of subsets
are 2n.
2. The set containing all the collection of subsets is known as

a) Subset
b) Power set
c) Union set
d) None of the mentioned View
Answer

Answer: b
Explanation: Power set contains all the subsets as its elements.
3. If a set is empty then number of subsets will be
a) 1
b) 2
c) 0
d) 4
View Answer

Answer: a
Explanation: The set has zero elements so 2o = 1.
4. If the number of subsets of a set are 4 then the number of elements in that sets are
a) 1
b) 2
c) 3
d) 4
View Answer

Answer: b
Explanation: The number of elements be x then x2 = 4 thus x=2.
5. The number of subsets of a set is 5.
a) True
b) False View
Answer

Answer: b
Explanation: The number of subsets will always be a power of 2.
6. The number of subsets of a set can be odd or even.
a) True
b) False View
Answer

Answer: a
Explanation: The number of subsets will be odd in case of empty set
otherwise even.
7. Let a set be A={1, 2, 3} then the number of subsets containing two elements will be
a) 4
b) 3
c) 5
d) 8
View Answer

Answer: b
Explanation: The subsets will be {1, 2}, {2, 3}, {1, 3}.
8. Let the set be A= {a, b, c, {a,b}} then which of the following is false?
a) {a, b} Є A
b) a Є A
c) {a} Є A
d) b, c ЄA
View Answer

Answer: c
Explanation: Only elements belongs to a set, {a} is a subset of
this set.
9. If A={1, 2, 3, 4}, then the number of the subsets of A that contain the element 2 but not 3, is?
a) 16
b) 4
c) 8
d) 24
View Answer

Answer: b
Explanation: The subsets would be {1, 2, 4},{1, 2}, {2, 3}, {2}.
10. Let A(1), A(2), A(3),…….., A(100) be 100 sets such that number of elements in A(i)=i+1
and A(1) is subset of A(2), A(2)is subset of A(3),….., A(99) is subset of A(100). The number of
elements in union of the all the sets are: n(A(1) U A(2) U A(3) …..U A(100)).
a) 99
b) 100
c) 101
d) 102
View Answer

Answer: c
Explanation: Since all sets are subsets of A(100) therefore in
union only elements of A(100)will come.A(100) contains 101
elements.

1. A function is said to be if and only if f(a) = f(b)


implies that a = b for all a and b in the domain of f.
a) One-to-many
b) One-to-one
c) Many-to-many
d) Many-to-one
View Answer

Answer: b
Explanation: A function is one-to-one if and only if f(a)≠f(b)
whenever a≠b.
2. The function f(x)=x+1 from the set of integers to itself is onto. Is it True or False?
a) True
b) False View
Answer

Answer: a
Explanation: For every integer “y” there is an integer “x ” such
that f(x) = y.
3. The value of ⌊ 1/2.⌊ 5/2⌋ ⌋ is
a) 1
b) 2
c) 3
d) 0.5
View Answer

Answer: a
Explanation: The value of ⌊ 5/2⌋ is 2 so, the value of ⌊ 1/2.2⌋ is
1.
4. Which of the following function f: Z X Z → Z is not onto?
a) f(a, b) = a + b
b) f(a, b) = a
c) f(a, b) = |b|
d) f(a, b) = a – b View
Answer

Answer: c
Explanation: The function is not onto as f(a)≠b.
5. The domain of the function that assign to each pair of integers the maximum of these two
integers is
a) N
b) Z
c) Z +
d) Z+ X Z+
View Answer

Answer: d
Explanation: The domain of the integers is Z+ X Z+.
6. Let f and g be the function from the set of integers to itself, defined by f(x) = 2x + 1 and g(x)
= 3x + 4. Then the composition of f and g is
a) 6x + 9
b) 6x + 7
c) 6x + 6
d) 6x + 8

View Answer
Answer: a
Explanation: The composition of f and g is given by f(g(x)) which
is equal to 2(3x + 4) + 1.
7. bytes are required to encode 2000 bits of data.
a) 1
b) 2
c) 3
d) 8
View Answer

Answer: b
Explanation: Two bytes are required to encode 2000 (actually with
2 bytes you can encode up to and including 65,535.
8. The inverse of function f(x) = x3 + 2 is
a) f -1 (y) = (y – 2) 1/2
b) f -1 (y) = (y – 2) 1/3
c) f -1 (y) = (y) 1/3
d) f -1 (y) = (y – 2)
View Answer

Answer: b
Explanation: To find the inverse of the function equate f(x) then
find the value of x in terms of y such that f -1 (y) = x.
9. The function f(x) = x3 is bijection from R to R. Is it True or False?
a) True
b) False View
Answer

Answer: a
Explanation: The function f(x) = x3 is one to one as no two
values in domain are assigned the same value of the function and
it is onto as all R of the co domain is images of elements in the
domain.
10. The g -1({0}) for the function g(x)= ⌊ x⌋ is
a) {x | 0 ≤ x < 1}
b) {x | 0 < x ≤ 1}
c) {x | 0 < x < 1}
d) {x | 0 ≤ x ≤ 1}
View Answer

Answer: d
Explanation: g({0}) for the function g(x) is {x | 0 ≤ x ≤ 1}. Put g(x)
= y and find the value of x in terms of y such that ⌊ x⌋ = y.

1. If f(x) = (x3 – 1) / (3x + 1) then f(x) is?


a) O(x2)
b) O(x)
c) O(x2 / 3)
d) O(1)
Answer: a
Explanation: 0 < (x3 – 1) / (3x + 1) < x2.
2. If f(x) = 3x2 + x3logx, then f(x) is?
a) O(x2)
b) O(x3)
c) O(x)
d) O(1) View
Answer

Answer: b
Explanation: 0 < 3x2 < x3, it follows that 0 < 3x2 + x3logx < x3.
Consequently, f(x) = O(x3).
3. The big-O notation for f(n) = (nlogn + n2)(n3 + 2) is?
a) O(n2)
b) O(3n)
c) O(n4)
d) O(n5) View
Answer

Answer: d
Explanation: 0 < n3 + 2 < n3, it follows that (nlogn + n2)(n3 + 2)
is less than equal to n5.
4. The big-theta notation for function f(n) = 2n3 + n – 1 is?
a) n
b) n2
c) n3
d) n4
View Answer

Answer: c
Explanation: 2n3 + n – 1 is less than equal to n3.
5. The big-theta notation for f(n) = nlog(n2 + 1) + n2logn is?
a) n2logn
b) n2
c) logn
d) nlog(n2)
View Answer

Answer: a
Explanation: n2logn < n3, it follows that nlog(n2 + 1) + n2logn is
less than n3 and greater than n2logn.
5. The big-omega notation for f(x, y) = x5y3 + x4y4 + x3y5 is?
a) x5y3
b) x5y5
c) x3y3
d) x4y4 View
Answer

Answer: c
Explanation: x5y3, x4y4 and x3y5 is greater than or equal to x3y3.
6. If f1(x) is O(g(x)) and f2(x) is o(g(x)), then f1(x) + f2(x) is?
a) O(g(x))
b) o(g(x))
c) O(g(x)) + o(g(x))
d) None of the mentioned
View Answer

Answer: a
Explanation: f2(x) is less than O(g(x)). So, f1(x) + f2(x) upper
bound is O(g(x)).
7. The little-o notation for f(x) = xlogx is?
a) x
b) x3
c) x2
d) xlogx View
Answer

Answer: c
Explanation: Find the limit for xlogx / x2 as x tends to infinity.
8. The big-O notation for f(n) = 2log(n!) + (n2 + 1)logn is?
a) n
b) n2
c) nlogn
d) n2logn
View Answer

Answer: d
Explanation: log(n!) < n2logn, it follows that 2log(n!) + (n2 +
1)logn is less than or equal n2logn.
9. The big-O notation for f(x) = 5logx is?
a) 1
b) x
c) x2
d) x3
View Answer

Answer: b
Explanation: logx < x, it follows that 5logx < x.
10. The big-Omega notation for f(x) = 2x4 + x2 – 4 is?
a) x2
b) x3
c) x
d) x4
View Answer

Answer: d
Explanation: 2x4 + x2 – 4 is greater than or equal to x4.

1. What is the domain of a function?


a) the maximal set of numbers for which a function is defined
b) the maximal set of numbers which a function can take values
c) it is a set of natural numbers for which a function is defined
d) none of the mentioned View
Answer

Answer: a
Explanation: Domain is the set of all the numbers on which a
function is defined. It may be real as well.
2. What is domain of function f(x)= x1/2? a) (2, ∞)
b) (-∞, 1)
c) [0, ∞)
d) None of the mentioned
View Answer

Answer: c
Explanation: A square root function is not defined for negative
real numbers.
3. What is the range of a function?
a) the maximal set of numbers for which a function is defined
b) the maximal set of numbers which a function can take values
c) it is set of natural numbers for which a function is defined
d) none of the mentioned View
Answer

Answer: b
Explanation: Range is the set of all values which a function may take.
4. What is domain of function f(x) = x-1 for it to be defined everywhere on domain?
a) (2, ∞)
b) (-∞, ∞) – {0}
c) [0, ∞)
d) None of the mentioned
View Answer

Answer: b
Explanation: Function x-1 is not defined for x=0, otherwise it
defined for every real number.
5. The range of function f(x) = sin(x) is (-∞, ∞).
a) True
b) False View
Answer

Answer: b
Explanation: A sine function takes values between -1 and 1,thus
range is [-1, 1].
6. Codomain is the subset of range.
a) True
b) False View
Answer

Answer: b
Explanation: Range is the subset of codomain, that is every value
in the range is in codomain but vice-versa it is not true.
7. What is range of function f(x) = x-1 which is defined everywhere on its domain?
a) (-∞, ∞)
b) (-∞, ∞) – {0}
c) [0, ∞)
d) None of the mentioned
View Answer

Answer: a
Explanation: Function x-1 may take any real number hence it’s
range is all real numbers.
8. If f(x) = 2x then range of the function is? a) (-∞, ∞)
b) (-∞, ∞) – {0}
c) (0, ∞)
d) None of the mentioned
View Answer

Answer: c
Explanation: The function cannot take negative values,hence range
is (0, ∞).
9. If f(x) = x2 + 4 then range of f(x) is given by? a) [4, ∞)
b) (-∞, ∞) – {0}
c) (0, ∞)
d) None of the mentioned
View Answer

Answer: a
Explanation: Since minimum value of x2 is 0, thus x2 +4 may take
any value between [4,∞).
10. Let f(x)=sin2(x) + log(x) then domain of f(x) is (-∞, ∞).
a) True
b) False View
Answer

Answer: b
Explanation: Domain is (0, ∞), since log(x) is not defined for
negative numbers and zero.

This set of Discrete Mathematics Multiple Choice Questions &


Answers (MCQs) focuses on “Number of Functions”.

1. An injection is a function which is?


a) many-one
b) one-one
c) onto
d) none of the mentioned

View Answer
Answer: b
Explanation: One-One functions are also known as injection.
2. A mapping f : X → Y is one one if
a) f(x1) ≠ f(x2) for all x1, x2 in X
b) If f(x1) = f(x2) then x1 = x2 for all x1, x2 in X
c) f(x1) = f(x2) for all x1, x2 in X
d) None of the mentioned View
Answer

Answer: b
Explanation: In one one function every element in A should have
unique image in B,thus if two image are equal this means there
preimages are same.
3. A function is defined by mapping f : A → B such that A contains m elements and B contains
n elements and m ≤ n then number of one one functions are
a) nCm x m!
b) nCm x n!
c) 0
d) none of the mentioned View
Answer

Answer: a
Explanation: From n elements in B we need to select m elements
and then arrange them in all ways, thus answer=nCm x m!.
4. A function is defined by mapping f : A -> B such that A contains m elements and B contains
n elements and m>n then number of one one functions are
a) nCm x m!
b) nCm x n!
c) 0
d) none of the mentioned View
Answer

Answer: c
Explanation: Since for function to be defined every element
should have a image, since m > n atleast one element need to have
same image, thus we can’t form any unique images and hence the
number of one one function are zero.
5. For an onto function range is equivalent to codomain.
a) True
b) False View
Answer

Answer: a
Explanation: Since in onto function every image should have
preimage thus all the elements in codomain should have preimages.
6. Onto function are known as injection.
a) True
b) False
Answer: b
Explanation: Onto functions are known as surjection.
7. Set A has 3 elements and set B has 4 elements then number of injections defined from A to B
are?
a) 12
b) 24
c) 36
d) 48
View Answer

Answer: b
Explanation:Injections will be 4C3 x 3!=24.
8. A function is defined by mapping f : A → B such that A contains m elements and B contains
n elements and 1≤n≤m then number of onto functions are
a) r=1∑r=n nCr (-1)n-r rm
b) r=1∑r=n nCr (-1)n-r rn
c) r=1∑r=n nCr (-1)m-r rn
d) None of the mentioned View
Answer

Answer: a
Explanation: The number of onto function is equal tpo the
coffecient of xm in m!(ex – 1)n.
9. A function is defined by mapping f:A→B such that A contains m elements and B contains n
elements and m > n then number of bijections are

a) nCm x m!
b) nCm x n!
c) 0
d) none of the mentioned View
Answer

Answer: c
Explanation: Since we can’t define any one one function in such
case so number of bujections will be zero.
10. A bijection is a function which is many-one and onto.
a) True
b) False View
Answer

Answer: b
Explanation: A bijection is a function which is one-one(injection)
and onto(surjection).

1. A floor function map a real number to


a) smallest previous integer
b) greatest previous integer
c) smallest following integer
d) none of the mentioned
View Answer

Answer: b
Explanation: Floor function f(x) is the largest integer not
greater than x.
2. A ceil function map a real number to
a) smallest previous integer
b) greatest previous integer
c) smallest following integer
d) none of the mentioned View
Answer

Answer: c
Explanation: Ceil function f(x) is the smallest integer not less
than x.
3. A function f(x) is defined as f(x) = x – [x], where [.] represents GIF then
a) f(x) will be intergral part of x
b) f(x) will be fractional part of x
c) f(x) will always be 0
d) none of the mentioned View
Answer

Answer: b
Explanation: A integral part of a number is subtracted from that
number we are left with the fractional part of that number.
4. Floor(2.4) + Ceil(2.9) is equal to
a) 4
b) 6
c) 5
d) none of the mentioned View
Answer

Answer: c
Explanation: Floor(2.4) = 2, Ceil(2.9) = 3, 2 + 3 = 5.
5. For some integer n such that x < n < x + 1, ceil(x) < n.
a) True
b) False View
Answer

Answer: b
Explanation: If x < n < x + 1 then ceil(x) = n.
6. For some number x, Floor(x) <= x <= Ceil(x).
a) True
b) False View
Answer

Answer: a
Explanation: Floor function f(x) is the largest integer not
greater than x and ceil function f(x) is the smallest integer not
less than x.
7. If x, and y are positive numbers both are less than one, then maximum value of floor(x + y)
is?
a) 0
b) 1
c) 2
d) -1
View Answer

Answer: b
Explanation: Since x < 1 and y < 1 this implies x + y < 2 which
means maximium value of floor(x + y) is 1.
8. If x, and y are positive numbers both are less than one, then maximum value of ceil(x + y) is?
a) 0
b) 1
c) 2
d) -1
View Answer

Answer: c
Explanation: Since x < 1 and y < 1 this implies x + y < 2 which
means maximum value of ceil(x + y) is 2.
9. If X = Floor(X) = Ceil(X) then
a) X is a fractional number
b) X is a Integer
c) X is less than 1
d) none of the mentioned View
Answer

Answer: b
Explanation: Only in case of integers X = Floor(X) = Ceil(X) holds
good.
10. Let n be some integer greater than 1,then floor((n-1)/n) is 1.
a) True
b) False View
Answer

Answer: b
Explanation: Since (n-1)/n will always be less than one thus f
floor((n-1)/n) is 0

1. For an inverse to exist it is necessary that a function should be

a) injection
b) bijection
c) surjection
d) none of the mentioned View
Answer

Answer: b
Explanation: Inverse exist only for those functions which are one
one and onto.
2. If f(x) = y then f-1(y) is equal to
a) y
b) x
c) x2
d) none of the mentioned View
Answer

Answer: b
Explanation: On giving inverse, image the function returns
preimage thus f-1 (y) = x.
3. A function f(x) is defined from A to B then f -1 is defined
a) from A to B
b) from B to A
c) depends on the inverse of function
d) none of the mentioned View
Answer

Answer: b
Explanation: Inverse associate each element in B with
corresponding element in A.
4. If f is a function defined from R to R, is given by f(x) = 3x –
5 then f –1(x) is given by
a) 1/(3x-5)
b) (x+5)/3
c) does not exist since it is not a bijection
d) none of the mentioned View
Answer

Answer: b
Explanation: y = 3x-5, x = (y+5)/3, f -1(x) = (x+5)/3.
5. For some bijective function inverse of that function is not bijective.
a) True
b) False View
Answer

Answer: b
Explanation: If f(x) is a bijection than f -1(x) is also a bijection.
6. f(x) is a bijection than f -1(x) is a mirror image of f(x) around y = x.
a) True
b) False View
Answer

Answer: a
Explanation: Inverse of a function is the mirror image of function
in line y = x.
7. If f is a function defined from R to R, is given by f(x) = x2 then f -1(x) is given by?
a) 1/(3x-5)
b) (x+5)/3
c) does not exist since it is not a bijection
d) none of the mentioned View
Answer

Answer: c
Explanation: It is not a one one function hence Inverse does not exist.
8. For any function fof -1(x) is equal to?
a) x
b) 1
c) x2
d) none of the mentioned View
Answer

Answer: a
Explanation:Compostion of a function with its inverse gives x.
9. The solution to f(x) = f -1(x) are
a) no solutions in any case
b) same as solution to f(x) = x
c) infinite number of solution for every case
d) none of the mentioned View
Answer

Answer: b
Explanation: Inverse of a function is the mirror image of function
in line y = x.
10. Let f(x) = x then number of solution to f(x) = f -1(x) is zero.
a) True
b) False View
Answer

Answer: b
Explanation: Since inverse of a function is the mirror image of
function in line y = x, therefore in this case infinte solution
will exist.

1. How many even 4 digit whole numbers are there? a) 1358


b) 7250
c) 4500
d) 3600
View Answer

Answer: c
Explanation: The thousands digit cannot be zero, so there are 9
choices. There are 10 possibilities for the hundreds digit and 10
possibilities for the tens digit. The units digit can be 0, 2, 4, 6
or 8, so there are 5 choices. By the basic counting principle, the
number of even five digit whole numbers is 9 × 10 × 10 × 5 = 45,00.
2. In a multiple-choice question paper of 15 questions, the answers can be A, B, C or D. The
number of different ways of answering the question paper are
a) 65536 x 47
b) 194536 x 45
c) 23650 x 49
d) 11287435
View Answer

Answer: a
Explanation: There are 415 = 65536 x 47 different ways of
answering the exam paper of 15 MCQs.
3. How many words with seven letters are there that start with a vowel and end with an A? Note
that they don’t have to be real words and letters can be repeated.
a) 45087902
b) 64387659
c) 12765800
d) 59406880
View Answer

Answer: d
Explanation: The first letter must be a vowel, so there are 5
choices. The second letter can be any one of 26, the third letter
can be any one of 26, the fourth letter can be any one of 26 and
fifth and sixth letters can be any of 26 choices. The last letter
must be an A, so there is only 1 choice. By the basic counting
principle, the number of ‘words’ is 5 × 26 × 26 × 26 × 26 × 26 ×
1 = 59406880.
4. Neela has twelve different skirts, ten different tops, eight different pairs of shoes, three
different necklaces and five different bracelets. In how many ways can Neela dress up?
a) 50057
b) 14400
c) 34870
d) 56732
View Answer

Answer: b
Explanation: By the basic counting principle, the number of
different ways = 12 × 10 × 8 × 3 × 5 = 14400. Note that shoes
come in pairs. So she must choose one pair of shoes from ten
pairs, not one shoe from twenty.
5. How many five-digit numbers can be made from the digits 1 to 7 if repetition is allowed?
a) 16807
b) 54629
c) 23467
d) 32354
View Answer

Answer: a
Explanation: 75 = 16807 ways of making the numbers consisting of
five digits if repetition is allowed.
6. For her English literature course, Ruchika has to choose one novel to study from a list of ten,
one poem from a list of fifteen and one short story from a list of seven. How many different
choices does Rachel have?
a) 34900
b) 26500
c) 12000
d) 10500
View Answer

Answer: d
Explanation: By the Basic Counting Principle, the number of
different choices is 10 × 15 × 7 = 10500.
7. There are two different Geography books, five different Natural Sciences books, three
different History books and four different Mathematics books on a shelf. In how many different
ways can they be arranged if all the books of the same subjects stand together?
a) 353450
b) 638364
c) 829440
d) 768700
View Answer

Answer: c
Explanation: There are four groups of books which can be arranged
in 4! different ways. Among those books, two are Geography books,
five are Natural Sciences books, three are History books and four
are Mathematics books. Therefore, there are 4! × 2! × 5! × 3! × 4!
= 829440 ways to arrange the books.
8. The code for a safe is of the form PPPQQQQ where P is any number from 0 to 9 and Q
represents the letters of the alphabet. How many codes are possible for each of the following
cases? Note that the digits and letters of the alphabet can be repeated.
a) 874261140
b) 537856330
c) 549872700
d) 456976000
View Answer

Answer: d
Explanation: 103 × 264 = 456976000 possible codes are formed for
the safe with the alphanumeric digits.
9. Amit must choose a seven-digit PIN number and each digit can be chosen from 0 to 9. How
many different possible PIN numbers can Amit choose?
a) 10000000
b) 9900000
c) 67285000
d) 39654900
View Answer
Answer: a
Explanation: By the basic counting principle, the total number of
PIN numbers Amit can choose is 10 × 10 × 10 × 10 × 10 × 10 × 10 =
10,000000.
10. A head boy, two deputy head boys, a head girl and 3 deputy head girls must be chosen out
of a student council consisting of 14 girls and 16 boys. In how many ways can they are chosen?
a) 98072
b) 27384
c) 36428
d) 44389
View Answer

Answer: b
Explanation: There are 16 × 15 × 14 + 14 × 13 × 12 × 11 = 27384
ways to choose from a student council.

1. How many substrings (of all lengths inclusive) can be formed from a character string of
length 8? (Assume all characters to be distinct)
a) 14
b) 21
c) 54
d) 37
View Answer

Answer: d
Explanation: Let’s consider the given string is CLEAN, so set of
string of length 1 = {C,L,E,A,N} ; cardinality of set = 5 set of
string of length 2 = {CL,EE,EA,NN}, set of string of length 3 =
{CLE,LEE,EAN}, set of strings of length 4 = {CLEN,LEAN}, set of
strings of length 5 = {CLEAN} and set of string of length 0 = {}
and we cannot have any substring of length 6 as given string has
only 5 length. So total no of substrings are possible = 0 length
substring + 1 length substring
+ 2 length substrings +3 length substrings + 4 length substrings +
5 length substrings = 1 + 5 + 4 + 3 + 2 + 1 = 16 means for 1
length string to n length substrings it will sum of the n natural
no from
1 to n.
so 1+2+3+…+n = n(n+1)2 so total no substrings possible = 0 length
strings + n(n+1)2 = 1+ [n(n+1)2] so total no of substrings
possible in n length string (All length inclusive )= 1 +
[n(n+1)2]=8(8+1)2 = 37.
2. The number of diagonals can be drawn in a hexagon is
a) 9
b) 32
c) 16
d) 21
View Answer

Answer: a
Explanation: A hexagon has 6 sides. We obtain the diagonals by
joining the vertices in pairs.
Total number of sides and diagonals = 6C2 = 6∗ 52∗ 1 = 5×3 = 15. This
includes its 6 sides also. So, Diagonals = 15 – 6 = 9. Hence, the
number of diagonals is 9.
3. The number of binary strings of 17 zeros and 8 ones in which no two ones are adjacent is

a) 43758
b) 24310
c) 32654
d) 29803
View Answer

Answer: a
Explanation: First place 17 zeroes side by side _ 0 _ 0 _ 0 … 0 _
and 8 1’s can be placed in any of the (17+1) available gaps hence
the number of ways = n+1Ck = 43758.
4. How many words that can be formed with the letters of the word ‘SWIMMING’ such that
the vowels do not come together? Assume that words are of with or without meaning.
a) 430
b) 623
c) 729
d) 1239
View Answer

Answer: c
Explanation: The word ‘SWIMMING contains 8 letters. Of which, I
occurs twice and M occurs twice. Therefore, the number of words
formed by this word = 8!2!∗ 2! = 10080. In order to find the
number of permutations that can be formed where the two vowels I
and I come together, we group
the letters that should come together and consider that group as
one letter. So, the letters are S, W, M, M, N, G, (I, I). So, the
number of letters are 7 the number of ways in which 7 letters can
be arranged is 7! = 5040. In I and I, the number of ways in which
I and I can be arranged is 2!. Hence, the total number of ways in
which the letters
of the ‘SWIMMING’ can be arranged such that vowels are always
together are 7!2!∗ 2! = 5040 ways. The number of words in which
the vowels do not come together is = (10080 – 5040) = 5040.
5. A number lock contains 6 digits. How many different zip codes can be made with the digits
0–9 if repetition of the digits is allowed upto 3 digits from the beginning and the first digit is not
0?
a) 254307
b) 453600
c) 458760
d) 972340
View Answer

Answer: b
Explanation: For the first position, there are 9 possible choices
(since 0 is not allowed). After that number is chosen, there are
10 possible choices (since 0 is now allowed) for the second
digit, for
the third digit there are 10 possible choices, 9 possible choices
for the fourth digit and 8 possible choices for the fifth digit
and 7 possible choices for the sixth digit. The count of number
locks = 453600.
6. Let M be a sequence of 9 distinct integers sorted in ascending order. How many distinct pairs
of sequences, N and O are there such that i) each are sorted in ascending order, ii) N has 5 and O
has 4 elements, and iii) the result of merging N and O gives that sequence?
a) 84
b) 35
c) 194
d) 138
View Answer

Answer: a
Explanation: Selecting any 3 elements from given 9 elements gives 9C3
= 84 number of distinct pairs of sequences.
7. 14 different letters of alphabet are given, words with 6 letters are formed from these given
letters. How many number of words are there which have at least one letter repeated?
a) 892742
b) 999988
c) 213216
d) 786730
View Answer

Answer: b
Explanation: Number of words which have at least one letter replaced
= total number of words – total number of words in which no letter
is repeated, => 106 – 12P6 => 1000000 − 924 = 999988.
8. In how many ways can 10 boys be seated in a row having 28 seats such that no two friends
occupy adjacent seats?
a) 13P5
b) 9P29
c) 19P10
d) 15P7
View Answer

Answer: c
Explanation: First let us take the 18 unoccupied seats. They create
19 slots i.e., one on the left of each seat and one on the right
of the last one. So we can place the 10 boys in any of these 19
slots that are, 19P10 ways.
9. In how many ways can the letters of the word SANFOUNDRY be rearranged such that the
vowels always appear together?
a) (8+3)!2!
b) 6!2!
c) 8!*3!
d) 4!8!
View Answer

Answer: c
Explanation: Take AOU together and treat it like 1 entity and
arrange SNFNDRY in 8! Ways. Then, the AOU can be arranged in 3!
ways. So, total arrangements = 8! * 3! = 40320 * 6 = 241920.
10. How many ways can 8 prizes be given away to 7 students, if each student is eligible for all
the prizes?
a) 40325
b) 40320
c) 40520
d) 40720
View Answer

Answer: b
Explanation: Now the first student is eligible to receive any of
the 8 available prizes (so 8 ways), the second student will
receive a prize from rest 7 available prizes (so 7 ways), the
third student will receive his prize from the rest 6 prizes
available(so 6 ways) and so on. So total ways would be 8! =
8*7*6*5*4*3*2*1 = 40320. Hence, the 7 prizes can be distributed
in 40320 ways

1. In a playground, 3 sisters and 8 other girls are playing together. In a particular game, how
many ways can all the girls be seated in a circular order so that the three sisters are not seated
together? a) 457993
b) 3386880
c) 6544873
d) 56549
View Answer

Answer: b
Explanation: There are 3 sisters and 8 other girls in total of 11
girls. The number of ways to arrange these 11 girls in a circular
manner = (11– 1)! = 10!. These three sisters can now rearrange
themselves in 3! ways. By the multiplication theorem, the number
of ways so that
3 sisters always come together in the arrangement = 8! × 3!.
Hence, the required number of ways in which the arrangement can
take place if none of the 3 sisters is seated together: 10! – (8!
× 3!) = 3628800 – (40320 * 6) = 3628800 – 241920 = 3386880.
2. How many numbers of three digits can be formed with digits 1, 3, 5, 7 and 9?
a) 983
b) 120
c) 345
d) 5430
View Answer

Answer: b
Explanation: Here number of digits, n = 5 and number of places to
be filled-up r = 3. Hence, the required number is 5P3 = 5!/2!*3!
= 120.
3. The size of a multiset is 6 which is equal to the number of elements in it with counting
repetitions (a multiset is an unordered collection
of elements where the elements may repeat any number of times).
Determine the number of multisets can be grouped from n distinct
elements so that at least one element occurs exactly twice?
a) 326
b) 28
c) 45
d) 62
View Answer

Answer: c
Explanation: There are six places to be filled in the multiset
using the n distinct elements. At least one element has to occur
exactly twice and that would leave 4 more places in the multiset
means that at most four elements can occur exactly once. Thus
there are two mutually exclusive cases as follows: 1) Exactly one
element occurs exactly twice and select this element in n ways.
Fill up the remaining four spots using 5 distinct elements from
the remaining n−1 elements in n-1C4 ways. 2) Exactly four elements
that occur at least once each.
Hence, the total number of ways to form the multiset is
nC2 + n * n-1C4 = 6C2 + 6 * 6-1C4 = 45.
4. How many words can be formed with the letters of the word ‘CASTLE’ when ‘O’ and ‘A’
occupying end places.
a) 217
b) 48
c) 75
d) 186
View Answer

Answer: b
Explanation: When ‘O’ and ‘A’ are occupying end-places =>
A.S.T.L. (CE). We can see that (CE) are fixed, hence A, S, T, L
can be arranged in 4! Ways and (C, E) can be arranged themselves
is 2! ways. So, the number of words formed = 4! x 2! = 48 ways.
5. Determine the number of ways of choosing a cricket team (consists of 11 players) out of 18
players if a particular player is never chosen. a) 12798
b) 22800
c) 31824
d) 43290
View Answer

Answer: c
Explanation: If a particular player is never chosen that would
mean 11 players are selected out of 18 players. Hence, required
number of ways = 18C11 = 31824.
6. How many different choices can be made from 5 roses, 4 marigold and 8 sunflowers if at
least one flower is to be chosen for making of garland?
a) 269
b) 270
c) 281
d) 320
View Answer

Answer: a
Explanation: Number of ways of selecting roses = (5+1) = 6 ways,
number of ways of selecting marigold = (4+1) = 5 ways, and the
number of ways of selecting sunflowers = (8+1) = 9 ways. Total
number of ways of selecting flowers= 6 * 5 * 9 = 270. But this
includes when no flowers or zero flowers is selected (There is no
flowers of a different type, hence n=0 => 2n = 20 = 1). Hence,
the number of ways of selecting at least one fruit = 270 – 1 =
269.
7. In how many ways 6 pens can be selected from 15 identical black pens?
a) 9*3!
b) 21
c) 14!
d) 1
View Answer

Answer: d
Explanation: Here the pens are identical, the total number of ways
of selecting 6 pens is 1.
8. Determine the number of ways of selecting one or more letters from the letters BBBBBB?
a) 6
b) 73
c) 23
d) 56
View Answer

Answer: a
Explanation: The number of ways of selecting one ‘B’s = 1,
selecting two ‘B’s = 1, selecting three ‘B’s = 1, selecting four
‘B’s = 1, selecting five ‘B’s = 1 and selecting six ‘B’s = 1.
Hence, the required number of ways = 6.
9. Determine the number of ways such that 5 men and 5 women be seated at a round table if no
two women are seated together.
a) 654870
b) 144521
c) 362160
d) 5634
View Answer

Answer: c
Explanation: The men and women can be seated alternately so that
no two women will sit together. Hence, 4 women can be seated on
alternate seats at a round table in (4 – 1)! or 6
ways. Now, the 5 men can be seated in the remaining seats in 5! or
120 ways. Therefore the total number of ways in this case will be
(10-1)! – (120 * 6) = 362160.
10. Find the number of ways in which 4 people E, F, G, H, A, C can be seated at a round table,
such that E and F must always sit together.
a) 32
b) 290
c) 124
d) 48
View Answer

Answer: d
Explanation: E and F can sit together in all arrangements in 2!
Ways. Now, the arrangement of the 5 people in a circle can be
done in (5 – 1)! or 24 ways. Therefore, the total number of ways
will be 24 x 2 = 48.

1. Calculate sum of divisors of n = 1900. a) 6530


b) 5346
c) 3387
d) 4123
View Answer

Answer: d
Explanation: The prime factorization of 1800 is 19 * 22 * 52 and
S(22) = 1 + 2 + 4 = 7
S(52) = 1 + 5 + 25 = 31
Therefore, S(1800) = 19 * 7 * 31 = 4123.
2. Given the factorization of a number n, then the sum of divisors can be computed in
a) linear time
b) polynomial time
c) O(logn)
d) o(n+1)
View Answer

Answer: b
Explanation: The exact number of running time depends on the
computational model. When analyzing arithmetic with large numbers,
we usually count either bit operations or arithmetic operations of
size O(logn) (where n is the input size). Now, given the
factorization of a number n, then the sum of divisors can be
computed in polynomial time.
3. Calculate the sum of divisors of N = 9600. a) 23250
b) 47780
c) 54298
d) 31620
View Answer

Answer: d
Explanation: The prime factorization of 1800 is 3 * 27 * 52 and
S(3) = 1 + 3 = 4
S(22) = 1 + 2 + 4 + 8 + 16 + 32 + 64 + 128 = 255
S(52) = 1 + 5 + 25 = 31
Therefore, S(1800) = 4 * 255 * 31 = 31620.
4. Find the number of odd positive integers of the number 456.
a) 54
b) 27
c) 16
d) 8
View Answer

Answer: d
Explanation: To find the number of odd factors (which includes
1), we can exclude any power of 2 and do the same. So, for 456,
we have (3 + 1)(1 + 1) = 8 odd positive factors.
5. The number of even positive integers of 3200 is
a) 24
b) 32
c) 164
d) 209
View Answer

Answer: a
Explanation: To find the number of even factors, we can multiply
the number of even factors by the power of 2. For 3200, we have
(5 + 1)(1
+ 1)(2) = 24 even factors.
6. What is the sum of divisors of the number 1872? a) 12493
b) 5438
c) 45862
d) 654
View Answer

Answer: a
Explanation: The prime factorization of 1872 is 13 * 32 * 24 and S(24)
= 1 + 2 + 4 + 8 + 16 = 31, S(52) = 1 + 5 + 25 = 31. Therefore, S(1872)
= 31 * 31 * 13 = 12493.
7. Find the odd positive integer of the number 6500.
a) 43
b) 17
c) 12
d) 87
View Answer

Answer: c
Explanation: To find the number of odd factors, we can exclude any
power of 2 and do the same. So, for 6500, we have (5 + 1)(1 + 1) =
6 * 2 = 12 odd positive factors.
8. How many even positive integers are there in the number 7362?
a) 16
b) 58
c) 35
d) 165
View Answer
Answer: a
Explanation: To find the number of even factors, we can multiply
the number of even factors by the power of 2. For 5065, we have
(3 + 1)(1
+ 1)(2) = 4 * 2 * 2 = 16 even factors.
9. Calculate sum of divisors of n = 8620. a) 7549
b) 54201
c) 18102
d) 654
View Answer

Answer: c
Explanation: The prime factorization of 1800 is 431 * 22 * 5 and
S(22) = 1 + 2 + 4 = 7
S(52) = 1 + 5 = 6
Therefore, S(1800) = 6 * 7 * 431 = 18102.
10. Find the odd positive integer of the number 4380. a) 108
b) 48
c) 75
d) 8
View Answer

Answer: b
Explanation: To find the number of odd factors, we can exclude any
power of 2 and do the same. So, for 6500, we have (5 + 1)(3 + 1)(1
+ 1) = 6 * 4 * 2 = 48 odd positive factors.

1. For a gaming competition, 8 girls are planning on splitting up into 3 (non-empty) groups.
How many ways can they split up into these groups? a) 465
b) 1056
c) 966
d) 3215
View Answer

Answer: c
Explanation: Using the inclusion-exclusion principle, the total
number of ways of splitting the girls into 3 groups is 38 + 3.(28)
+ 3.(18). However, since the three groups are identical we need to
divide by 3!. Hence, the answer is 966.
2. In a picnic with 20 persons where 6 chocolates will be given to the top 8 children(the
chocolates are distinct: first, second). How many ways can this be done?
a) 18C6
b) 20P6
c) 25C4 * 6!
d) 19P5
View Answer

Answer: b
Explanation: This is a permutation problem since the chocolates
are distinct. The answer is P(20, 6) -> the number of ways to
arrange 20 things taken 6 at a time -> which is 20!(20−6)! =
20*19*18*17*16*15.
3. How many ways can one choose 20 cookies from 45 different types (assuming there are at
least 20 of each type)?
a) 64C21 * 15
b) 64C20
c) 44C20 * 2!
d) 65C22
View Answer

Answer: b
Explanation: Imagine the 20 cookies one is choosing are
indistinguishable dots. The 45 different types of cookies are like
45 distinguishable boxes and so the answer is C(45 + 20-1, 20) =
64C20.
4. Assume that it is an afternoon. What is the time on the 24 hour clock after 146 hours?
a) 12:10 pm
b) 8:30 am
c) 3 am
d) 2 pm View
Answer

Answer: d
Explanation: Divide 146 with 24. The remainder is the time on the
24 hour clock. So, 146 = 6*24 + 2 and the result is 2pm.
5. There are 28 identical oranges that are to be distributed among 8 distinct girls. How many
ways are there to distribute the oranges?
a) 22P7
b) 34C6
c) 35C7
d) 28C8 View
Answer

Answer: c
Explanation: By the definition of star and bar problem, there are
n+r-1Cr-1 possible distributions of n identical objects among r
distinct bins. Now, there are n = 28 identical objects and r = 8
distinct bins. Using the formula above, there are 35C7 ways to
distribute the oranges.
6. There are 5 distinct fruits. How many ways can they be planted into identical fruit plants?
a) 87
b) 52
c) 76
d) 128
View Answer
Answer: b
Explanation: These fruits can be placed into 1, 2, 3, 4 or 5
fruit plants. The number of distributions of fruits into fruit
plants will thus be the sum of Stirling numbers of the second
kind: S(5,1) + S(5,2)
+ S(5,3) + S(5,4) + S(5,5) = 1 + 15 + 25 + 10 + 1 = 52.
7. A woman has 14 identical pens to distribute among a group of 10 distinct students. How
many ways are there to distribute the 14 pens such that each student gets at least one pencil?
a) 15C10
b) 10C5 * 11
c) 15C8 * 4!
d) 13C9 View
Answer

Answer: d
Explanation: For this type of problem, n>=r must be true and so
according to stars and bars model, the number of possible
arrangements of stars and bars is n-1Cr-1 or equivalently, there
are n-1Cr-1 distributions of n identical objects into r distinct
non-empty bins. In this example, there are n = 14 identical
objects to be distributed among r=10 distinct bins. Using the
above formula, the number of possible distributions is 13C9.
8. Suppose that M is the product of k distinct primes. Find the number of ways to write N as the
product of positive integers(>1), where the order of terms does not matter.
a) MCN-k
b) NCM
c) N * Bk
d) Bk
View Answer

Answer: d
Explanation: To solve the problem first find the prime
factorization of each term of the product, and place the factors
of each term into a box. Then, since N is the product of distinct
prime factors, each prime factor appears in a unique box. Since
the product of all of these terms is N, each prime factor must be
in a box. Conversely, for any arrangement of these n distinct
primes into r identical boxes, multiply the primes in a box to
create a term and the product of these terms results in N. This
establishes the bijection and the number of ways is Bk which is
Bell number.
9. How many ways are there to place 7 differently colored toys into
5 identical urns if the urns can be empty? Note that all balls
have to be used.
a) 320
b) 438
c) 1287
d) 855
View Answer

Answer: d
Explanation: The problem can be described as distinct objects
into any number of identical bins and this number can be found
with B7 =
∑S(7,k), where S(7,k) is the number of distributions of 5
distinct objects into k identical non-empty bins, so that S(7,1)
= 1, S(7,2)
= 63, S(7,3) = 301, S(7,4) = 350 and S(7,5) = 140. These values
can be found using the recurrence relation identity for Stirling
numbers of the second kind. Thus, B7 = 1 + 63 + 301 + 350 + 140 =
855.
10. Suppose, there are 7 of your friends who want to eat pizza (8 distinct people in total). You
order a 16-cut pizza (16 identical slices). How many distributions of pizza slices are there if each
person gets at least one slice of pizza?
a) 346
b) 6435
c) 3214
d) 765
View Answer

Answer: b
Explanation: This problem can be viewed as identical objects
distributed into distinct non-empty bins. Using the formula for
these kind of distributions n-1Cr-1 = 15C7 = 6435. Thus, there are
distributions of the pizza slices.

This set of Discrete Mathematics Multiple Choice Questions & Answers (MCQs) focuses on
“Advanced Counting Techniques – Recurrence Relation”.

1. Consider the recurrence relation a1=4, an=5n+an-1. The value of a64 is


a) 10399
b) 23760
c) 75100
d) 53700
View Answer

Answer: a
Explanation: an=5n+an-1
= 5n + 5(n-1) + … + an-2
= 5n + 5(n-1) + 5(n − 2) +…+ a1
= 5n + 5(n-1) + 5(n − 2) +…+ 4 [since, a1=4]
= 5n + 5(n-1) + 5(n − 2) +…+ 5.1 – 1
= 5(n + (n − 1)+…+2 + 1) – 1
= 5 * n(n+1)/ 2 – 1
an = 5 * n(n+1)/ 2 – 1
Now, n=64 so the answer is a64 = 10399.
2. Determine the solution of the recurrence relation Fn=20Fn-1 − 25Fn-2 where F0=4
and F1=14.
a) an = 14*5n-1
b) an = 7/2*2n−1/2*6n
c) an = 7/2*2n−3/4*6n+1
d) an = 3*2n−1/2*3n
View Answer
Answer: b
Explanation: The characteristic equation of the recurrence relation is → x2−20x+36=0
So, (x-2)(x-18)=0. Hence, there are two real roots x1=2 and x2=18. Therefore the solution to the
recurrence relation will have the form: an=a2n+b18n. To find a and b, set n=0 and n=1 to get a system
of two equations with two unknowns: 4=a20+b180=a+b and 3=a21+b61=2a+6b. Solving this system
gives b=-1/2 and a=7/2. So the solution to the recurrence relation is,
an = 7/2*2n−1/2*6n.
3. What is the recurrence relation for 1, 7, 31, 127, 499?
a) bn+1=5bn-1+3
b) bn=4bn+7!
c) bn=4bn-1+3
d) bn=bn-1+1
View Answer

Answer: c
Explanation: Look at the differences between terms: 1, 7, 31, 124,…. and these are growing by a
factor of 4. So, 1⋅ 4=4, 7⋅ 4=28, 31⋅ 4=124, and so on. Note that we always end up with 3 less than
the next term. So, bn=4bn-1+3 is the recurrence relation and the initial condition is b0=1.
4. If Sn=4Sn-1+12n, where S0=6 and S1=7, find the solution for the recurrence
relation. a) an=7(2n)−29/6n6n
b) an=6(6n)+6/7n6n
c) an=6(3n+1)−5n
d) an=nn−2/6n6n
View Answer

Answer: b
Explanation: The characteristic equation of the recurrence relation is → x2−4x-12=0
So, (x-6)(x+2)=0. Only the characteristic root is 6. Therefore the solution to the recurrence relation will
have the form: an=a.6n+b.n.6n. To find a and b, set n=0 and n=1 to get a system of two equations with
two unknowns: 6=a60+b.0.60=a and 7=a61+b.1.61=2a+6b. Solving this system gives a=6 and b=6/7.
So the solution to the recurrence relation is, an=6(6n)−6/7n6n.
5. Find the value of a4 for the recurrence relation an=2an-1+3, with
a0=6. a) 320
b) 221
c) 141
d) 65
View Answer

Answer: c
Explanation: When n=1, a1=2a0+3, Now a2=2a1+3. By substitution, we get a2=2(2a0+3)+3.
Regrouping the terms, we get a4=141, where a0=6.
6. The solution to the recurrence relation an=an-1+2n, with initial term a0=2 are
a) 4n+7
b) 2(1+n)
c) 3n2
d) 5*(n+1)/2
View Answer

Answer: b
Explanation: When n=1, a1=a0+2. By substitution we get, a2=a1+2 ⇒ a2=(a0+2)+2 and so on. So
the solution to the recurrence relation, subject to the initial condition should be an=2+2n=2(1+n).
7. Determine the solution for the recurrence relation bn=8bn-1−12bn-2 with b0=3 and
b1=4. a) 7/2*2n−1/2*6n
b) 2/3*7n-5*4n
c) 4!*6n
d) 2/8n
View Answer

Answer: a
Explanation: Rewrite the recurrence relation bn-8bn-1+12bn-2=0. Now from the characteristic
equation: x2−8x+12=0 we have x: (x−2)(x−6)=0, so x=2 and x=6 are the characteristic roots.
Therefore the solution to the recurrence relation will have the form: bn=b2n+c6n. To find b and c, set
n=0 and n=1 to get a system of two equations with two unknowns: 3=b20+c60=b+c, and
4=b21+c61=2b+6c. Solving this system gives c=-1/2 and b=7/2. So the solution to the recurrence
relation is, bn=7/2*2n−1/2*6n.
8. What is the solution to the recurrence relation an=5an-1+6an-2?
a) 2n2
b) 6n
c) (3/2)n
d) n!*3
View Answer

Answer: b
Explanation: Check for the left side of the equation with all the options into the recurrence relation.
Then, we get that 6n is the required solution to the recurrence relation an=5an-1 + 6an-2.
9. Determine the value of a2 for the recurrence relation an = 17an-1 + 30n with a0=3.
a) 4387
b) 5484
c) 238
d) 1437
View Answer

Answer: d
Explanation: When n=1, a1=17a0+30, Now a2=17a1+30*2. By substitution, we get
a2=17(17a0+30)+60. Then regrouping the terms, we get a2=1437, where a0=3.
10. Determine the solution for the recurrence relation an = 6an-1−8an-2 provided
initial conditions a0=3 and a1=5.
a) an = 4 * 2n – 3n
b) an = 3 * 7n – 5*3n
c) an = 5 * 7n
d) an = 3! * 5n
View Answer

Answer: b
Explanation: The characteristic polynomial is x2−6x+8. By solving the characteristic equation,
x2−6x+8=0 we get x=2 and x=4, these are the characteristic roots. Therefore we know that the solution
to the recurrence relation has the form an=a*2n+b*4n, for some constants a and
b. Now, by using the initial conditions a0 and a1 we have: a=7/2 and b=-1/2. Therefore the solution to
the recurrence relation is: an = 4 * 2n – 1*3n = 7/2 * 2n – 1/2*3n

1. Consider the recurrence relation a1=4, an=5n+an-1. The value of a64 is


a) 10399
b) 23760
c) 75100
d) 53700
View Answer

Answer: a
Explanation: an=5n+an-1
= 5n + 5(n-1) + … + an-2
= 5n + 5(n-1) + 5(n − 2) +…+ a1
= 5n + 5(n-1) + 5(n − 2) +…+ 4 [since, a1=4]
= 5n + 5(n-1) + 5(n − 2) +…+ 5.1 – 1
= 5(n + (n − 1)+…+2 + 1) – 1
= 5 * n(n+1)/ 2 – 1
an = 5 * n(n+1)/ 2 – 1
Now, n=64 so the answer is a64 = 10399.
2. Determine the solution of the recurrence relation Fn=20Fn-1 − 25Fn-2 where F0=4
and F1=14.
a) an = 14*5n-1
b) an = 7/2*2n−1/2*6n
c) an = 7/2*2n−3/4*6n+1
d) an = 3*2n−1/2*3n
View Answer

Answer: b
Explanation: The characteristic equation of the recurrence relation is → x2−20x+36=0
So, (x-2)(x-18)=0. Hence, there are two real roots x1=2 and x2=18. Therefore the solution to the
recurrence relation will have the form: an=a2n+b18n. To find a and b, set n=0 and n=1 to get a system
of two equations with two unknowns: 4=a20+b180=a+b and 3=a21+b61=2a+6b. Solving this system
gives b=-1/2 and a=7/2. So the solution to the recurrence relation is,
an = 7/2*2n−1/2*6n.
3. What is the recurrence relation for 1, 7, 31, 127, 499?
a) bn+1=5bn-1+3
b) bn=4bn+7!
c) bn=4bn-1+3
d) bn=bn-1+1
View Answer

Answer: c
Explanation: Look at the differences between terms: 1, 7, 31, 124,…. and these are growing by a
factor of 4. So, 1⋅ 4=4, 7⋅ 4=28, 31⋅ 4=124, and so on. Note that we always end up with 3 less than
the next term. So, bn=4bn-1+3 is the recurrence relation and the initial condition is b0=1.
4. If Sn=4Sn-1+12n, where S0=6 and S1=7, find the solution for the recurrence
relation. a) an=7(2n)−29/6n6n
b) an=6(6n)+6/7n6n
c) an=6(3n+1)−5n
d) an=nn−2/6n6n
View Answer

Answer: b
Explanation: The characteristic equation of the recurrence relation is → x2−4x-12=0
So, (x-6)(x+2)=0. Only the characteristic root is 6. Therefore the solution to the recurrence relation will
have the form: an=a.6n+b.n.6n. To find a and b, set n=0 and n=1 to get a system of two equations with
two unknowns: 6=a60+b.0.60=a and 7=a61+b.1.61=2a+6b. Solving this system gives a=6 and b=6/7.
So the solution to the recurrence relation is, an=6(6n)−6/7n6n.
5. Find the value of a4 for the recurrence relation an=2an-1+3, with
a0=6. a) 320
b) 221
c) 141
d) 65
View Answer

Answer: c
Explanation: When n=1, a1=2a0+3, Now a2=2a1+3. By substitution, we get a2=2(2a0+3)+3.
Regrouping the terms, we get a4=141, where a0=6.
6. The solution to the recurrence relation an=an-1+2n, with initial term a0=2 are
a) 4n+7
b) 2(1+n)
c) 3n2
d) 5*(n+1)/2
View Answer

Answer: b
Explanation: When n=1, a1=a0+2. By substitution we get, a2=a1+2 ⇒ a2=(a0+2)+2 and so on. So
the solution to the recurrence relation, subject to the initial condition should be an=2+2n=2(1+n).
7. Determine the solution for the recurrence relation bn=8bn-1−12bn-2 with b0=3 and
b1=4. a) 7/2*2n−1/2*6n
b) 2/3*7n-5*4n
c) 4!*6n
d) 2/8n
View Answer

Answer: a
Explanation: Rewrite the recurrence relation bn-8bn-1+12bn-2=0. Now from the characteristic
equation: x2−8x+12=0 we have x: (x−2)(x−6)=0, so x=2 and x=6 are the characteristic roots.
Therefore the solution to the recurrence relation will have the form: bn=b2n+c6n. To find b and c, set
n=0 and n=1 to get a system of two equations with two unknowns: 3=b20+c60=b+c, and
4=b21+c61=2b+6c. Solving this system gives c=-1/2 and b=7/2. So the solution to the recurrence
relation is, bn=7/2*2n−1/2*6n.
8. What is the solution to the recurrence relation an=5an-1+6an-2?
a) 2n2
b) 6n
c) (3/2)n
d) n!*3
View Answer

Answer: b
Explanation: Check for the left side of the equation with all the options into the recurrence relation.
Then, we get that 6n is the required solution to the recurrence relation an=5an-1 + 6an-2.
9. Determine the value of a2 for the recurrence relation an = 17an-1 + 30n with a0=3.
a) 4387
b) 5484
c) 238
d) 1437
View Answer

Answer: d
Explanation: When n=1, a1=17a0+30, Now a2=17a1+30*2. By substitution, we get
a2=17(17a0+30)+60. Then regrouping the terms, we get a2=1437, where a0=3.
10. Determine the solution for the recurrence relation an = 6an-1−8an-2 provided
initial conditions a0=3 and a1=5.
a) an = 4 * 2n – 3n
b) an = 3 * 7n – 5*3n
c) an = 5 * 7n
d) an = 3! * 5n
View Answer

Answer: b
Explanation: The characteristic polynomial is x2−6x+8. By solving the characteristic equation,
x2−6x+8=0 we get x=2 and x=4, these are the characteristic roots. Therefore we know that the solution
to the recurrence relation has the form an=a*2n+b*4n, for some constants a and
b. Now, by using the initial conditions a0 and a1 we have: a=7/2 and b=-1/2. Therefore the solution to
the recurrence relation is: an = 4 * 2n – 1*3n = 7/2 * 2n – 1/2*3n.

1. Which of the following is contained in a recursive grammar?


a) semantic rules
b) production rules
c) recursive language
d) recursive
function View
Answer

Answer: b
Explanation: In natural language semantics, recursive grammar plays a vital role as well as in syntax.
A recursive grammar in a context free language is a formal grammar which consists of recursive
production rules.
2. is the consequence of dynamic programming.
a) Bellman equation
b) Frobenius equation
c) Linear equation
d) Boolean
expression View
Answer

Answer: a
Explanation: Dynamic programming can lead to recursive optimization that can restate a multistep
optimization problem in its recursive form. The Bellman equation that writes the value of the
optimization problem at an earlier time in terms of its value at a later time is the result of dynamic
programming.
3. How many types of self-referential recursive data are there in computer programs?
a) 6
b) 2
c) 10
d) 4
View Answer

Answer: b
Explanation: There are two types of self-referential definitions and these are inductive and coinductive
definitions. An inductively defined recursive data definition must have to specify how to construct
instances of the data. For example, linked lists are defined as an inductively recursive data definition.
4. recursion consists of multiple self-references.
a) binary recursion
b) single recursion
c) multiple recursion
d) coinductive recursion
View Answer

Answer: c
Explanation: A recursion which consists of multiple self-references and requires exponential time and
space is called multiple recursion. Multiple recursions include tree traversal of a graph, such as in a
depth-first search. However, single recursion is more efficient than multiple recursion.
5. The argument of each recursive call is the content of a field of the original output.
This definite characteristic belongs to which of the following function?
a) Structurally recursive function
b) Generativity recursive function
c) General function
d) Indirect recursive
function View Answer

Answer: a
Explanation: A structurally recursive function has a characteristic that the argument to each recursive
call is the content of a field of the original input. This recursion function includes mostly all tree
traversals which includes binary tree creation and search, XML processing etc.
6. The mutual recursion is also termed as
a) indirect recursion
b) constructive recursion
c) generative recursion
d) definitive
recursion View
Answer

Answer: a
Explanation: When a function is not called by itself but by another function which it has called either
directly or indirectly is termed as Indirect recursion. Mutual recursion is a more symmetric term of
Indirect recursion.
7. In which of the following problems recurrence relation holds?
a) Optimal substructure
b) Tower of Hanoi
c) Hallmark substitution
d) Longest common subsequence
View Answer

Answer: b
Explanation: We can have recurrence relation for tower of hanoi and that is hn = 2 hn-1 + 1h1
= 1, for n number of disks in one peg.
8. Which of the following functions generates new data at each step of a method?
a) corecursive function
b) structural recursive function
c) unirecursive function
d) indirect
function View
Answer

Answer: a
Explanation: The generatively recursive functions or corecursive functions is defined as generation of
the new data at each step that is successive approximation in Regula Falsi method. In terms of loop
variants, there is no such loop variant, and termination depends on error of approximation.
9. Every recursive algorithm must have the problem of
a) overhead of repeated function calls
b) collision of different function calls
c) searching for all duplicate elements
d) make only two recursive
calls View Answer

Answer: a
Explanation: Due to the overhead of repeated function calls and returns, recursive algorithms may be
inefficient for small data. Any recursion can be replaced by iteration with an explicit call stack
whereas iteration can be replaced with tail recursion.
10. If the height of a binary tree is 54, how many null pointers are there as
children? a) 1267
b) 358
c) 56
d) 255
View Answer

Answer: d
Explanation: Depth-first search (DFS) algorithm of a binary tree, is a trivial example of
short-circuiting. We can have a standard recursive algorithm in case of DFS. Now, a perfect binary
tree of height h has 2h+1 Null pointers as children.
h = 54
254+1
255.
1. How many binary relations are there on a set S
with 9 distinct elements?
a) 290
b) 2100
c) 281
d) 260
View Answer

Answer: c
Explanation: S is the set with 9 elements. A relation
on S is defined as S x S. There are 92 number of
ordered pairs in relation. So, the number of binary
relations is 2(9*9) = 281.
2. number of reflexive relations are
there on a set of 11 distinct elements.
a) 2110
b) 3121
c) 290

d) 2132
View Answer
Answer: a
Explanation: Let A be a set consists of n distinct
elements. There are 2(n*n)-n number of reflexive
relations that can be formed. So, here the answer is
2(11*11)-11 = 2110.
3. The number of reflexive as well as symmetric
relations on a set with 14 distinct elements is

a) 4120
b) 270
c) 3201
d) 291
View Answer

Answer: d
Explanation: Let A be a set consists of n distinct
elements. There are 2(n*(n-1))/2 number of
reflexive and symmetric relations that can be
formed. So, here the answer is 214*(14-1)/2 = 291.
4. The number of symmetric relations on a set with
15 distinct elements is
a) 2196
b) 250
c) 2320
d) 278
View Answer
Answer: a
Explanation: Let S be a set consists of n distinct
elements. There are 2(n-1)*(n-1) number of
reflexive and symmetric relations that can be
formed. So, here the answer is 2(15-1)*(15-1) =
2196.
5. Suppose S is a finite set with 7 elements. How
many elements are there in the largest
equivalence relation on S?
a) 56
b) 78
c) 49
d) 100
View Answer

Answer: c
Explanation: Let R is an equivalence relation on the
set S and so it satisfies the reflexive, symmetric and
transitive property. The largest equivalence relation
means it should contain the largest number of
ordered pairs. Since we can have n2 ordered pairs in
R x R where n belongs to S and all these ordered
pairs are present in this relation; its the largest
equivalence relation.So there are n2 elements i.e 72
= 49 elements in the largest equivalence relation.
6. is the rank of the largest
equivalence relation on a set of 20 elements.
a) 320
b) 2400
c) 20
d) 1
View Answer

Answer: d
Explanation: The rank of an equivalence relation is
the number of an equivalence classes. If we have a1,
a2, a3, …, an elements then a1 and a2 will be in the
same equivalence class because everything is
related and so on. In this case, there is only one
equivalence class.
7. How many elements are there in the smallest
equivalence relation on a set with 8 elements?
a) 102
b) 8
c) 48
d) 32
View Answer

Answer: b
Explanation: Let R is an equivalence relation on the
set S with n elements and so it satisfies reflexive,
symmetric and transitive properties. The smallest
equivalence relation means it should contain
minimum number of ordered pairs i.e along with
symmetric and transitive properties it must always
satisfy reflexive property. So, the smallest
equivalence relation will have n ordered pairs and
so the answer is 8.
8. The rank of smallest equivalence relation on a set
with 12 distinct elements is
a) 12
b) 144
c) 136
d) 79
View Answer

Answer: a
Explanation: In the case of smallest equivalence
relation, each element is in one equivalence class
like {a1}, {a2}, … are equivalence classes. So, the
rank or number of equivalence classes is n for a set
with n elements and so the answer is 12.
9. If a set A has 8 elements and a set B has 10
elements, how many relations are there from A
to B?
a) 290
b) 380
c) 164
d) 280
View Answer

Answer: d
Explanation: Let, a relation R from A to B is a
subset of A×B. As the maximum number of subsets
(Elements in the powerset) is 2mn, there are 2mn
number of relations from A to B and so the
answer is 280.
10. Synonym for binary relation is
a) equivalence relation
b) dyadic relation
c) orthogonal relation
d) one to many relations
View Answer

Answer: b
Explanation: A binary relation on a set S is a set of
ordered pairs of elements of S. It is a subset of the
cartesian product S2 = S x S. The terms
correspondence, dyadic relation and 2-place relation
are synonyms for the binary relation

1. R is a binary relation on a set S and R is reflexive


if and only if
a) r(R) = R
b) s(R) = R
c) t(R) = R
d) f(R) = R
View
Answer

Answer: a
Explanation: Let reflexive closure of R:r(R) = R. If
R is reflexive, it satisfies all the condition in the
definition of reflexive closure. So, a reflexive
closure of a relation is the smallest number of
reflexive relation contain in R. Hence, R = r(R).
2. If R1 and R2 are binary relations from set A to
set B, then the equality holds.
a) (Rc)c = Rc
b) (A x B)c = Φ
c) (R1 U R2)c = R1c ∪ R2c
d) (R1 U R2)c = R1c ∩ R2c
View Answer

Answer: c
Explanation: To proof (R1 U R2)c = R1c ∪ R2c,
if <x,y> belongs to (R1 U R2)c
⇔ <y,x> ∈ (R1 U R2)
⇔ <y,x> ∈ R1 or <y,x> ∈ R2
⇔ <x,y> ∈ R1c or <x,y> ∈ R2c
⇔ <x,y> ∈ R1c ∪ R2c.
3. The condition for a binary relation to be
symmetric is
a) s(R) = R
b) R ∪ R = R
c) R = Rc
d) f(R) = R
View
Answer

Answer: c
Explanation: If <a,b> ∈ R then <b,a> ∈ R, where
a and b belong to two different sets and so its
symmetric.
Rc also contains <b,a>
Rc = R.
4. number of reflexive closure exists in a
relation R = {(0,1), (1,1), (1,3), (2,1), (2,2), (3,0)}
where {0, 1, 2, 3} ∈ A.
a) 26
b) 6
c) 8
d) 36
View Answer

Answer: b
Explanation: The reflexive closure of R is the
relation, R ∪ Δ = { (a,b) | (a,b) R (a,a) | a A }.
Hence, R ∪ Δ = {(0,1), (1,1), (1,3), (2,1), (2,2),
(3,0)} and the answer is 6.
5. The transitive closure of the relation {(0,1), (1,2),
(2,2), (3,4), (5,3), (5,4)} on the set {1, 2, 3, 4, 5} is

a) {(0,1), (1,2), (2,2), (3,4)}


b) {(0,0), (1,1), (2,2), (3,3), (4,4), (5,5)}
c) {(0,1), (1,1), (2,2), (5,3), (5,4)}
d) {(0,1), (0,2), (1,2), (2,2), (3,4), (5,3), (5,4)}
View Answer
Answer: d
Explanation: Let R be a relation on a set A. The
connectivity relation on R* consists of pairs (a,b)
such that there is a path of length at least one from a
to b in R. Mathematically, R* = R1 ∪ R2 ∪ R3
∪ … ∪ Rn. Hence the answer is {(0,1), (0,2), (1,2),
(2,2), (3,4), (5,3), (5,4)}.
6. Amongst the properties {reflexivity, symmetry,
antisymmetry, transitivity} the relation R={(a,b) ∈
N2 | a!= b} satisfies property.
a) symmetry
b) transitivity
c) antisymmetry
d) reflexivity
View
Answer

Answer: a
Explanation: It is not reflexive as aRa is not
possible. It is symmetric as if aRb then bRa. It is not
antisymmetric as aRb and bRa are possible and we
can have a!=b. It is not transitive as if aRb and bRc
then aRc need not be true. This is violated when c=a.
So the answer is symmetry property.
7. The number of equivalence relations of the set {3,
6, 9, 12, 18} is
a) 4
b) 25
c) 22
d) 90
View Answer

Answer: a
Explanation: Number of equivalence Relations are
given by BELL number. The nth of these numbers
i.e, Bn counts the number of different ways to
partition a set that has exactly n elements, or
equivalently, the number of equivalence relations on
it. Let’s say, 1 -> Equivalence relation with 1
element; 1 2 -> Equivalence relation with 2 element;
2 3 5 -> Equivalence relation with 3 element; 5 7 10
15 -> Equivalence relation with 4 element. Hence,
the answer is 4.
8. Let R1 and R2 be two equivalence relations on a
set. Is R1 ∪ R2 an equivalence relation?
a) an equivalence relation
b) reflexive closure of relation
c) not an equivalence relation
d) partial equivalence relation
View Answer

Answer: a
Explanation: R1 union R2 is not equivalence
relation because transitivity property of closure need
not hold. For instance, (x, y) can be in R1 and (y, z)
be in R2 and (x, z) not in either R1 or R2. However,
R1 intersection R2 is an equivalence relation.
9. A relation R is defined on the set of integers
as aRb if and only if a+b is even and R is termed
as

a) an equivalence relation with one equivalence


class
b) an equivalence relation with two equivalence
classes
c) an equivalence relation
d) an equivalence relation with three equivalence
classes
View Answer

Answer: b
Explanation: R is reflexive as (a+b) is even for any
integer; R is symmetric as if (a+b) is even (b+a) is
also even; R is transitive as if ((a+b)+c) is even,
then (a+(b+c)) is also even.
So, R is an equivalence relation. For set of natural
numbers, sum of even numbers always give even,
sum of odd numbers always give even and sum of
any even and any odd number always give odd. So,
must have two equivalence classes -> one for even
and one for odd.
{…, -4, -2, 0, 2, … } and {…, -3, -1, 1, 3, … }.
10. The binary relation U = Φ (empty set) on a set A
= {11, 23, 35} is
a) Neither reflexive nor symmetric
b) Symmetric and reflexive
c) Transitive and reflexive
d) Transitive and symmetric
View Answer

Answer: d
Explanation: U = Φ (empty set) on a set A = {11, 23,
35} need to be hold Irreflexive, symmetric,
anti-symmetric, asymmetric and transitive closure
property, but it is not Reflexive as it does not
contain any self loop in itself

1. The binary relation {(1,1), (2,1), (2,2), (2,3), (2,4),


(3,1), (3,2)} on the set {1, 2, 3} is
a) reflective, symmetric and transitive
b) irreflexive, symmetric and transitive
c) neither reflective, nor irreflexive but transitive
d) irreflexive and antisymmetric
View Answer

Answer: c
Explanation: Not reflexive -> (3,3) not present; not
irreflexive -> (1, 1) is present; not symmetric -> (2,
1) is present but not (1, 2); not antisymmetric – (2, 3)
and (3, 2) are present; not asymmetric -> asymmetry
requires both antisymmetry and irreflexivity. So, it
is transitive closure of relation.
2. Consider the relation: R’ (x, y) if and only if x,
y>0 over the set of non-zero rational numbers,then
R’ is
a) not equivalence relation
b) an equivalence relation
c) transitive and asymmetry relation
d) reflexive and antisymmetric relation
View Answer

Answer: b
Explanation: Reflexive: a, a>0
Symmetric: if a, b>0 then both must be +ve or -ve,
which means b, a > 0 also exists
Transitive: if a, b>0 and b, c>0 then to have b as
same number, both pairs must be +ve or -ve which
implies a, c>0. Hence, R’ is an equivalence relation.
3. Let S be a set of n>0 elements. Let be the number
Br of binary relations on S and let Bf be the number
of functions from S to S. The expression for Br and
Bf, in terms of n should be
a) n2 and 2(n+1)2
b) n3 and n(n+1)
c) n and n(n+6)
d) 2(n*n) and nn
View Answer
Answer: d
Explanation: For a set with n elements the number
of binary relations should be 2(n*n) and the number
of functions should be nn. Hence Br = 2(n*n) and
Bf = nn.
4. Let A be a set of k (k>0) elements. Which is
larger between the number of binary relations (say,
Nr) on A and the number of functions (say, Nf)
from A to A?
a) number of relations
b) number of functions
c) the element set
d) number of subsets of the relation
View Answer

Answer: a
Explanation: For a set with k elements the number
of binary relations should be 2(n*n) and the number
of functions should be nn. Now, 2(n*n) => n2log (2)
[taking log] and nn => nlog (n) [taking log]. It is
known that n2log (2) > nlog (n). Hence, the number
of binary relations > the number of functions i.e, Nr
> Nf.
5. Consider the binary relation, A = {(a,b) | b = a –
1 and a, b belong to {1, 2, 3}}. The reflexive
transitive closure of A is?
a) {(a,b) | a >= b and a, b belong to {1, 2, 3}}
b) {(a,b) | a > b and a, b belong to {1, 2, 3}}
c) {(a,b) | a <= b and a, b belong to {1, 2, 3}}
d) {(a,b) | a = b and a, b belong to {1, 2, 3}}
View Answer

Answer: a
Explanation: By definition of Transitive closure we
have that a is related to all smaller b (as every a is
related to b – 1) and from the reflexive property a is
related to a.
6. Let R1 be a relation from A = {1, 3, 5, 7} to B =
{2, 4, 6, 8} and R2 be another relation from B to C
= {1, 2, 3, 4} as defined below:
i. An element a in A is related to an element b in
B (under R1) if a * b is divisible by 3.
ii. An element a in B is related to an element b in C
(under R2) if a * b is even but not divisible by 3.
Which is the composite relation R1R2 from A to
C? a) R1R2 = {(1, 2), (1, 4), (3, 3), (5, 4), (5,6), (7,
3)}
b) Φ
c) R1R2 = {(1, 2), (1,6), (3, 2), (3, 4), (5, 4), (7, 2)}
d) R1R2 = {(2,2), (3, 2), (3, 4), (5, 1), (5, 3), (7, 1)}
View Answer

Answer: b
Explanation: By definition, i) R1 = {(1,6), (3,2),
(3,4), (3,6), (3,8), (5,6), (7,6)} and ii) R2 = {(1,2),
(1,4), (1,8), (5,2), (5,4), (5,8), (7,2), (7,4), (7,8)}. So,
R1R2 = Φ.
7. The time complexity of computing the transitive
closure of a binary relation on a set of n elements
should be
a) O(n)
b) O(logn)
c) O(n(n+(3/2)))
d) O(n3)
View Answer

Answer: d
Explanation: Calculation of transitive closure results
into matrix multiplication. We can do matrix
multiplication in O(n3) time. There are better
algorithms that do less than cubic time.
8. Let A and B be two non-empty relations on a set
S. Which of the following statements is false?
a) A and B are transitive ⇒ A∩B is transitive
b) A and B are symmetric ⇒ A∪B is symmetric
c) A and B are transitive ⇒ A∪B is not transitive
d) A and B are reflexive ⇒ A∩B is
reflexive View Answer

Answer: c
Explanation: In terms of set theory, the binary
relation R defined on the set X is a transitive
relation if, for all a, b, c ∈ X, if aRb and bRc, then
aRc. If there are two relations on a set satisfying
transitive property then there union must satisfy
transitive property.
9. Determine the characteristics of the relation aRb
if a2 = b2.
a) Transitive and symmetric
b) Reflexive and asymmetry
c) Trichotomy, antisymmetry, and irreflexive
d) Symmetric, Reflexive, and transitive
View Answer

Answer: d
Explanation: Since, x2 = y2 is just a special case of
equality, so all properties that apply to x = y also
apply to this case. Hence, the relation satisfies
symmetric, reflexive and transitive closure.
10. Let R be a relation between A and B. R
is asymmetric if and only if
a) Intersection of D(A) and R is empty, where D(A)
represents diagonal of set
b) R-1 is a subset of R, where R-1 represents
inverse of R
c) Intersection of R and R-1 is D(A)
d) D(A) is a subset of R, where D(A) represents
diagonal of set
View Answer

Answer: a
Explanation: A relation is asymmetric if and only if
it is both antisymmetric and irreflexive. As a
consequence, a relation is transitive and asymmetric
if and only if it is a strict partial order. If D(A) is a
diagonal of A set and intersection of D(A) and R is
empty, then R is asymmetric.

1. Let a set S = {2, 4, 8, 16, 32} and <= be the


partial order defined by S <= R if a divides b.
Number of edges in the Hasse diagram of is
a) 6
b) 5
c) 9
d) 4
View Answer

Answer: b
Explanation: Hasse Diagram is:
32
/
16
/
8
/ \
2 4
So, the number of edges should be: 4.
2. The less-than relation, <, on a set of real numbers
is
a) not a partial ordering because it is not asymmetric
and irreflexive equals antisymmetric
b) a partial ordering since it is asymmetric and
reflexive
c) a partial ordering since it is antisymmetric and
reflexive
d) not a partial ordering because it is not
antisymmetric and reflexive
View Answer

Answer: a
Explanation: Relation less than a set of real numbers
is not antisymmetric and reflexive. Relation is not
POSET because it is irreflexive. Again, aRb != bRa
unless a=b and so it is antisymmetric. A relation
may be ‘not asymmetric and not reflexive but still
antisymmetric, as {(1,1) (1,2)}. So, the relation is
not a partial ordering because it is not asymmetric
and irreflexive equals antisymmetric.
3. If the longest chain in a partial order is of length l,
then the partial order can be written as
disjoint antichains.
a) l2
b) l+1
c) l
d) ll
View Answer

Answer: c
Explanation: If the length of the longest chain in a
partial order is l, then the elements in the POSET
can be partitioned into l disjoint antichains.
4. Suppose X = {a, b, c, d} and π1 is the partition of
X, π1 = {{a, b, c}, d}. The number of ordered pairs
of the equivalence relations induced by
a) 15
b) 10
c) 34
d) 5
View Answer

Answer: b
Explanation: The ordered pairs of the equivalence
relations induced = {(a,a), (a,b), (a,c), (b,a), (b,b),
(b,c), (c,a), (c,b), (c,c), (d,d)}. Poset -> equivalence
relations = each partition power set – Φ.
5. A partial order P is defined on the set of natural
numbers as follows. Here a/b denotes integer
division. i)(0, 0) ∊ P. ii)(a, b) ∊ P if and only if
a
% 10 ≤ b % 10 and (a/10, b/10) ∊ P. Consider the
following ordered pairs:
i. (101, 22) ii. (22, 101) iii. (145, 265) iv. (0, 153)
The ordered pairs of natural numbers are contained
in P are and
a) (145, 265) and (0, 153)
b) (22, 101) and (0, 153)
c) (101, 22) and (145, 265)
d) (101, 22) and (0, 153)
View Answer

Answer: d
Explanation: For ordered pair (a, b), to be in P, each
digit in a starting from unit place must not be larger
than the corresponding digit in b. This condition is
satisfied by options (iii) (145, 265) => 5 ≤ 5, 4 < 6
and 1 < 2; (iv) (0, 153) => 0 < 3 and no need to
examine further.
6. The inclusion of sets into R = {{1, 2}, {1,
2, 3}, {1, 3, 5}, {1, 2, 4}, {1, 2, 3, 4, 5}} is
necessary and sufficient to make R a complete
lattice under the partial order defined by set
containment.
a) {1}, {2, 4}
b) {1}, {1, 2, 3}
c) {1}
d) {1}, {1, 3}, {1, 2, 3, 4}, {1, 2, 3, 5}
View Answer

Answer: c
Explanation: A lattice is complete if every subset of
partial order set has a supremum and infimum
element. For example, here we are given a partial
order set R. Now it will be a complete lattice if
whatever be the subset we choose, it has a
supremum and infimum element. Here relation
given is set containment, so supremum element will
be just union of all sets in the subset we choose.
Similarly, the infimum element will be just an
intersection of all the sets in the subset we choose.
As R now is not complete lattice, because although
it has a supremum for every subset we choose, but
some subsets have no infimum. For example, if we
take subset {{1, 3, 5}, {1, 2, 4}}, then intersection
of sets in this is {1}, which is not present in R. So
clearly, if we add set {1} in R, we will solve the
problem. So adding {1} is necessary and sufficient
condition for R to be a complete lattice.
7. Consider the ordering relation a | b ⊆ N x N over
natural numbers N such that a | b if there exists c
belong to N such that a*c=b. Then
a) | is an equivalence relation
b) It is a total order
c) Every subset of N has an upper bound under |
d) (N,|) is a lattice but not a complete
lattice View Answer

Answer: d
Explanation: A set is called lattice if every finite
subset has a least upper bound and greatest lower
bound. It is termed as a complete lattice if every
subset has a least upper bound and greatest lower
bound. As every subset of this will not have LUB
and GLB so (N,|) is a lattice but not a complete
lattice.
8. Consider the set N* of finite sequences of natural
numbers with a denoting that sequence a is a prefix
of sequence b. Then, which of the following is
true?
a) Every non-empty subset of has a greatest lower
bound
b) It is uncountable
c) Every non-empty finite subset of has a least
upper bound
d) Every non-empty subset of has a least upper
bound
View Answer

Answer: a
Explanation: Consider any sequence like “45, 8, 7,
2” – it can have many (infinite) least upper bounds
like “45, 8, 7, 2, 5”, “45, 8, 7, 2, 1” and so on but it
can have only 1 greatest lower bound – “45, 8, 7”
because we are using the prefix relation. So, every
non-empty subset has a greatest lower bound.
9. A partial order ≤ is defined on the set S = {x, b1,
b2, … bn, y} as x ≤ bi for all i and bi ≤ y for all i,
where n ≥ 1. The number of total orders on the set
S which contain the partial order ≤ is
a) n+4
b) n2
c) n!
d) 3
View Answer

Answer: c
Explanation: To make this partial order a total order,
we need the relation to hold for every two element
of the partial order. Currently, there is no relation
between any bi and bj. So, for every bi and bj, we
have to add either (bi, bj) or (bj, bi) in total order.
So, this translates to giving an ordering for n
elements between x and y, which can be done in n!
ways.
10. Let (A, ≤) be a partial order with two minimal
elements a, b and a maximum element c. Let P:A –
>
{True, False} be a predicate defined on A. Suppose
that P(a) = True, P(b) = False and P(a) ⇒ P(b) for
all satisfying a ≤ b, where ⇒ stands for logical
implication. Which of the following statements
cannot be true?
a) P(x) = True for all x S such that x ≠ b
b) P(x) = False for all x ∈ S such that b ≤ x and x ≠
c
c) P(x) = False for all x ∈ S such that x ≠ a and x ≠
c
d) P(x) = False for all x ∈ S such that a ≤ x and b ≤
x
View Answer

Answer: d
Explanation: Here, maximum element is c and so c
is of a higher order than any other element in A.
Minimal elements are a and b: No other element in
A is of lower order than either a or b.
We are given P(a) = True. So, for all x such that a≤x,
P(x) must be True. We do have at least one such x,
which is c as it is the maximum element. So, P(x) =
False for all x ∈ S such that a ≤ x and b ≤ x -> cannot
be true. P(x) = True for all x S such that x ≠
b -> can be True as all elements mapped to TRUE
doesn’t violate the given implication. P(x) = False
for all x ∈ S such that x ≠ a and x ≠ c -> can be
True if a is related only to c. P(x) = False for all x
∈ S such that b ≤ x and x ≠ c -> can be True as b≤x
ensures x≠a and for all other elements P(x) can be
False without violating the given implication

1. Suppose a relation R = {(3, 3), (5, 5), (5, 3), (5, 5),
(6, 6)} on S = {3, 5, 6}. Here R is known as

a) equivalence relation
b) reflexive relation
c) symmetric relation
d) transitive relation
View Answer

Answer: a
Explanation: Here, [3] = {3, 5}, [5] = {3, 5}, [5] =
{5}. We can see that [3] = [5] and that S/R will be
{[3], [6]} which is a partition of S. Thus, we can
choose either {3, 6} or {5, 6} as a set of
representatives of the equivalence classes.
2. Consider the congruence 45≡3(mod 7). Find the
set of equivalence class representatives.
a) {…, 0, 7, 14, 28, …}
b) {…, -3, 0, 6, 21, …}
c) {…, 0, 4, 8, 16, …}
d) {…, 3, 8, 15, 21, …}
View Answer

Answer: a
Explanation: Note that a set of class representatives
is the subset of a set which contains exactly one
element from each equivalence class. Now, for
integers n, a and b, we have congruence a≡b(mod n),
then the set of equivalence classes are {…, -2n, -n,
0, n, 2n,…}, {…, 1-2n, 1-n, 1, 1+n, 1+2n,…}. The
required answer is {…, 0, 7, 14, 28, …}.
3. Which of the following relations is the reflexive
relation over the set {1, 2, 3, 4}?
a) {(0,0), (1,1), (2,2), (2,3)}
b) {(1,1), (1,2), (2,2), (3,3), (4,3), (4,4)}
c) {,(1,1), (1,2), (2,1), (2,3), (3,4)}
d) {(0,1), (1,1), (2,3), (2,2), (3,4), (3,1)
View Answer

Answer: b
Explanation: {(1,1), (1,2), (2,2), (3,3), (4,3), (4,4)}
is a reflexive relation because it contains set =
{(1,1), (2,2), (3,3), (4,4)}.
4. Determine the partitions of the set {3, 4, 5, 6, 7}
from the following subsets.
a) {3,5}, {3,6,7}, {4,5,6}
b) {3}, {4,6}, {5}, {7}
c) {3,4,6}, {7}
d) {5,6}, {5,7}
View Answer

Answer: b
Explanation: {3,5}, {3,6,7}, {4,5,6}. It is not a
partition because these sets are not pairwise disjoint.
The elements 3, 5 and 6 appear repeatedly these sets.
{1}, {2,3,6}, {4}, {5} – this is a partition as they
are pairwise disjoint. {3,4,6}, {7} – this is not a
partition as element 5 is missing.
{5,6}, {5,7} – this is not a partition because it is
missing the elements 3, 4 in any of the sets.
5. Determine the number of equivalence classes that
can be described by the set {2, 4, 5}.
a) 125
b) 5
c) 16
d) 72
View Answer

Answer: b
Explanation: Suppose B={2, 4, 5} and B×B = (2,2),
(4,4), (5,5), (2,4), (4,2), (4,5), (5,4), (2,5), (5,2). A
relation R on set B is said to be equivalence relation
if R is reflexive, Symmetric, transitive. Hence, total
number of equivalence relation=5 out of 23=8
relations.
6. Determine the number of possible relations in an
antisymmetric set with 19 elements.
a) 23585
b) 2.02 * 1087
c) 9.34 * 791
d) 35893
View Answer

Answer: b
Explanation: Number of antisymmetric relation is
given:-|A|=n, |AxA|=n xn. Then, N=total number of
diagonal will n and we know that N = 2n *
3(n2-n)/2. So, the number of relations should be =
2.02 * 1087.
7. For a, b ∈ Z define a | b to mean that a divides b
is a relation which does not satisfy
a) irreflexive and symmetric relation
b) reflexive relation and symmetric relation
c) transitive relation
d) symmetric relation
View Answer

Answer: b
Explanation: Suppose, a=0, then we know that 0
does not divide 0, 0 ∤ 0 and it is not reflexive.
Again, 2 | 4 but 4 does not 2 and so it is not a
symmetric relation.
8. Which of the following is an equivalence relation
on R, for a, b ∈ Z?
a) (a-b) ∈ Z
b) (a2+c) ∈ Z
c) (ab+cd)/2 ∈ Z
d) (2c3)/3 ∈ Z
View Answer

Answer: b
Explanation: Let a ∈ R, then a−a = 0 and 0 ∈ Z, so
it is reflexive. To see that a-b ∈ Z is symmetric,
then a−b ∈ Z -&gt say, a−b = m, where m ∈ Z ⇒
b−a = −(a−b)=−m and −m ∈ Z. Thus, a-b is
symmetric. To see that a-b is transitive, let a, b, c ∈
R. Thus, a−b ∈ Z; b−c ∈ Z. Let a−b = i and b−c = j,
for integers i,j ∈ Z. Then a−c ='(a−b)+(b−c)=i + j.
So, a−c ∈ Z. Therefore a – c is transitive. Hence,
(a-b) is an equivalence relation on the set R. Rest of
the options are not equivalence relations.
9. Determine the set of all integers a such that a ≡ 3
(mod 7) such that −21 ≤ x ≤ 21.
a) {−21, −18, −11, −4, 3, 10, 16}
b) {−21, −18, −11, −4, 3, 10, 17, 24}
c) {−24, -19, -15, 5, 0, 6, 10}
d) {−23, −17, −11, 0, 2, 8, 16}
View Answer

Answer: b
Explanation: For an integer a we have x ≡ 3 (mod 7)
if and only if a = 7m + 3. Thus, by calculating
multiples of 7, add 3 and restrict the value of a, so
that −21 ≤ x ≤ 21. The set for a = {−21, −18, −11,
−4, 3, 10, 17, 24}.
10. For a, b ∈ R define a = b to mean that |x| = |y|.
If [x] is an equivalence relation in R. Find the
equivalence relation for [17].
a) {,…,-11, -7, 0, 7, 11,…}
b) {2, 4, 9, 11, 15,…}
c) {-17, 17}
d) {5, 25, 125,…}
View Answer

Answer: c
Explanation: We can find that [17] = {a ∈ R|a = 17}
= {a ∈ R||a| = |17|} = {-17, 17} and [−17] = {a ∈ R|a
= −17} = {a ∈ R||a| = |−17|}= {−17, 17}. Hence, the
required equivalence relation is {-17, 17}

1. A directed graph or digraph can have directed cycle in which


a) starting node and ending node are different
b) starting node and ending node are same
c) minimum four vertices can be there
d) ending node does not exist
View Answer

Answer: b
Explanation: If the start node and end node are same in the path of a graph then it is
termed as directed cycle i.e, c0 = cn. For instance, a c b a is a simple cycle in which start
and end nodes are same(a). But, a c b b a is not a simple cycle as there is a loop <b,b>.
2. Let, D = <A, R> be a directed graph or digraph,then D’ = <A’, R’> is a subgraph if

a) A’ ⊂ A and R’ = R ∩ (A’ x A’)


b) A’ ⊂ A and R ⊂ R’ ∩ (A’ x A’)
c) R’ = R ∩ (A’ x A’)
d) A’ ⊆ A and R ⊆ R’ ∩ (A’ x
A’) View Answer

Answer: a
Explanation: A directed graph or digraph is an ordered pair D<A, R> where A(is a set of
nodes of D) is a set and R(the elements of R are the arcs of D) is a binary relation on A.
The relation R is called the incidence relation on D. Now, a digraph is a subgraph of D if
i)A’ ⊂ A and ii)R’ = R ∩ (A’ x A’). If D’ D, D’ is a proper subgraph of D.
3. The graph representing universal relation is called
a) complete digraph
b) partial digraph
c) empty graph
d) partial subgraph
View Answer

Answer: a
Explanation: Consider, A is a graph with vertices {a, b, c, d} and the universal relation is
A x A. The graph representing universal relation is called a complete graph and all
ordered pairs are present there.
4. What is a complete digraph?
a) connection of nodes without containing any cycle
b) connecting nodes to make at least three complete cycles
c) start node and end node in a graph are same having a cycle
d) connection of every node with every other node including itself in a digraph
View Answer

Answer: d
Explanation: Every node should be connected to every other node including itself in a
digraph is the complete digraph. Now, graphs are connected, strongly connected and
disconnected
5. Disconnected components can be created in case of
a) undirected graphs
b) partial subgraphs
c) disconnected graphs
d) complete graphs
View Answer

Answer: c
Explanation: By the deletion of one edge from either connected or strongly connected
graphs the graph obtained is termed as a disconnected graph. It can have connected
components separated by the deletion of the edges. The edge that has to be deleted called
cut edge.
6. A simple graph can have
a) multiple edges
b) self loops
c) parallel edges
d) no multiple edges, self-loops and parallel
edges View Answer

Answer: d
Explanation: If a graph say G = <V, E> has no parallel or multiple edges and no self
loops contained in it is called a simple graph. An undirected graph may have multiple
edges and self-loops.
7. Degree of a graph with 12 vertices is
a) 25
b) 56
c) 24
d) 212
View Answer

Answer: c
Explanation: Number of edges incident on a graph is known as degree of a vertex. Sum
of degrees of each vertex is called total degree of the graph. Total degree = 2 * number of
vertices. So, if there are 24 vertices then total degree is 24.
8. In a finite graph the number of vertices of odd degree is always
a) even
b) odd
c) even or odd
d) infinite
View Answer

Answer: a
Explanation: In any finite graph, sum of degree of all the vertices = 2 * number of edges.
Sum of degree of all the vertices with even degree + sum of degree of all the vertices with
odd degree = 2 * number of edges. Now, even number + sum of degree of all the vertices
with odd degree = even number. It is possible if and only if number of odd degree
vertices are even.
9. An undirected graph has 8 vertices labelled 1, 2, …,8 and 31 edges. Vertices 1, 3, 5, 7
have degree 8 and vertices 2, 4, 6, 8 have degree 7. What is the degree of vertex 8?
a) 15
b) 8
c) 5
d) 23
View Answer
Answer: b
Explanation: Vertices 1, 3, 5, 7 have degree 8 and vertices 2, 4, 6, 8 have degree 7. By
definition, sum of degree= 2 * No of edges
Let x = degree of vertex 8
8 + 7 + 8 + 7 + 8 + 7 + 8 + x = 2 * 31
53 + x = 61
x=8
Hence, degree of vertex 8 is 8.
10. G is an undirected graph with n vertices and 26 edges such that each vertex of G has a
degree at least 4. Then the maximum possible value of n is
a) 7
b) 43
c) 13
d) 10
View Answer

Answer: c
Explanation: Let m be min degree and M be a max degree of a graph, then m ≤ 2E/V ≤ M.
Here, m=4, E=26, v=?
So, 4 ≤ (2*26)/V
V ≤ (52/4)
V ≤ 13 ⇒ V = 13.

This set of Discrete Mathematics Multiple Choice Questions & Answers (MCQs) focuses
on “Graphs – Hasse Diagrams”.

1. Hasse diagrams are first made by


a) A.R. Hasse
b) Helmut Hasse
c) Dennis Hasse
d) T.P. Hasse
View Answer

Answer: b
Explanation: Hasse diagrams can be described as the transitive reduction as an abstract
directed acyclic graph. This graph drawing techniques are constructed by Helmut
Hasse(1948).
2. If a partial order is drawn as a Hasse diagram in which no two edges cross, its covering
graph is called
a) upward planar
b) downward planar
c) lattice
d) biconnected components
Answer: a
Explanation: In a Hasse diagram if no two edges cross each other in the drawing of
partial order Hasse diagram, then its covering graph called the upward planar.
3. If the partial order of a set has at most one minimal element, then to test whether it has
a non-crossing Hasse diagram its time complexity
a) NP-complete
b) O(n2)
c) O(n+2)
d) O(n3)
View Answer

Answer: a
Explanation: If the partial order has at most one minimal element, or it has at most one
maximal element, then to test whether a partial order with multiple sources and sinks can
be drawn as a crossing-free Hasse diagram or not it’s time complexity is NP-complete.
4. Which of the following relation is a partial order as well as an equivalence relation?
a) equal to(=)
b) less than(<)
c) greater than(>)
d) not equal to(!=)
View Answer

Answer: a
Explanation: The identity relation = on any set is a partial order in which every two
distinct elements are incomparable and that depicts the relation of both a partial order and
an equivalence relation. For non-linear orders, there are many advanced properties of
posets.
5. The relation ≤ is a partial order if it is
a) reflexive, antisymmetric and transitive
b) reflexive, symmetric
c) asymmetric, transitive
d) irreflexive and transitive
View Answer

Answer: a
Explanation: Let A is a set and ≤ is a relation on A, then ≤ is a partial order if it satisfies
reflexive, antisymmetric, and transitive, i.e., for all x, y and z in P. That means, x ≤ x
(reflexivity);
if x ≤ y and y ≤ x then x = y (antisymmetry) and if x ≤ y and y ≤ z then x ≤ z
(transitivity).
6. In which of the following relations every pair of elements is comparable?
a) ≤
b) !=
c) >=
d) ==
View Answer

Answer: a
Explanation: In the ≤(or less than and equal to) relation, every pair of elements is
comparable.
7. In a poset (S, ⪯ ), if there is no element n∈S with m<n, then which of the following is
true?
a) an element n exists for which m=n
b) An element m is maximal in the poset
c) A set with the same subset of the poset
d) An element m is minimal in the poset
View Answer

Answer: b
Explanation: By the definition, an element m exists in a poset (S, ⪯ ) is maximal if and
only if there is no n∈S with m≺ n.
8. In a poset P({v, x, y, z}, ⊆) which of the following is the greatest element?
a){v, x, y, z}
b)1
c)∅
d){vx, xy, yz}
View Answer

Answer: a
Explanation: We know that, in a Hasse diagram, the maximal element(s) are the top
and the minimal elements are at the bottom of the diagram. In the given poset, {v, x, y,
z} is the maximal or greatest element and ∅ is the minimal or least element.
9. Suppose P1 is a partially ordered class and a cut of P1 is pair (D, T) of
nonempty subclasses of P1 satisfies which of the following properties?
a) D∩T=Ø
b) D∪T=P1
c) xyz∈T
d) z∈T and zx∈D
View Answer

Answer: a
Explanation: Suppose P1 is a partially ordered class and a cut of P1 is pair (D, T) of
nonempty subclasses of P1 satisfies the following properties: i) D∩T=Ø and D∪T=P1 ii)
If z∈D and y≤z, then y∈D and iii) If z∈T and y≥z, then y∈T.
10. Let G be the graph defined as the Hasse diagram for the ⊆ relation on the set S{1,
2,…, 18}. How many edges are there in G?
a) 43722
b) 2359296
c) 6487535
d) 131963
View Answer

Answer: b
Explanation: Here the total number of elements in S is 18 and so number of vertices in
Hasse diagram are 218. Hence, the number of edges in Hasse diagram are 18 *
218-1=2359296.

1. A Poset in which every pair of elements has both a least upper bound and a
greatest lower bound is termed as
a) sublattice
b) lattice
c) trail
d) walk
View Answer

Answer: b
Explanation: A poset in which every pair of elements has both a least upper bound and a
greatest lower bound is called a lattice. A lattice can contain sublattices which are subsets
of that lattice.
2. In the poset (Z+, |) (where Z+ is the set of all positive integers and | is the divides
relation) are the integers 9 and 351 comparable?
a) comparable
b) not comparable
c) comparable but not determined
d) determined but not comparable
View Answer

Answer: a
Explanation: The two integers 9 and 351 are comparable since 9|351 i.e, 9 divides 351.
But 5 and 127 are not comparable since 5 | 127 i.e 5 does not divide 127.
3. If every two elements of a poset are comparable then the poset is called
a) sub ordered poset
b) totally ordered poset
c) sub lattice
d) semigroup
View Answer

Answer: b
Explanation: A poset (P, <=) is known as totally ordered if every two elements of the
poset are comparable. “<=” is called a total order and a totally ordered set is also termed
as a chain.
4. and are the two binary operations defined for lattices.
a) Join, meet
b) Addition, subtraction
c) Union, intersection
d) Multiplication, modulo division
View Answer

Answer: a
Explanation: Join and meet are the binary operations reserved for lattices. The join of two
elements is their least upper bound. It is denoted by V, not to be confused with
disjunction. The meet of two elements is their greatest lower bound. It is denoted by ∧
and not to be confused with a conjunction.
5. A has a greatest element and a least element which satisfy 0<=a<=1 for
every a in the lattice(say, L).
a) semilattice
b) join semilattice
c) meet semilattice
d) bounded lattice
View Answer

Answer: d
Explanation: A lattice that has additionally a supremum element and an infimum element
which satisfy 0<=a<=1, for every an in the lattice is called a bounded lattice. A partially
ordered set is a bounded lattice if and only if every finite set (including the empty set) of
elements has a join and a meet.
6. The graph given below is an example of

discrete-mathematics-questions-answers-lattices-q6
a) non-lattice poset
b) semilattice
c) partial lattice
d) bounded lattice
View Answer

Answer: a
Explanation: The graph is an example of non-lattice poset where b and c have common
upper bounds d, e and f but none of them is the least upper bound.
7. A sublattice(say, S) of a lattice(say, L) is a convex sublattice of L if
a) x>=z, where x in S implies z in S, for every element x, y in L
b) x=y and y<=z, where x, y in S implies z in S, for every element x, y, z in L
c) x<=y<=z, where x, y in S implies z in S, for every element x, y, z in L
d) x=y and y>=z, where x, y in S implies z in S, for every element x, y, z in
L View Answer

Answer: c
Explanation: A sublattice S of a lattice L is a convex sublattice of L, if x ≤ z ≤ y and x, y
in S implies that z belongs to S, for all elements x, y, z in L.
8. The graph is the smallest non-modular lattice N5. A lattice is if and only if it
does not have a isomorphic to N5.

discrete-mathematics-questions-answers-lattices-q8
a) non-modular, complete lattice
b) moduler, semilattice
c) non-modular, sublattice
d) modular, sublattice
View Answer

Answer: d
Explanation: A lattice (L, ∨, ∧) is modular if for all elements a, b, c of L, the following
identity holds->modular identity: (a ∧ c) ∨ (b ∧ c) = [(a ∧ c) ∨ b] ∧ c. This condition
is equivalent to the following axiom -> modular law: a ≤ c implies a ∨ (b ∧ c)
= (a ∨ b) ∧ c. A lattice is modular if and only if it does not have a sublattice isomorphic
to N5.
9. Every poset that is a complete semilattice must always be a
a) sublattice
b) complete lattice
c) free lattice
d) partial lattice
View Answer

Answer: b
Explanation: A poset is called a complete lattice if all its subsets have both a join and a
meet. Every complete lattice is a bounded lattice. Every poset that is a complete
semilattice must always be a complete lattice.
10. A free semilattice has the property.
a) intersection
b) commutative and associative
c) identity
d) universal
View Answer

Answer: d
Explanation: Any set X may be used to generate the free semilattice FX. The free
semilattice is defined to consist of all of the finite subsets of X with the semilattice
operation given by ordinary set union; the free semilattice has the universal property.

1. The maximum number of edges in a bipartite graph on 14 vertices is


a) 56
b) 14
c) 49
d) 87
View Answer

Answer: c
Explanation: Maximum number of edges occur in a complete bipartite graph when every
vertex has an edge to every opposite vertex in the graph. Number of edges in a complete
bipartite graph is a*b, where a and b are no. of vertices on each side. This quantity is
maximum when a = b i.e. when there are 7 vertices on each side. So answer is 7 * 7 = 49.
2. In a the degree of each and every vertex is equal.
a) regular graph
b) point graph
c) star graph
d) euler graph
View Answer

Answer: c
Explanation: A regular graph has the same degree in each of its vertices. In a regular
bipartite graph, if the common degree of each vertices is 1, the two parts are of the same
size.
3. The time complexity to test whether a graph is bipartite or not is said to be
using depth first search.
a) O(n3)
b) linear time
c) O(1)
d) O(nlogn)
View Answer

Answer: b
Explanation: It is possible to test whether a graph is bipartite, and to return either a
two-coloring (if it is bipartite) or an odd cycle (if it is not) in linear time i.e, O(n) using
depth first search. In case of the intersection of n line segments or other simple shapes in
the Euclidean graph, it is possible to test whether the graph is bipartite and it will
return either a two-coloring or an odd cycle in time O(nlogn), even though the graph
itself has up to O(n2) edges.
4. The partition V = V1 ∪ V2 in a bipartite graph G1 is called
a) bipartition of G1
b) 2-vertex set of G1
c) sub bipartite graphs
d) disjoint vertex set
View Answer

Answer: b
Explanation: A graph G1(V, E) is called bipartite if its vertex set V(G) can be
decomposed into two non-empty disjoint subsets V1(G1) and V2(G1) in such a way that
each edge e ∈ E(G) has its one end joint in V1(G1) and other endpoint in V2(G1). The
partition V = V1 ∪ V2 in a bipartite graph G1 is called bipartition of G1.
5. What is the maximum number of edges in a bipartite graph on 14 vertices?
a) 78
b) 15
c) 214
d) 49
View Answer

Answer: d
Explanation: By definition, the maximum possible number of edges in a bipartite graph
on ‘n’ vertices = (1/4) x n2.
Substituting n = 14, we get maximum number of edges in a bipartite graph on 14
vertices,= (1/4) x (14)2
= (1/4) x 14 x 14
= 49
∴ Maximum number of edges in a bipartite graph on 14 vertices = 49.
6. In a complete bipartite graph, the intersection of two sub graphs is
a) 1
b) null
c) 210
d) 412
View Answer

Answer: b
Explanation: In a complete Bipartite graph, there must exist a partition say, V(G)=X∪Y
and X∩Y=∅ , that means all edges share a vertex from both set X and Y.
7. Bipartite graphs are used in
a) modern coding theory
b) colouring graphs
c) neural networks
d) chemical bonds
View Answer
Answer: a
Explanation: All types of cyclic graphs are examples of cyclic graphs. A cyclic graph is
considered bipartite if all the cycles involved are of even length. Bipartite graphs are
widely used in modern coding theory apart from being used in modeling relationships.
8. All closed walks are of length in a bipartite graph.
a) infinite
b) even
c) odd
d) odd prime
View Answer

Answer: b
Explanation: In a bipartite graph G all closed walks must be of even length as well as all
cycles in G are of even length. Then only the graph is considered a bipartite graph.
9. Every complete bipartite graph must not be

a) planar graph
b) line graph
c) complete graph
d) subgraph
View Answer

Answer: c
Explanation: The below bipartite graph is not a complete graph as there is no edge
between A-B, B-C, C-D, C-Q, P-Q, Q-R, Q-D and so it is not a complete graph.
discrete-mathematics-questions-answers-bipartite-graphs-q9
10. The spectrum of a graph is if and only if it is graph.
a) symmetry, bipartite
b) transitive, bipartite
c) cyclic, Euler
d) reflexive, planar
Answer: a
Explanation: A graph is bipartite if and only if it does not contain an odd cycle.
The spectrum of a graph is symmetric if and only if it is a bipartite graph. These
are the characteristics of the graph

1. In a 7-node directed cyclic graph, the number of Hamiltonian cycle is to be

a) 728
b) 450
c) 360
d) 260
View Answer

Answer: c
Explanation: A Hamiltonian cycle in a connected graph G is defined as a closed path
that traverses every vertex of G exactly once except the starting vertex, at which the
path also terminates. In an n-complete graph, there are (n-1)!/2 hamiltonian cycles and
so the answer is 360.
2. If each and every vertex in G has degree at most 23 then G can have a
vertex colouring of
a) 24
b) 23
c) 176
d) 54
View Answer

Answer: a
Explanation: A vertex colouring of a graph G = (V’,E’) with m colours is a mapping
f:V’ -> {1,…,m} such that f(u)!=f(v) for every (u,v) belongs to E’. Since in worst case
the graph can be complete, d+1 colours are necessary for graph containing vertices
with degree at most ‘d’. So, the required answer is 24.
3. Triangle free graphs have the property of clique number is
a) less than 2
b) equal to 2
c) greater than 3
d) more than 10
View Answer

Answer: d
Explanation: In an undirected triangle-free graph no three vertices can form a triangle
of edges. It can be described as graphs with clique number less than 2 and the graphs
with girth greater than 4.
4. Berge graph is similar to due to strong perfect graph theorem.
a) line graph
b) perfect graph
c) bar graph
d) triangle free graph
View Answer

Answer: b
Explanation: In a perfect graph, the chromatic number of each and every induced
subgraph is equal to the size of the largest clique of that subgraph. These perfect
graphs are same as Berge graphs due to strong perfect graph theorem.
5. Let D be a simple graph on 10 vertices such that there is a vertex of degree
1, a vertex of degree 2, a vertex of degree 3, a vertex of degree 4, a vertex of
degree 5, a vertex of degree 6, a vertex of degree 7, a vertex of degree 8 and a
vertex of degree 9. What can be the degree of the last vertex?
a) 4
b) 0
c) 2
d) 5
View Answer

Answer: c
Explanation: We know that sum of degrees of all vertices = 2X no of edges. Say
number of edges is E. Degree of last vertex is x, 1+2+3+4+5+6+7++8+9+x = 2XE
=>45+x = 2XE
Now putting options we get answer 0 or 5
But one vertex of degree 9 means it connected to all other vertexes. So, the degree
must be 5.
6. A is a graph which has the same number of edges as its
complement must have number of vertices congruent to 4m or 4m modulo 4(for
integral values of number of edges).
a) Subgraph
b) Hamiltonian graph
c) Euler graph
d) Self complementary graph
View Answer

Answer: d
Explanation: It is the definition of self complementary graph. It is a graph that is
isomorphic to its complement.
7. In a the vertex set and the edge set are finite sets.
a) finite graph
b) bipartite graph
c) infinite graph
d) connected graph
View Answer
Answer: b
Explanation: In graph theory, most common graphs are considered to be finite otherwise
it is an infinite graph. Now, a finite graph is a graph in which the vertex set and the edge
set are described as the finite sets.
8. If G is the forest with 54 vertices and 17 connected components, G has
total number of edges.
a) 38
b) 37
c) 17/54
d) 17/53
View Answer

Answer: b
Explanation: Here we are given a forest with 54 vertices and 17 components. A
component is itself a tree and since there are 17 components means that every
component has a root, therefore we have 17 roots. Each new vertex of the forest
contributes to a single edge to a forest. So for remaining 54-17 = 37 vertices we can
have m-n=37 edges. Hence, answer is 37.
9. The number of edges in a regular graph of degree 46 and 8 vertices is

a) 347
b) 230
c) 184
d) 186
View Answer

Answer: c
Explanation: In a complete graph which is (n-1) regular (where n is the number of
vertices) has edges n*(n-1)/2. In the graph n vertices are adjacent to n-1 vertices and
an edge contributes two degree so dividing by 2. Hence, in a d regular graph number
of edges will be n*d/2 = 46*8/2 = 184.
10. An undirected graph G has bit strings of length 100 in its vertices and
there is an edge between vertex u and vertex v if and only if u and v differ in
exactly one bit position. Determine the ratio of the chromatic number of G to
the diameter of G?
a) 1/2101
b) 1/50
c) 1/100
d) 1/20
View Answer

Answer: b
Explanation: For the given condition we can simply design a K-Map and mark an
edge between every two adjacent cells in K-map. Hence, that will give us a
Bipartite graph and chromatic number for this = 2. Hence the ratio is 2/n=2/100=1/50
and the given graph is actually a hypercube graph

1. A bridge can not be a part of


a) a simple cycle
b) a tree
c) a clique with size ≥ 3 whose every edge is a bridge
d) a graph which contains cycles
View Answer

Answer: a
Explanation: In a connected graph, a bridge is an edge whose removal disconnects the
graph. In a cycle if we remove an edge, it will still be connected. So, bridge cannot be
part of a cycle. A clique is any complete subgraph of a graph.
2. Any subset of edges that connects all the vertices and has minimum total
weight, if all the edge weights of an undirected graph are positive is called

a) subgraph
b) tree
c) hamiltonian cycle
d) grid
View Answer

Answer: b
Explanation: If all the edge weights of an undirected graph are positive, any subset of
edges that connects all the vertices and has minimum total weight is termed as a tree.
In this case, we need to have a minimum spanning tree need to be exact.
3. G is a simple undirected graph and some vertices of G are of odd degree.
Add a node n to G and make it adjacent to each odd degree vertex of G.
The resultant graph is
a) Complete bipartite graph
b) Hamiltonian cycle
c) Regular graph
d) Euler graph
View Answer

Answer: d
Explanation: In any simple undirected graph, total degree of all vertices is even (since
each edge contributes 2 degrees). So number of vertices having odd degrees must be
even, otherwise, their sum would have been odd, making total degree also odd. Now
single vertex n is connected to all these even number of vertices (which have odd
degrees). So, degree of n is also even. Moreover, now degree of all vertices which are
connected to v is increased by 1, hence earlier
vertices which had odd degree now have even degree. So now, all vertices in the graph
have even degree, which is necessary and sufficient condition for euler graph.
4. Let G be a directed graph whose vertex set is the set of numbers from 1 to
50. There is an edge from a vertex i to a vertex j if and only if either j = i + 1 or j
= 3i. Calculate the minimum number of edges in a path in G from vertex 1 to
vertex 50.
a) 98
b) 13
c) 6
d) 34
View Answer

Answer: c
Explanation: Edge set consists of edges from i to j using either 1) j = i+1 OR 2) j=3i.
The trick to solving this question is to think in a reverse way. Instead of finding a path
from 1 to 50, try to find a path from 100 to 1. The edge sequence with the minimum
number of edges is 1 – 3 – 9 – 10 – 11 – 33 which consists of 6 edges.
5. What is the number of vertices in an undirected connected graph with 39
edges, 7 vertices of degree 2, 2 vertices of degree 5 and remaining of
degree 6?
a) 11
b) 14
c) 18
d) 19
View Answer

Answer: c
Explanation: We know that, sum of degree of all the vertices = 2 * number of edges
2*7 + 5*2 + 6*x = 39*2
x=9
Number of vertices = 7 + 2 + 9 = 18.
6. is the maximum number of edges in an acyclic undirected graph with
k vertices.
a) k-1
b) k2
c) 2k+3
d) k3+4
View Answer

Answer: a
Explanation: This is possible with spanning trees since, a spanning tree with k nodes
has k – 1 edges.
7. The minimum number of edges in a connected cyclic graph on n vertices is

a) n – 1
b) n
c) 2n+3
d) n+1
View Answer

Answer: b
Explanation: For making a cyclic graph, the minimum number of edges have to be
equal to the number of vertices. SO, the answer should be n minimum edges.
8. The maximum number of edges in a 8-node undirected graph without self
loops is
a) 45
b) 61
c) 28
d) 17
View Answer

Answer: c
Explanation: In a graph of n vertices we can draw an edge from a vertex to n-1 vertex
we will do it for n vertices and so total number of edges is n*(n-1). Now each edge is
counted twice so the required maximum number of edges is
n*(n-1)/2. Hence, 8*(8-1)/2 = 28 edges.
9. Let G be an arbitrary graph with v nodes and k components. If a vertex is
removed from G, the number of components in the resultant graph must
necessarily lie down between and
a) n-1 and n+1
b) v and k
c) k+1 and v-k
d) k-1 and v-1
View Answer

Answer: d
Explanation: If a vertex is removed from the graph, lower bound: number of
components decreased by one = k-1 (remove an isolated vertex which was a
component) and upper bound: number of components = v-1 (consider a vertex
connected to all other vertices in a component as in a star and all other vertices outside
this component being isolated. Now, removing the considered vertex makes all other
(v-1) vertices isolated making (v-1) components.
10. The 2n vertices of a graph G corresponds to all subsets of a set of size n,
for n>=4. Two vertices of G are adjacent if and only if the corresponding sets
intersect in exactly two elements.
The number of connected components in G can be
a) n+2
b) 3n/2
c) n2
d) 2n
View Answer

Answer: b
Explanation: n+1(subsets of size < 2 are all disconnected) (subsets of size >= 2 are all
connected)+1(subset of size >= 2 are all connected)=n+2 is the number of connected
components in G.

1. A graph which has the same number of edges as its complement must
have number of vertices congruent to or modulo 4(for integral
values of number of edges).
a) 6k, 6k-1
b) 4k, 4k+1
c) k, k+2
d) 2k+1, k
View Answer

Answer: c
Explanation: By using invariant of isomorphism and property of edges of graph and its
complement, we have: a) number of edges of isomorphic graphs must be the same.
b) number of edge of a graph + number of edges of complementary graph = Number of
edges in Kn(complete graph), where n is the number of vertices in each of the 2 graphs
which will be the same. So we know number of edges in Kn = n(n-1)/2. So number of
edges of each of the above 2 graph(a graph and its complement) = n(n-1)/4. So this
means the number of vertices in each of the 2 graphs should be of the form “4x” or
“4x+1” for integral value of number of edges which is necessary. Hence the required
answer is 4x or 4x+1 so that on doing modulo we get 0 which is the definition of
congruence.
2. Every Isomorphic graph must have representation.
a) cyclic
b) adjacency list
c) tree
d) adjacency matrix
View Answer

Answer: d
Explanation: A graph can exist in different forms having the same number of
vertices, edges and also the same edge connectivity, such graphs are called
isomorphic graphs. Two graphs G1 and G2 are said to be isomorphic if −> 1) their
number of components (vertices and edges) are same and 2) their edge
connectivity is retained. Isomorphic graphs must have adjacency matrix
representation.
3. A cycle on n vertices is isomorphic to its complement. What is the value of n?
a) 5
b) 32
c) 17
d) 8
View Answer

Answer: a
Explanation: A cycle with n vertices has n edges. Number of edges in cycle = n and
number of edges in its complement = (n*(n−1)/2) – n. To be isomorphism, both graphs
should have equal number of edges. This gives, (n*(n-1)/2) – n = n
⇒ n=5
4. How many perfect matchings are there in a complete graph of 10 vertices?
a) 60
b) 945
c) 756
d) 127
View Answer

Answer: b
Explanation: Perfect matching is a set of edges such that each vertex appears only once
and all vertices appear at least once (exactly one appearance). So for n vertices perfect
matching will have n/2 edges and there won’t be any perfect matching if n is odd. For
n=10, we can choose the first edge in 10C2 = 45 ways, second in 8C2=28 ways, third in
6C2=15 ways and so on. So, the total number of ways 45*28*15*6*1=113400. But
perfect matching being a set, order of elements is not important and the permutations 5!
of the 5 edges are same only. So, total number of perfect matching is 113400/5! = 945.
5. A graph G has the degree of each vertex is ≥ 3 say, deg(V) ≥ 3 ∀ V ∈ G
such that 3|V| ≤ 2|E| and 3|R| ≤ 2|E|, then the graph is said to be (R
denotes region in the graph)
a) Planner graph
b) Polyhedral graph
c) Homomorphic graph
d) Isomorphic graph
View Answer

Answer: b
Explanation: A simple connected planar graph is called a polyhedral graph if the degree
of each vertex is(V) ≥ 3 such that deg(V) ≥ 3 ∀ V ∈ G and two conditions must satisfy
i) 3|V| ≤ 2|E| and ii) 3|R| ≤ 2|E|.
6. A complete n-node graph Kn is planar if and only if
a) n ≥ 6
b) n2 = n + 1
c) n ≤ 4
d) n + 3
View Answer

Answer: c
Explanation: Any graph with 4 or less vertices is planar, any graph with 8 or less edges
is planar and a complete n-node graph Kn is planar if and only if n ≤ 4.
7. A graph is if and only if it does not contain a subgraph
homeomorphic to k5 or k3,3.
a) bipartite graph
b) planar graph
c) line graph
d) euler subgraph
View Answer

Answer: b
Explanation: A graph is known as planar graph if and only if it does not contain a
subgraph homeomorphic to k5 or k3,3.
8. An isomorphism of graphs G and H is a bijection f the vertex sets of G and
H. Such that any two vertices u and v of G are adjacent in G if and only if

a) f(u) and f(v) are contained in G but not contained in H


b) f(u) and f(v) are adjacent in H
c) f(u * v) = f(u) + f(v)
d) f(u) = f(u)2 + f(v)2
View Answer

Answer: b
Explanation: Two graphs G and H are said to be isomorphic to each other if there exist
a one to one correspondence, say f between the vertex sets V(G) and V(H) and a one
to one correspondence g between the edge sets E(G) and E(H) with the following
conditions:-
(i) for every vertex u in G, there exists a vertex u’ in H such that u’=f(u) and
vice versa.
(ii) for every edge uv in G, g(uv)=f(u)*f(v)=u’v’ is H.
9. What is the grade of a planar graph consisting of 8 vertices and 15 edges?
a) 30
b) 15
c) 45
d) 106
View Answer

Answer: a
Explanation: If G is a planar graph with n vertices and m edges then r(G) = 2m
i.e. the grade or rank of G is equal to the twofold of the number of edges in G. So, the
rank of the graph is 2*15=30 having 8 vertices and 15 edges.
10. A is a graph with no homomorphism to any proper subgraph.
a) poset
b) core
c) walk
d) trail
View Answer

Answer: b
Explanation: A core can be defined as a graph that does not retract to any proper
subgraph. Every graph G is homomorphically equivalent to a unique core called
the core of G

1. Which algorithm efficiently calculates the single source shortest paths in


a Directed Acyclic Graph?
a) topological sort
b) hash table
c) binary search
d) radix sort
View Answer

Answer: a
Explanation: For Directed Acyclic graph, single source shortest distances can be
calculated in O(V+E) time. For that purpose Topological Sorting can be used.
Topological Sorting of any graph represents a linear ordering of the graph.
2. The of a graph G consists of all vertices and edges of G.
a) edge graph
b) line graph
c) path complement graph
d) eulerian circuit
View Answer

Answer: d
Explanation: we know that he Eulerian circuit in a graph G is a circuit that includes all
vertices and edges of G. A graph that can have Eulerian circuit, also can have a Eulerian
graph.
3. A in a graph G is a circuit which consists of every vertex (except
first/last vertex) of G exactly once.
a) Euler path
b) Hamiltonian path
c) Planar graph
d) Path complement graph
View Answer

Answer: b
Explanation: The Eulerian path in a graph say, G is a walk from one vertex to another,
that can pass through all vertices of G as well as traverses exactly once every edge of
G. Therefore, an Eulerian path can not be a circuit. A Hamiltonian path is a walk that
contains every vertex of the graph exactly once. Hence, a Hamiltonian path is not a
circuit.
4. A walk has Closed property if
a) v0=vk
b) v0>=vk
c) v < 0
d) vk > 1
View Answer

Answer: a
Explanation: A walk in a graph is said to be closed if the starting vertex is the same as
the ending vertex, that is v0=vk, it is described as Open otherwise.
5. A trail in a graph can be described as
a) a walk without repeated edges
b) a cycle with repeated edges
c) a walk with repeated edges
d) a line graph with one or more
vertices View Answer

Answer: a
Explanation: Suppose in a graph G a trail could be defined as a walk with no
repeated edges. Suppose a walk can be defined as efgh. There are no repeated edges
so this walk is a trail.
6. Let a graph can be denoted as ncfkedn a kind of
a) cycle graph
b) line graph
c) hamiltonian graph
d) path graph
View Answer

Answer: a
Explanation: In the graph ncfkedn, no edges are repeated in the walk, which makes it a
trail and then start and end vertex n is same making it a cycle graph.
7. Determine the edge count of a path complement graph with 14 vertices.
a) 502
b) 345
c) 78
d) 69
View Answer

Answer: c
Explanation: Let, an n-path complement graph Pn’ is the graph complement of the
path graph Pn. Since Pn is self-complementary, P4’ is isomorphic to P4.
Now, Pn’ has an edge count = 1⁄2(n-2)(n-1). So, the required edge count is=78.
8. The sum of an n-node graph and its complement graph produces a graph
called
a) complete graph
b) bipartite graph
c) star graph
d) path-complement graph
View Answer

Answer: a
Explanation: Suppose, the complement G’ of a graph G is known as
edge-complement graph which consists of with the same vertex set but whose edge set
contains the edges not present in G. The graph sum G+G’ on an
n-node graph G is called the complete graph say, Kn.
9. In a directed weighted graph, if the weight of every edge is decreased by 10
units, does any change occur to the shortest path in the modified graph?
a) 209
b) 65
c) 57
d) 43
View Answer

Answer: c
Explanation: The shortest path will change in the modified graph. Suppose that the
shortest path is of weight 21 and has 7 edges and there is another path with 4 edges and
total weight 17. Now, the weight of the first shortest path is increased by 7*10 and
becomes 21 + 70 and the weight of the second path is increased by 4*10 and becomes
17 + 40. So the shortest path changes to the other path with weight as 57.
10. Let G(V, E) be a directed graph where every edge has weight as either 1, 2
or 5, what is the algorithm used for the shortest path from a given source
vertex to a given destination vertex to get the time complexity of O(V+E)?
a) BFS
b) DFS
c) Binary search
d) Radix sort
View Answer

Answer: a
Explanation: In BFS due to the least number of edges between two vertices and so if all
the edges in a graph are of same weight, then to find the shortest path BFS can be used
for efficiency. So we have to split all edges of weight 5 into two edges of weight 2 each
and one edge of weight 1. In the worst case, all edges
are of weight 1. To split all edges, O(E) operations can be done and so the time complexity
becomes which is equal to O(V+E)

1. The chromatic number of a graph is the property of


a) graph coloring
b) graph ordering
c) group ordering
d) group coloring
View Answer

Answer: b
Explanation: A graph coloring is an assignment of labels to the vertices of a graph
such that no two adjacent vertices share the same labels is called the colors of the
graph. Now, the chromatic number of any graph is the minimal number of colors
for which such an assignment is possible.
2. If a graph G is k-colorable and k<n, for any integer n then it is
a) n-colorable
b) n2 nodes
c) (k+n)-colorable
d) (k3+n3+1) nodes
View Answer

Answer: a
Explanation: The chromatic number of a graph is the minimal number of colors for
which a graph coloring is possible. A graph G is termed as k-colorable if there exists a
graph coloring on G with k colors. If a graph is k-colorable, then it is n-colorable for
any n>k.
3. If Cn is the nth cyclic graph, where n>3 and n is odd. Determine the value of
X(Cn).
a) 32572
b) 16631
c) 3
d) 310
View Answer

Answer: c
Explanation: Here n is odd and X(Cn)! = 2. Since there are two adjacent edges in Cn.
Now, a graph coloring for Cn exists where vertices are colored red and blue
alternatively and another edge is with a different colour say orange, then the value of
X(Cn) becomes 3.
4. Determine the density of a planar graph with 34 edges and 13 nodes.
a) 22/21
b) 12/23
c) 328
d) 576
View Answer

Answer: a
Explanation: The density of a planar graph or network is described as the ratio of the
number of edges(E) to the number of possible edges in a network with(N) nodes. So, D
= E − N + 1/ 2 N − 5. Hence, the required answer is:
D=(34-13+1)/(2*13-5) = 22/21. A completely sparse planar graph has density 0 and a
completely dense planar graph has degree 1.
5. If the number of vertices of a chromatic polynomial PG is 56, what is the
degree of PG?
a) 344
b) 73
c) 265
d) 56
View Answer

Answer: d
Explanation: The chromatic polynomial PG of a graph G is a polynomial in which
every natural number k returns the number PG(k) of k-colorings of G. Since, the
degree of PG is equal to the number of vertices of G, the required answer is 56.
6. For a connected planar simple graph G=(V, E) with e=|E|=16 and v=|V|=9,
then find the number of regions that are created when drawing a planar
representation of the graph?
a) 321
b) 9
c) 1024
d) 596
View Answer

Answer: b
Explanation: We know that the number of regions in a planar representation of the
graph is r=e-v+2, then the required answer is r=16-9+2=9.
7. For a connected planar simple graph G=(V, E) with e=|E|=16 and v=|V|=9,
then find the number of regions that are created when drawing a planar
representation of the graph?
a) 321
b) 9
c) 1024
d) 596
View Answer

Answer: b
Explanation: Note that K3,3 and K5 are the “smallest” non-planar graphs
because in that every non-planar graph contains them. According to
Kuratowski’s theorem, a graph is defined as non-planar if and only if it contains a
subgraph homomorphic to K3,3 or K5.
8. Suppose G be a connected planar graph of order n≥5 and size m. If the
length of the smallest cycle in G is 5, then which of the following is true?
a) (m+n)4>=mn
b) m≤5/3(n−2)
c) (m2+n)/3
d) n>=(6/5)(n+1)
View Answer

Answer: b
Explanation: Because G is connected and planar, Euler’s theorem is bound to be
involved. Let f denote the number of faces so that n−m+f=2. Because the length of the
smallest cycle in G is 5, every face has at least 5 edges adjacent to it. This means
2m≥5f because every edge is adjacent to two faces. Plugging this in yields
2=n−m+f≤n−m+2/5m=n−3/5m, and hence m≤5/3(n−2).
9. What is the number of edges of the greatest planar subgraph of K3,2 where
m,n≤3?
a) 18
b) 6
c) 128
d) 702
View Answer

Answer: b
Explanation: The plane graph with an edge at most 6+2(m−3) is the greatest planar
graph. So, in this case the number of edges is 6.
10. A non-planar graph can have
a) complete graph
b) subgraph
c) line graph
d) bar graph
View Answer

Answer: b
Explanation: A non-planar graph can have removed edges and vertices so that it
contains subgraphs. However, non-planar graphs cannot be drawn in a plane and so no
edge of the graph can cross it.

1. A direct product of a group G possess which of the following characteristics?


a) a multiplication of subgroups of G
b) a factorization via subgroups of G
c) a superset of subgroups of G
d) a maximal power set of subgroups
View Answer
Answer: b
Explanation: A direct product of a group G is a factorization via subgroups of G when
the intersection is nontrivial, say X and Y, such that G = XY, X intersect Y
= 1, and [X, Y]=1 and X, Y are normal in G.
2. In invariant algebra, some generators of group G1 that goes either into itself
or zero under with any other element of the algebra.
a) commutation
b) permutation
c) combination
d) lattice
View Answer

Answer: a
Explanation: Some generators of group G1 in group theory which goes either into
itself or zero under commutation with any other element of the whole algebra is
called invariant subalgebra.
3. Which of the following can be embedded in an algebraically closed group?
a) infinite group
b) stargraph
c) a countable group
d) a semilattice
View Answer

Answer: c
Explanation: We know that any countable group can always be embedded in an algebraically
closed group.
4. Which of the following is the set of m×m invertible matrices?
a) a permutation group of degree m2
b) a general linear group of degree m
c) a sublattice group of degree m
d) a isomorphic graph of m nodes
View Answer

Answer: b
Explanation: The general linear group of degree m is the set of m×m invertible
matrices, consists of a general linear group of degree m having the ordinary matrix
multiplication operation.
5. If any group is a manifold what is the dimension of that group?
a) same as manifold
b) same as vector space
c) infinite
d) finite
View Answer

Answer: a
Explanation: If a group is a (topological) manifold, then the dimension of a group will
be the dimension of this manifold. A linear representation F of a group G1 on a vector
space V’ has the dimension of V’.
6. A Latin square graph is a representation of a
a) quasi group
b) homomorphic group
c) semigroup
d) subgroup
View Answer

Answer: a
Explanation: We know that any group is a representation of a graph. Now, a Quasi
Group can be represented by a Latin Square matrix or by a Latin Square graph.
7. There exists between group homology and group cohomology of a
finite group.
a) homomorphism
b) isomorphism
c) automorphism
d) semilattice structure
View Answer

Answer: a
Explanation: We know that there exists an isomorphism between group homology and
group cohomology of finite group. Let S’ denote the set of all integers, and let G’ be a
finite cyclic Group and for every S then G’-module N, we have S’S’n(G’, A) is
isomorphic to S’n+1(G’, A).
8. In basic ring theory, any ring R1 may be embedded in its own
a) semilattice
b) endomorphism ring
c) homomorphic ring
d) subgroup
View Answer

Answer: b
Explanation: We know that in basic ring theory, any ring R with its identity can be
embedded in its own endomorphism ring and this is one of the most important
characterization of rings. The endomorphism ring can contain a copy of its ring.
9. In Modern particle physics there must exist
a) group theory
b) graph theory
c) lattice structure
d) invariant semigroup
View Answer
Answer: a
Explanation: Modern particle physics exists with group theory. Group theory can
predict the existence of many elementary particles. Depending on different symmetries,
the structure and behaviour of molecules and crystals can be defined.
10. For any graph say G, Cayley graph is
a) canonial
b) not canonical
c) isomorphic
d) homomorphic
View Answer

Answer: b
Explanation: A different Cayley graph will be given for each choice of a generating
set. Hence, the Cayley graph is not canonical.

some extra questions

1 . Which of the following sets are null sets ?

A. { } B. ø
C. Both (a) and (b) D.
{0}
Answer & Explanation
Answer: C

Explanation:

Both (a) and (b)

View Answer Workspace Report


2 . Let R be a non-empty relation on a
collection of sets defined by ARB if
and only if A ∩ B = Ø Then (pick the
TRUE statement)
A. R is relexive and transitive
B. R is an equivalence relation
C. R is symmetric and not transitive
D. R is not relexive and not
symmetric Answer & Explanation
Answer: C

Explanation:

R is symmetric and not transitive


View Answer Workspace Report
3 . The binary relation S = Φ (empty set)

on set A = {1, 2,3} is


A. transitive and relexive
B. symmetric and relexive
C. transitive and symmetric
D. neither reflexive nor symmetric
Answer & Explanation
Answer: C

Explanation:
transitive and symmetric
View Answer Workspace Report
4 . Number of subsets of a set of order

three is
A. 2 B. 4
C. 6 D. 8
Answer & Explanation
Answer: D

Explanation:

8
View Answer Workspace Report
5 . "n/m" means that n is a factor of m,

then the relation T is


A. relexive, transitive and not symmetric
B. relexive, transitive and symmetric
C. transitive and symmetric
D. relexive and symmetric
Answer & Explanation
Answer: A

Explanation:
relexive, transitive and not symmetric View
Answer Workspace Report

6 . If R be a symmetric and transitvie relation on a set A, then


A. R is not reflexive and hence not an equivalence relation
B. R is reflexive and hence an equivalence relation
C. R is reflexive and hence a partial order
D. None of these
Answer & Explanation
Answer: D

Explanation:

None of these
View Answer Workspace Report
7 . Let P(S) denote the power set of set S. Which of the following is
always TRUE ?
A. S ∉ P(S)
B. P(P(S)) = P(S)
C. P(S) ∩ S = P (S)
D. P(S) ∩ P(P(S)) = [ φ ]
Answer & Explanation
Answer: D

Explanation:

P(S) ∩ P(P(S)) = [ φ ]
View Answer Workspace Report
8 . The number of elements in the Power set P(S) of the set S = [ [ Φ] , 1, [ 2, 3
]] is
A. 2 B. 4
C. 6 D. 8
Answer & Explanation
Answer: D

Explanation:

8
View Answer Workspace Report
9 . If A and B are sets and A∪ B= A ∩ B, then
A. A = B B. A=Φ
C. B = Φ D. none of these
Answer & Explanation
Answer: A

Explanation:

A=B
View Answer Workspace Report
10 . Let S be an infinite set and S1, S2, S3, ..., Sn be sets such that S1 ∪S2
∪S3∪......Sn = S then
A. atleast one of the sets Si is a finite set
B. atleast one of the sets Si is an ininite set
C. not more than one of the set Si can be inite
D. none of these
Answer & Explanation
Answer: B

Explanation:

atleast one of the sets Si is an ininite set

11 . If X and Y are two sets, then X ∩ (Y ∪ X) C equals


A. Ψ B. X
C. Y D. None of these
Answer & Explanation
Answer: A

Explanation:

Ψ
View Answer Workspace Report
12 . If f : X -> Y and a, b ⊆ X, then f (a ∩ b) is equal to
A. f(a) - f(b)
B. f(a) ∩ f(b)
C. f(b) - f(a)
D. a proper subset of f(a) ∩ f(b)
Answer & Explanation
Answer: D

Explanation:

a proper subset of f(a) ∩ f(b) View


Answer Workspace Report
13 . Let f : R → R be defined by f(x)= {x+2 (x ≤ -1) { x2 (-1 ≤ x ≤1) {2 - x (x ≥ 1)
Then value of f (-1.75) + f (0.5) + f (1.5) is
A. 0 B. 1
C. 2 D. None of these
Answer & Explanation
Answer: B

Explanation:

1
View Answer Workspace Report
14 . A relation R is defined on the set of positive integers as xRy if 2x + y ≤
5. The realation R is
A.
reflexive
B. transitive
C. symmetric
D. None of these
Answer & Explanation
Answer: B

Explanation:

transitive
View Answer Workspace Report
15 . Let R be na equivalence relation on the set {1,2,3,4,5,6} given by
{(1,1),(1,5),(2,2),(2,3),(2,6),(3,2),(3,3),(3,6),(4,4),(5,1),(5,5),(6,2),(6,6),(6,6)}.
The partition included by R is
A. {1,2,3,4,5,6} B. {{1,3,5,6},{2,4}}
C. {{1,2,3,4},{5,6}} D. {{1,5},{2,3,6},{4}}
Answer & Explanation
Answer: D

Explanation:

{{1,5},{2,3,6},{4}}

16 . Which of the following sets is a null set ? I. X = {x | x= 9, 2x = 4 } II. Y = {x


| x= 2x.x ≠ 0 } III. Z = { x | x-8 = 4 }
A. I and II only B. I, II and III
C. I and III only D. II and III only
Answer & Explanation
Answer: A
Explanation:

I and II only
View Answer Workspace Report
17 . A Relation R is defined on the set of integers as xRy if (x + y) is even.
Which of the following statements is TRUE?
A. R is an equivalence relation having three equivalence classes
B. R is an equivalence relation having two equivalence classes
C. R is an equivalence relation having one equivalence class
D. R is not an equivalence relation
Answer & Explanation
Answer: B

Explanation:

R is an equivalence relation having two equivalence classes View


Answer Workspace Report
18 . The number of elements in the power set of the set {{a, b}, c} is
A. 2 B. 4
C. 6 D. 8
Answer & Explanation
Answer: B

Explanation:

4
View Answer Workspace Report
19 . If R = ((1, 1), (3, 1), (2, 3), (4, 2)), then which of the following represents
R2, where R2 is R composite R?
A. ((1, 1), (2, 1), (4, 3), (3, 1))
B. ((1, 1), (3, 1), (2, 3), (4, 2))
C. 1(1, 3), (3, 3), (3, 4), (3, 2))
D. f(1, 1), (9, 1), (4, 9), (16, 4))
Answer & Explanation
Answer: A

Explanation:

((1, 1), (2, 1), (4, 3), (3, 1))


View Answer Workspace Report
20 . If f : R---->R defined by f(x) = x2 + 1, then values of f -1 (17) and f -1(-3) are
respectively
A. {4,-4},Ψ B. {Ψ},{3,-3}
C. {3,-3},{Ψ} D. {Ψ}, (4, - 4)
Answer & Explanation
Answer: A

Explanation:

{4,-4},Ψ

21 . If every element of a group G is its own inverse, then G is


A. abeian B. cyclic
C. finite D. infinite
Answer & Explanation
Answer: A

Explanation:

abeian
View Answer Workspace Report
22 . The universal relation A x A on A is
A. anti-symmetric
B. an equivalence relation
C. a partial ordering relation
D. not symmetric and not anti-symmetric
Answer & Explanation
Answer: B

Explanation:

an equivalence relation
View Answer Workspace Report
23 . Total number of diferent partitions of a set having four elements is
A. 5 B. 10
C. 15 D. 20
Answer & Explanation
Answer: C

Explanation:

15
View Answer Workspace Report
24 . A partition of {1, 2, 3, 4, 5} is the family
A. {(1, 2, 3),(5)}
B. {(1, 2,), (3, 4, 5)}
C. {φ(1, 2),(3, 4),(5)}
D. {(1, 2),(3, 4),(3, 5)}
Answer & Explanation
Answer: B

Explanation:

{(1, 2,), (3, 4, 5)}


View Answer Workspace Report
25 . Let s(w) denote the set of all the letters in w where w is an English word.
Let us denote set equality, subset and union relations by =, ⊂ and ∪
respectively. Which of the following is NOT true?
A. s(ten) ⊂ s(twenty)
B. s(stored) = s(sorted)
C. s(sixty) ⊂ (s(six) ∪ s(twenty)
D.
None of these Answer
& Explanation Answer:
D

Explanation:

None of these

26 . In a beauty contest, half the number of experts voted for Mr. A and two
thirds voted for Mr. B. 10 voted for both and 6 did not vote for either. How
many experts were there in all ?
A. 18 B. 24
C. 36 D. 44
Answer & Explanation
Answer: B

Explanation:

24
View Answer Workspace Report
27 . Let n(A) denotes the number of elements in set A. If n(A) =p and n(B) =
q, then how many ordered pairs (a, b) are there with a ∈ A and b ∈ B ?
A. p x q B. p + q
C. 2 pq D. 4 pq Answer
& Explanation Answer:
A

Explanation:

pxq
View Answer Workspace Report
28 . The set of all Equivalence classes of a set A of cardinality C
A. forms a partition of A
B. is of cardinality 2C
C. has the same cardinality as A
D. none of these
Answer & Explanation
Answer: A

Explanation:

forms a partition of A
View Answer Workspace Report
29 . Let Z denote the set of all integers. Define f : Z —> Z by f(x) = {x / 2 (x is
even) 0 (x is odd) then f is
A. one-one and onto
B.
one-one but not onto
C. onto but not one-one
D. neither one-one nor-onto
Answer & Explanation
Answer: C

Explanation:

onto but not one-one


View Answer Workspace Report
30 . Let R be a relation "(x -y) is divisible by m", where x, y, m are integers and
m > 1, then R is
A. partial order
B. equivalence relation
C. symmetric but not transitive
D. anti symmetric and not
transitive Answer & Explanation
Answer: B

Explanation:

equivalence relation

31 . A subset H of a group(G,*) is a group if


A. a,b ∈ H ⇒ a * b ∈ H
B. a ∈ H⇒ a-1 ∈ H
C.
a,b ∈ H ⇒ a * b-1 ∈ H
D. H contains the identity element
Answer & Explanation
Answer: C

Explanation:

a,b ∈ H ⇒ a * b-1 ∈ H
View Answer Workspace Report
32 . If A = {1, 2, 3} then relation S = {(1, 1), (2, 2)} is
A. symmetric only
B. anti-symmetric only
C. an equivalence relation
D. both symmetric and anti-symmetric
Answer & Explanation
Answer: D

Explanation:

both symmetric and anti-symmetric


View Answer Workspace Report
33 . Which of the following statements is true?
A. Empty relation φ is reflexive
B. Every equivalence relation is a partial-ordering relation.
C. Number of relations form A = {x, y, z} to B= {1, 2} is 64.
D. Properties of a relation being symmetric and being ant-symmetric are
negative of each other.
Answer & Explanation
Answer: C

Explanation:

Number of relations form A = {x, y, z} to B= {1, 2} is 64.


View Answer Workspace Report
34 . Let A = {1, 2,.....3 } Define ~ by x ~ y ⇔ x divides y. Then ~ is
A. symmetric
B.
an equivalence relation
C. a partial-ordering relation
D. relexive, but not a partial-ordering
Answer & Explanation
Answer: C

Explanation:
a partial-ordering relation
View Answer Workspace Report
35 . G(e, a, b, c} is an abelian group with 'e' as identity element. The order of
the other elements are
A. 2,2,4 B. 2,2,3
C. 2,3,4 D. 3,3,3
Answer & Explanation
Answer: B

Explanation:

2,2,3

36 . If f : A ---> B is a bijective function, then f -1 of f =


A. f
B. f -1
C. f o f -1
D. IA(Identity map of the set
A) Answer & Explanation
Answer: D

Explanation:

IA(Identity map of the set A) View


Answer Workspace Report
37 . The set of all real numbers under the usual multiplication operation is not
a group since
A. zero has no inverse
B. identity element does not exist
C. multiplication is not associative
D. multiplication is not a binary operation
Answer & Explanation
Answer: A

Explanation:

zero has no inverse


View Answer Workspace Report
38 . If (G, .) is a group such that (ab)- 1 = b-1 a-1, ∀ a, b ∈ G, then G is a/an
A. abelian group
B. non-abelian group
C. commutative semi group
D. None of these
Answer & Explanation
Answer: A

Explanation:

abelian group
View Answer Workspace Report
39 . If * is defined on R* as a * b = (ab/2) then identity element in the group (R*,
*) is
A. 1 B. 2
C. 1/2 D. 1/3
Answer & Explanation
Answer: B

Explanation:

2
View Answer Workspace Report
40 . If (G, .) is a group such that a2 = e, ∀ a ∈ G, then G is
A. semi group B. abelian group
C. non-abelian group D. none of these
Answer & Explanation
Answer: B

Explanation:

abelian group

41 . Some group (G, 0) is known to be abelian. Then which one of the


following is TRUE for G ?
A. g = g-1 for every g ∈ G
B. g = g² for every g ∈ G
C. (g o h)²= g²o h² for every g,h ∈ G
D. G is of finite order
Answer & Explanation
Answer: C

Explanation:

(g o h)² = g²o h² for every g,h ∈ G


View Answer Workspace Report
42 . If the binary operation * is deined on a set of ordered pairs of real numbers
as (a, b) * (c, d) = (ad + bc, bd) and is associative, then (1, 2) * (3, 5) * (3, 4)
equals
A. (7,11) B. (23,11)
C. (32,40) D. (74,40)
Answer & Explanation
Answer: D

Explanation:

(74,40)
View Answer Workspace Report
43 . If A = (1, 2, 3, 4). Let ~ = ((1, 2), (1, 3), (4, 2). Then ~ is
A. reflexive B. transitive
C. symmetric D. not anti-symmetric
Answer & Explanation
Answer: B

Explanation:

transitive
View Answer Workspace Report
44 . Which of the following statements is false ?
A. If R is relexive, then R ∩ R-1≠ φ
B. R ∩ R-1≠ φ =>R is anti-symmetric.
C. If R, R' are reflexive relations in A, then R - R' is reflexive
D. If R, R' are equivalence relations in a set A, then R ∩ R' is also an
equivalence relation in A.
Answer & Explanation
Answer: C

Explanation:

If R, R' are reflexive relations in A, then R - R' is reflexive View


Answer Workspace Report
45 . If R = {(1, 2),(2, 3),(3, 3)} be a relation defined on A= {1, 2, 3} then R . R( = R2) is
A. R itself
B. {(1, 2),(1, 3),(3, 3)}
C. {(1, 3),(2, 3),(3, 3)}
D. {(2, 1),(1, 3),(2, 3)}
Answer & Explanation
Answer: C

Explanation:
{(1, 3),(2, 3),(3, 3)}

46 . Every set is a of itself


A. Compliment B. Proper subset
C. Improper subset D. None of the above
Answer & Explanation
Answer: C

Explanation:

Improper subset
View Answer Workspace Report 47 .
Empty set is a ?
A. Infinite Set B. Invalid Set
C. Finite Set D. None of the above
Answer & Explanation
Answer: C

Explanation:

Finite Set
View Answer Workspace Report
48 . A’ will contain how many elements from the original set A
A. 0 B. 1
C. InfiniteD. All elements in A
Answer & Explanation
Answer: A

Explanation:

0
View Answer Workspace Report
49 . A — B will contain elements in ?
A. A not in B B. B not in A
C. Both A and B D. Neither A nor B
Answer & Explanation
Answer: A

Explanation:

A not in B
View Answer Workspace Report
50 . A set has n elements, then the, number of elements in its power set is ?
A. 2n B. mxn
C. m + n D. m-n
Answer & Explanation
Answer: B

Explanation:

mxn

51 . The intersection of sets A and B is expressed as ?


A. AxB B. AnB
C. A-B D. A/B Answer
& Explanation Answer:
B

Explanation:

AnB
View Answer Workspace Report
52 . If R = {(1,1),(2,3),(4,5)}, then domain of the function is ?
A. Range R = {2,3,4,5} B. Range R {1,1,4,5}
C. Range R = {I,3,5} D. Range R {1,2,5}
Answer & Explanation
Answer: C

Explanation:

Range R = {I,3,5}
View Answer Workspace Report
53 . How many rational and irrational numbers are possible between 0 and 1 ?
A. 0 B. Finite
C. InfiniteD. 1
Answer & Explanation
Answer: C

Explanation:

Infinite
View Answer Workspace Report 54 .
In 4th quadrant ?
A. X < 0, Y > 0 B. X > 0, Y < 0
C. X > 0, Y > 0 D. X < 0, Y < 0
Answer & Explanation
Answer: B
Explanation:

X > 0, Y < 0
View Answer Workspace Report 55 .
In 2nd quadrant ?
A. X < 0, Y > 0 B. X > 0, Y > 0
C. X > 0, Y < 0 D. X < 0,Y < 0
Answer & Explanation
Answer: A

Explanation:

X < 0, Y > 0

56 . A—B is read as ?
A. Difference of B and A
B. Difference of A and B of B and A
C. Both a and b
D. None of the above
Answer & Explanation
Answer: B

Explanation:

Difference of A and B of B and A


View Answer Workspace Report
57 . If A =[5,6,7] and B=[7,8,9]then A U B is equal
to: A. [5,6,7] B. [7,8,9]
C. [5,6,7,8,9] D. None of these
Answer & Explanation
Answer: C

Explanation:

[5,6,7,8,9]
View Answer Workspace Report 58 .
In 3rd quadrant ?
A. X < 0, Y > 0 B. X > 0, Y < 0
C. X < 0, Y > 0 D. X < 0, Y > 0
Answer & Explanation
Answer: D

Explanation:
X < 0, Y > 0
View Answer Workspace Report
59 . If A is not equal to B, then the Cartesian product ?
A. A x B not equal B x A
B. A x B = B >< A
C. is not possible
D. None of the above
Answer & Explanation
Answer: A

Explanation:

A x B not equal B x A
View Answer Workspace Report
60 . If A = {0,2) and B = {1,3), then Cartesian product ?
A. AxB not equal BxA B. AxB = BxA
C. is not possible D. None of the above
Answer & Explanation
Answer: A

Explanation:

AxB not equal BxA

61 . A C B is read as ?
A. A is a subset of B
B. B is a subset of A
C. A is less than B
D. A is a proper subset of
B Answer & Explanation
Answer: D

Explanation:

A is a proper subset of B
View Answer Workspace Report
62 . If R = {(1,1),(2,3),(4,5)}, then domain of the function is ?
A. Dom R {1,3,5} B. Dom R = {I,2,4}
C. Dom R {1,1,4,5} D. Dom R = {2,3,4,5}
Answer & Explanation
Answer: A

Explanation:
Dom R = {I,2,4}
View Answer Workspace Report 63 .
In lst quadrant ?
A. X < 0, Y > 0 B. X > 0, Y < 0
C. X > 0, Y > 0 D. X < 0, Y < 0
Answer & Explanation
Answer: C

Explanation:

X > 0, Y > 0
View Answer Workspace Report
64 . The union of sets A and B is expressed as ?
A. A/B B. AUB
C. AxB D. A-B Answer
& Explanation Answer:
B

Explanation:

AUB
View Answer Workspace Report 65 .
(A’)’ = ?
A. A B. U
C. A’ D. U-A Answer
& Explanation Answer:
A

Explanation:

66 . The set of intelligent students in a class is.


A. A null set
B. A finite set
C. A singleton set
D. Not a well defined collection
Answer & Explanation
Answer: D

Explanation:

Not a well defined collection


View Answer Workspace Report 67 .
If A is any set, then
A. A ∩ A' = U B. A ∪ A' = U
C. A ∪ A' = ψ D. None of these
Answer & Explanation
Answer: A

Explanation:

A ∩ A' = U
View Answer Workspace Report
68 . If the set has p elements, b has q elements, the no of elements in A x B is
A. p² B. pq
C. p + q D. p+q+1
Answer & Explanation
Answer: B

Explanation:

pq
View Answer Workspace Report
69 . A survey showed that 63 % of the Americans like cheese whereas 76 %
like apples. If x % of Americans like both cheese and apples, then find the
range of x?
A. 0 ≤ x ≤ 23 % B. 4 ≤ x ≤ 35 % C. 0
≤ x ≤ 39 % D. 6 ≤ x ≤ 33 % Answer &
Explanation
Answer: C

Explanation:

0 ≤ x ≤ 39 %
View Answer Workspace Report
70 . If A and B are any two sets, then A ∪ (A ∩ B) is equal to.
A. A B. B
C. Ac D. Bc
Answer & Explanation
Answer: A

Explanation:

71 . The smallest set A such that A ∪ {1, 2} = {1, 2, 3, 5, 9} is


A. {2, 3, 5} B. {3, 5, 9}
C. {1,2, 5, 9} D. None of these
Answer & Explanation
Answer: B

Explanation:

{3, 5, 9}
View Answer Workspace Report
72 . If A, B and C are any three sets, then A – (B ∪ C) is equal to
A. (A - B) ∪ C B. (A - B) ∩ C
C. (A - B) ∩ (A - C) D. (A - B) ∪ (A - C)
Answer & Explanation
Answer: C

Explanation:

(A - B) ∩ (A - C)
View Answer Workspace Report
73 . If A and B are any two sets, then A ∩ (A ∪ B) is equal to
A. A B. B
C. Ac D. Bc
Answer & Explanation
Answer: A

Explanation:

A
View Answer Workspace Report
74 . Two finite sets have n and m elements. The number of elements in the
power set of first set is 48 more than the total number of elements in power set
of the second set. Then the values of m and n are
A. 6, 3 B. 7, 6
C. 6, 4 D. 7, 4
Answer & Explanation
Answer: C

Explanation:

6, 4
View Answer Workspace Report
75 . The number of proper subsets of the set {1, 2, 3} is.
A. 5 B. 6
C. 7 D. 8
Answer & Explanation
Answer: B
Explanation:

76 . If A ∩ B = B, then.
A. A = ψ B. B=ψ
C. A ⊂ B D. B⊂
A Answer & Explanation
Answer: C

Explanation:

B⊂ A
View Answer Workspace Report
77 . If x ≠ 1, and f(x) = x + 1 / x – 1 is a real function, then f(f(f(2))) is
A. 1 B. 2
C. 3 D. 4
Answer & Explanation
Answer: C

Explanation:

3
View Answer Workspace Report
78 . In a set – builder method, the null set is represented by
A. Φ B. {}
C. { x : x = x} D. {x:x≠
x} Answer & Explanation
Answer: D

Explanation:

{ x : x ≠ x}
View Answer Workspace Report 79 .
A = {x: x ≠ x }represents
A. {} B. {0}
C. {1} D. {x}
Answer & Explanation
Answer: A

Explanation:

{}
View Answer Workspace Report
80 . If A, B and C are any three sets, then A Χ (B ∪ C) is equal to.
A. (A ∪ B) Χ (A ∪ C)
B. (A Χ B) ∩ (A Χ C)
C. (A Χ B) ∪ (A Χ C)
D. None of these
Answer & Explanation
Answer: C

Explanation:

(A Χ B) ∪ (A Χ C)

81 . A set consisting of a definite number of elements is called a


A. Finite set B. Null set
C. Infinite set D. Singleton set
Answer & Explanation
Answer: A

Explanation:

Finite set
View Answer Workspace Report
82 . If A, B, C be three sets such that A ∪ B = A ∪ C and A ∩ B = A ∩ C, then.
A. A = B B. A=C
C. B = C D. A=B=
C Answer & Explanation
Answer: C

Explanation:

B=C
View Answer Workspace Report
83 . If A = {1, 2, 5} and B = {3, 4, 5, 9}, then A ? B is equal to If A = {x: x = 3n, n
? 6, n ? N} and B = {x: x = 9n, n ? 4, n ? N}, then which of the following is
false? A. A ? B = {9, 81, 729, 6561}
B. A U B = {3, 9, 27, 81,243, 729, 6561}
C. A-B = {3, 27,243} (d) A ? D = {3, 27, 243, 6561}
D. none of these
Answer & Explanation
Answer: A

Explanation:
A ? B = {9, 81, 729, 6561}
View Answer Workspace Report
84 . If A = {x ? C: x2 = 1} and B = {x ? C: x4 = 1}, then A ? B is equal
to A.{-i, i} B. {-1, 1}
C. {-1, 1, i, -i} D. none of these
Answer & Explanation
Answer: A

Explanation:

{-i, i}
View Answer Workspace Report
85 . If A and B are two sets containing respectively m and n distinct
elements. How many different relations can be defined for A and B?
A. 2m-n B. 2m/n
C. 2mn D. 2m+n
Answer & Explanation
Answer: C

Explanation:

2mn

86 . { (a, b) : a² +b² = 1} on the set S has the following relation


A. reflexive
B. symmetric
C. reflexive and transitive
D. none
Answer & Explanation
Answer: B

Explanation:

symmetric
View Answer Workspace Report
87 . If Y is the smallest set such that Y U {1, 2} = {1, 2, 3, 5, 9}, then Y is equal to
A. {1,2} B. {3,5,9}
C. {1,2,3,5,9} D. none of these
Answer & Explanation
Answer: B

Explanation:
{3,5,9}
View Answer Workspace Report
88 . If R is the relation “is greater than” from A ={1,2,3,4,5}to B={1,3,4} ,
Than R-1 is
A. {(3,4),(4,5),(3,5)}
B. {(1,2) ,(1,3),(1,4),(1,5)}
C. {(1,2), (1,3), (1,4), (3,4), (1,5), (3,5), (4,5)}
D. {(2,1), (3,1), (4,1),(4,3), (5,1), (5,3), (5,4)}
Answer & Explanation
Answer: C

Explanation:

{(1,2), (1,3), (1,4), (3,4), (1,5), (3,5), (4,5)}


View Answer Workspace Report
89 . If R is a relation on a finite set having a elements , then the number of
relations on A is
A. a² B. a
C. 2a D. 2a2
View Answer Workspace Report
90 . If R is a relation on a finite set having a elements , then the number of
relations on A is
A. a² B. a
C. 2a D. 2a² Answer
& Explanation Answer:
D

91 . Let R= {(x,y) :x, y belong to N, 2x+y =41}. The range is of the relation R is
A. {2n: n belongs to N, 1< n< 20}
B. { (2n+2) : n belongs to N, 1< n <20}
C. {(2n +1):n belongs to N , 1≤ n≤ 20}
D. {(2n-1) : n belongs to N, 1≤ n≤ 20}
Answer & Explanation
Answer: D

Explanation:

{(2n-1) : n belongs to N, 1≤ n≤ 20}


View Answer Workspace Report
92 . If A = {1, 2, 5} and B = {3, 4, 5, 9}, then A ? B is equal to
A. {1, 2, 5, 9}
B. {1, 2, 3, 4, 5, 9}
C. {1, 2, 3, 4, 9}
D. none of these
Answer & Explanation
Answer: C

Explanation:

{1, 2, 3, 4, 9}
View Answer Workspace Report
93 . If n (A) = 115, n (B) = 326, n (A-B) = 47, then n (A U B) is equal
to A.165 B. 370
C. 373 D. 422
Answer & Explanation
Answer: C

Explanation:

373
View Answer Workspace Report 94 .
Solve f(x) = √9-x² the range is
A. {x: 3≤ x ≤ 0} B. {x: 0≤ x ≤ 3}
C. {x: 0< x < 3} D. {x: 3< x <0}
Answer & Explanation
Answer: B

Explanation:

{x: 0≤ x ≤ 3}
View Answer Workspace Report
95 . The range of the function f(x) = │x – 1│
is A. R B. [0, ∞)
C. (0, - ∞) D. (- ∞, 0)
Answer & Explanation
Answer: B

96 . If f(x) = Log [(1 + x)/(1-x), then f (2x )/(1 + x²) is equal to


A. {f(x)}² B. {f(x)}³
C. 2 f (x) D. 3 f(x)
Answer & Explanation
Answer: C

Explanation:

2 f (x)
View Answer Workspace Report
97 . The range of the function f(x) = x / │x│
is A. R - {0} B. {-1, 1}
C. R – {-1, 1} D. None of these
Answer & Explanation
Answer: B

Explanation:

{-1, 1}
View Answer Workspace Report 98 .
A set is known by its .
A. Elements B. Members
C. Letters D. Values
Answer & Explanation
Answer: A

Explanation:

Elements
View Answer Workspace Report
99 . Let f = {(x, x² /1+x² ): x € R } be a function from R into R . range of x is
A. positive real numbers.
B. negative real numbers
C. non negative real numbers
D. any positive real number x such that 0≤ x <1
Answer & Explanation
Answer: D

Explanation:

any positive real number x such that 0≤ x <1


View Answer Workspace Report
100 . A set contains k elements. The power set of this set contains
A. k elements B. 2k elements
C. k2 elements D. 2k ? 2 elements
Answer & Explanation
Answer: B

101 . If A Ν B, then B’ – A’ is equal to


A. ? B. A'
C. B' D. A-B Answer
& Explanation Answer:
A
Explanation:

?
View Answer Workspace Report
102 . If for a ? N, aN = {ax: x ? N}, then the set 6 N ? 8 N is equal to
A. 8N B. 48N
C. 24N D. 72N Answer
& Explanation Answer:
B

Explanation:

48N
View Answer Workspace Report
103 . Let R be a relation N define by x + 2y = 8 . The domain of R
is A. {2,4,6} B. {2,4,8}
C. {1,2,3,4} D. {2,4,6,8}
Answer & Explanation
Answer: A

Explanation:

{2,4,6}
View Answer Workspace Report
104 . If X = {1, 2, 3, 4, 5, 6, 7, 8, 9} A = {x ? N: 30 < x2 < 70}, B = {x : x is a prime
number less than 10}, then which of the following is false: A.
A-B = {6, 8}
B. A? B = {7, 8}
C. A ? B = {2, 3, 5, 6, 8}
D. A U B = {2, 3, 5, 6, 7, 8}
Answer & Explanation
Answer: B

Explanation:

A? B = {7, 8}
View Answer Workspace Report
105 . If for two sets A and B, A U B = A? B = A, then we have
A. A = B B. B?A?
C. A- B ? f D. None of these
Answer & Explanation
Answer: A

Explanation:
A=B

106 . The set (A U B U C) ? (A ? B’ ? C’)’ ? C’ is equal to


A. B?C' B. A?C
C. B' ? C' D. none of these.
Answer & Explanation
Answer: A

Explanation:

B?C'
View Answer Workspace Report
107 . A survey shows that 63% of the Americans like cheese whereas 76%
like apples. If x% of the Americans like both cheese and apples, then we have
A. x ? 39 B. x ? 63
C. 39 ? x ? 63 D. none of these
Answer & Explanation
Answer: C

Explanation:

39 ? x ? 63
View Answer Workspace Report
108 . If aN = {an : n ? N} and bN ? cN = dN, where a, b, c ? N and b, c are
coprime, then
A. a = bd B. b = cd
C. c = bd D. d = bc
Answer & Explanation
Answer: D

Explanation:

d = bc
View Answer Workspace Report
109 . If A = {1, 2, 3, 6, 11,18, 21}, B = {5, 7, 9} and N is the universal set, then A’ U
((AU B) ? B’) is equal to
A. A B. B
C. N D. N-A
Answer & Explanation
Answer: C

Explanation:

N
View Answer Workspace Report
110 . Which of the following is not correct?
A. A ΝA' if and only if A = ?
B. B = C if and only if A U B = A U C and A ? B = A? C
C. A' Ν A if and only if A = X, where X is the universal set
D. All of these
Answer & Explanation
Answer: D

Explanation:

All of these

111 . If A Ν B then which of the following are correct?


A. A- B = ? B. AUB=B
C. A ? B = B - A D. All of these
Answer & Explanation
Answer: D

Explanation:

All of these
View Answer Workspace Report
112 . If A U B = AU C and A ? B = A ? C, then
A. B = C
B. B = C only when A Ν C
C. B = C only when A ? B
D. none of these
Answer & Explanation
Answer: A

Explanation:

B=C
View Answer Workspace Report
113 . If A = {(x, y) : x2 + y2 = 1; x, y ? R} and B = {(x, y): x2 + y2 = 4; x, y ?R} then
A. A ? B = ? B. A?B?f
C. A ? B = ? D. B ?A = B
Answer & Explanation
Answer: A

Explanation:

A?B=?
View Answer Workspace Report
114 . Let X be the universal set for sets A and B. If n (A) = 200, n (B) = 300 and
n (A?B) = 100, then n(A’? B’) is equal to 300 provided n(X) is equal to
A. 500 B. 600
C. 700 D. 800
Answer & Explanation
Answer: C

Explanation:

700
View Answer Workspace Report
115 . Which of the following does not have a proper subset
A. {x: x ? Q}
B. {x: x ? Q, 3 < x < 4
C. {x: x ? N, 3 < x < 4}
D. none of these
Answer & Explanation
Answer: C

Explanation:

{x: x ? N, 3 < x < 4}

116 . If A = {4n -3n -1: n ? N} and B = {9(n-1): n ? N}, then we have


A. B Ν A B. AΜB
C. A? B = ? D. none of these
Answer & Explanation
Answer: B

Explanation:

AΜB
View Answer Workspace Report
117 . Let A = {x: x is a digit in the number 3591}, B = {x: x ? N, x<10}. Which
of the following is false?
A. A? B = {1,3,5,9}
B. A-B = {2,4,6,7,8}
C. A U B = {1,2,3,4,5,6,7,8,9}
D. none of these
Answer & Explanation
Answer: B

Explanation:
A-B = {2,4,6,7,8}
View Answer Workspace Report
118 . Two finite sets have m and n elements. The total number of subsets of
the first set is 56 more than the total number of subsets of the second set. The
values of m and n are respectively
A. 4,1 B. 6, 3
C. 8, 5 D. none of these
Answer & Explanation
Answer: B

Explanation:

6, 3
View Answer Workspace Report
119 . Which of the following are true:
A. 1 and 2
B. {x: x >3, x ?0} ? ?
C. {x: x2-1= 0, x ?N} ? {-1, 1}
D. {x: x2 - 4x + 3 = 0} = {1, 3}
Answer & Explanation
Answer: A

Explanation:

1 and 2
View Answer Workspace Report
120 . Which of the following sets are null sets:
A. 2 and 3
B. {x: x < 5, x > 6}
C. {x: |x |< -4, x ?N}
D. Set of all prime numbers between 15 and 19
Answer & Explanation
Answer: A

Explanation:

2 and 3

121 . If A = {1, 2, 3, 4, 5}, then the number of proper subsets of A is


A. 30 B. 31
C. 32 D. 33
Answer & Explanation
Answer: B
Explanation:

31
View Answer Workspace Report
122 . Set of rational numbers Q is a subset of
A. The set of integers.
B. set of Natural numbers.
C. The set of Even integers.
D. The set of Complex numbers.
Answer & Explanation
Answer: D

Explanation:

The set of Complex numbers. View


Answer Workspace Report 123 .
{X | x ε N Ʌ x < 1} is the
A. Empty set.
B. singleton set.
C. a set with three points.
D. The set with two points.
Answer & Explanation
Answer: D

Explanation:

The set with two points.


View Answer Workspace Report
124 . P: 4 < 7, q: 6 > 11, conjuntion pɅ q is
A. TRUE B. FALSE
C. unknown D. not valid
Answer & Explanation
Answer: B

Explanation:

FALSE
View Answer Workspace Report
125 . A compound statement of form " if p then q " is called an
A. unknown B. conclusion
C. implication D. hypothesis
Answer & Explanation
Answer: C

Explanation:
implication

126 . Disjunction of two statements p and q is denoted by


A. p ˅ q
B. p ← q
C.
p→q
D. p ↔ q
Answer & Explanation
Answer: A

Explanation:

p˅q
View Answer Workspace Report 127 .
In a statement "if p then q" q is
A. unknown B. conclusion
C. hypothesis D. implication
Answer & Explanation
Answer: B

Explanation:

conclusion
View Answer Workspace Report
128 . A subset of B x A is called a
A. Relation from B to AB. Relation from A to B
C. Relation in B D. Relation in A
Answer & Explanation
Answer: A

Explanation:

Relation from B to A
View Answer Workspace Report
129 . A conditional statemnt is regarded as false only antecedent is true
and consequent is
A. Known B. Unknown
C. True D. False
Answer & Explanation
Answer: D

Explanation:

False
View Answer Workspace Report
130 . Number of subsets of a set of 4 elements
A. 2 B. 4
C. 8 D. 16
Answer & Explanation
Answer: D

Explanation:

16

131 . Set of first element of ordered pair forming a relation is called its
A. RangeB. Domain
C. Relation in A D. Relation in B
Answer & Explanation
Answer: A

Explanation:

Range
View Answer Workspace Report
132 . Identity relation in A = {1,2,3,4,5,6}
is A. {(1,1), (2,2),(3,3)}
B. {(4,4),(5,5)(6,6)}
C. {(1,1),(3,3)(4,4),(5,5)}
D. {(1,1),(2,2),(3,3),(4,4),(5,5),(6,6)}
Answer & Explanation
Answer: D

Explanation:

{(1,1),(2,2),(3,3),(4,4),(5,5),(6,6)}
View Answer Workspace Report
133 . Conjunction of two statement p and q is denoted by
A. p Ʌ q B. p←q
C. p → q D. p↔q
Answer & Explanation
Answer: A

Explanation:

pɅq
View Answer Workspace Report
134 . A subset of A x A is called a
A. Relation in A B. Relation in B
C. Relation from A to B D. Relation from B to A
Answer & Explanation
Answer: A

Explanation:

Relation in A
View Answer Workspace Report
135 . A conjunction of two statement p and q is true only if
A. p is true
B. q is true
C. both p and q are true
D. both p and q are false
Answer & Explanation
Answer: C

Explanation:

both p and q are true

136 . {x|x ε N, x ≤ 10} is the


A. Tabular method
B. Set builder notation
C. Descriptive method.
D. Non descriptive method
Answer & Explanation
Answer: B

Explanation:

Set builder notation


View Answer Workspace Report
137 . P: 7 < 4, q: 6 > 11, disjuntion p˅q is
A. Known B. Unknown
C. True D. False
Answer & Explanation
Answer: D

Explanation:

False
View Answer Workspace Report
138 . (A ∪ B) ∪ C =
A. A ∩ (B ∪ C) B. A ∪ ( B ∪ C)
C. A ∪ (B ∩ C) D. π
Answer & Explanation
Answer: B

Explanation:

A ∪ ( B ∪ C)
View Answer Workspace Report
139 . An implication or conditional "if p then q "is denoted by
A. p ← q B. pɅq
C. p → q D. p˅q
Answer & Explanation
Answer: C

Explanation:

p→q
View Answer Workspace Report
140 . Set of real number R is a subset of
A. The set of integers
B. The set of Even integers
C. The set of Natural numbers
D. The set of Complex numbers
Answer & Explanation
Answer: D

Explanation:

The set of Complex numbers

141 . A Subset of B x B is called a


A. Relation in A B. Relation in B
C. Relation from A to B D. Relation from B to A
Answer & Explanation
Answer: B

Explanation:

Relation in B
View Answer Workspace Report
142 . To draw general conclusions from well known facts is called
A. Induction B. Deduction
C. Proposition D. Knowledge
Answer & Explanation
Answer: A
Explanation:

Induction
View Answer Workspace Report
143 . A declarative statement which may be true or false but not both is called
A. Induction B. Deduction
C. Knowledge D. Proposition
Answer & Explanation
Answer: D

Explanation:

Proposition
View Answer Workspace Report
144 . Set of Second element of ordered pair forming a relation is called its
A. RangeB. Domain
C. Relation in A D. Relation in B
Answer & Explanation
Answer: B

Explanation:

Domain
View Answer Workspace Report
145 . Which of the following statements is FALSE?
A. 2 ∈ A ∪ B implies that if 2 ∈/ A then 2 ∈ B.
B. {2, 3} ⊆ A implies that 2 ∈ A and 3 ∈ A.
C. A ∩ B ⊇ {2, 3} implies that {2, 3} ⊆ A and {2, 3} ⊆ B
D. {2} ∈ A and {3} ∈ A implies that {2, 3} ⊆ A.
Answer & Explanation
Answer: D

Explanation:

{2} ∈ A and {3} ∈ A implies that {2, 3} ⊆ A.

146 . Let A = {0, 1} × {0, 1} and B = {a, b, c}. Suppose A is listed in


lexicographic order based on 0 < 1 and B is in alphabetic order. If A × B × A is
listed in lexicographic order, then the next element
A. ((1, 0), a,(0, 0))
B. ((1, 1), c,(0, 0))
C. ((1, 1), a,(0, 0))
D. ((1, 1), a,(1, 1))
Answer & Explanation
Answer: C
Explanation:

((1, 1), a,(0, 0))


View Answer Workspace Report
147 . Which of the following statements is TRUE?
A. For all sets A, B, and C, (A − B) ∩ (C − B) = (A ∩ C) − B.
B. For all sets A, B, and C, (A − B) ∩ (C − B) = A − (B ∪ C).
C. For all sets A, B, and C, A − (B − C) = (A − B) − C.
D. For all sets A, B, and C, if A ∪ C = B ∪ C then A =
B. Answer & Explanation
Answer: A

Explanation:

For all sets A, B, and C, (A − B) ∩ (C − B) = (A ∩ C) − B. View


Answer Workspace Report
148 . Which of the following statements is FALSE?
A. A − (C ∪ B) = (A − B) − C
B. B − (A ∪ C) = (B − C) − A
C. C − (B ∪ A) = (C − B) − A
D. A − (B ∪ C) = (B − C) − A
Answer & Explanation
Answer: D

Explanation:

A − (B ∪ C) = (B − C) − A
View Answer Workspace Report
149 . The power set P((A × B) ∪ (B × A)) has the same number of elements
as the power set P((A × B) ∪ (A × B)) if and only if
A. A = B
B. A = ∅ or B = ∅
C. B = ∅ or A = B
D. A = ∅ or B = ∅ or A = B
Answer & Explanation
Answer: D

Explanation:

A = ∅ or B = ∅ or A = B
View Answer Workspace Report
150 . Let σ = 452631 be a permutation on {1, 2, 3, 4, 5, 6} in one-line notation
(based on the usual order on integers). Which of the following is NOT a
correct cycle notation for σ?
A. (614)(532) B. (461)(352)
C. (253)(146) D. (325)(614)
Answer & Explanation
Answer: B

Explanation:

(461)(352)

151 . Let f : X → Y . Consider the statement, “For all subsets C and D of Y , f


−1 (C∩Dc ) = f −1 (C) ∩ [f −1 (D)]c . This statement is
A. True and equivalent to:
For all subsets C and D of Y , f −1 (C − D) = f −1 (C) − f −1 (D).
B. False and equivalent to:
For all subsets C and D of Y , f −1 (C − D) = f −1 (C) − f −1 (D).
C. True and equivalent to:
For all subsets C and D of Y , f −1 (C − D) = f −1 (C) − [f −1 (D)]c
D. False and equivalent to:
For all subsets C and D of Y , f −1 (C − D) = f −1 (C) − [f −1 (D)]c .
Answer & Explanation
Answer: A

Explanation:

True and equivalent to:


For all subsets C and D of Y , f −1 (C − D) = f −1 (C) − f −1 (D). View
Answer Workspace Report
152 . The number of partitions of {1, 2, 3, 4, 5} into three blocks is S(5, 3) = 25.
The total number of functions f : {1, 2, 3, 4, 5} → {1, 2, 3, 4} with |Image(f)| = 3 is
A. 4 Χ 6 B. 4 Χ 25
C. 25 Χ 6 D. 4 Χ 25 Χ 6
Answer & Explanation
Answer: D

Explanation:

4 Χ 25 Χ 6
View Answer Workspace Report
153 . Let f : X → Y and g : Y → Z. Let h = g ◦ f : X → Z. Suppose g is one-to-
one and onto. Which of the following is FALSE?
A. If f is one-to-one then h is one-to-one and onto
B. If f is not onto then h is not onto
C. If f is not one-to-one then h is not one-to-one
D. If f is one-to-one then h is one-to-one
Answer & Explanation
Answer: A

Explanation:

If f is one-to-one then h is one-to-one and onto


View Answer Workspace Report
154 . Which of the following statements is FALSE?
A. {2, 3, 4} ⊆ A implies that 2 ∈ A and {3, 4} ⊆ A
B. {2, 3, 4} ∈ A and {2, 3} ∈ B implies that {4} ⊆ A − B.
C. A ∩ B ⊇ {2, 3, 4} implies that {2, 3, 4} ⊆ A and {2, 3, 4} ⊆ B
D. A − B ⊇ {3, 4} and {1, 2} ⊆ B implies that {1, 2, 3, 4} ⊆ A ∪ B
Answer & Explanation
Answer: B

Explanation:

{2, 3, 4} ∈ A and {2, 3} ∈ B implies that {4} ⊆ A − B.


View Answer Workspace Report
155 . Let A = {0, 1} Χ {0, 1} Χ {0, 1} and B = {a, b, c} Χ {a, b, c} Χ {a, b, c}.
Suppose A is listed in lexicographic order based on 0 < 1 and B is listed in
lexicographic order based on a < b < c. If AΧB
A. ((1, 0, 0),(b, a, a),(0, 0, 0))
B. ((1, 0, 0),(a, a, a),(0, 0, 1))
C. ((1, 0, 0),(a, a, a),(1, 0, 0))
D. ((1, 0, 0),(a, a, a),(0, 0, 0))
Answer & Explanation
Answer: D

Explanation:

((1, 0, 0),(a, a, a),(0, 0, 0))

156 . Let A = {1, 2, 3} and let R = {(1, 1), (2, 2), (3, 3), (1, 3), (3, 2), (1, 2)}. Then
R is
A. reflexive and symmetric but not transitive
B. reflexive and transitive but not symmetric
C. symmetric and transitive but not reflexive
D. an equivalence relation
Answer & Explanation
Answer: B

Explanation:
reflexive and transitive but not symmetric
View Answer Workspace Report
157 . Let A {a, b, c} and let R = {(a, a)(a, b), (b, a)}. Then, R is
A. reflexive and symmetric but not transitive
B. reflexive and transitive but not symmetric
C. symmetric and transitive but not reflexive
D. an equivalence relation.
Answer & Explanation
Answer: C

Explanation:

symmetric and transitive but not reflexive


View Answer Workspace Report
158 . Let A = {1, 2, 3} then total number of element in A x A is
A. 3 B. 6
C. 9 D. 12
Answer & Explanation
Answer: C

Explanation:

9
View Answer Workspace Report
159 . Let R be the relation in the set N given by R = {(a, b): a = b – 2, b > 6}.
Choose the correct answer.
A. (2, 4) ∈ R
B. (3, 8) ∈ R
C. (6, 8) ∈ R
D. (8, 7) ∈ R
Answer & Explanation
Answer: C

Explanation:

(6, 8) ∈ R
View Answer Workspace Report
160 . Let R = {(3, 3), (6, 6), (9, 9), (3,6), (3, 9), (9, 12), (3,12), (6, 12), (12, 12)},
be a relation on the set
A. reflexive only
B. reflexive and transitive only
C. reflexive and symmetric only
D. an equivalence relation
Answer & Explanation
Answer: B
Explanation:

reflexive and transitive only

161 . The domain of the function f = {(1, 3), (3, 5), (2, 6)} is
A. 1, 3 and 2 B. {1, 3, 2}
C. {3, 5, 6} D. 3, 5 and 6
Answer & Explanation
Answer: B

Explanation:

1) A graph is a collection of....?


a. Row and columns
b. Vertices and edges
c. Equations
d. None of these
Show/Hide
Answer Answer =
B
Explanation: A graph contains the edges and vertices
2) The degree of any vertex of graph is......?
a. The number of edges incident with vertex
b. Number of vertex in a graph
c. Number of vertices adjacent to that vertex
d. Number of edges in a graph
Show/Hide Answer
Answer = A
Explanation: The number of edges connected on a vertex v with the self loop counted
twice is called the degree of vertex.
3) If for some positive integer k, degree of vertex d(v)=k for every vertex v of
the graph
G, then G is called.......?
a. K graph
b. K-regular graph
c. Empty graph
d. All of above
Show/Hide
Answers Answer =
B
Explanation: A graph in which all vertices are of equal degree is called regular graph.
4) A graph with no edges is known as empty graph. Empty graph is also
known as ?
a. Trivial graph
b. Regular graph
c. Bipartite graph
d. None of these
Show/Hide
Answer Answer =
A
Explanation: Trivial graph is the second name for empty graph.
5) Length of the walk of a graph is......?
a. The number of vertices in walk W
b. The number of edges in walk W
c. Total number of edges in a graph
d. Total number of vertices in a graph
Show/Hide Answer
Answer = B
Explanation: A walk is defined as finite altering sequence of vertices and edges. No
Edges appear more than once but vertex may appear more than once.
6) If the origin and terminus of a walk are same, the walk is known as....?
a. Open
b. Closed
c. Path
d. None of these
Show/Hide
Answer Answer =
B
Explanation: A walk which begins and ends with same vertex is called closed walk
otherwise it is open.
7) A graph G is called a......if it is a connected acyclic graph ?
a. Cyclic graph
b. Regular graph
c. Tree
d. Not a graph
Show/Hide
Answer Answer =
C
Explanation: No explanation for this question.
8) Eccentricity of a vertex denoted by e(v) is defined by....?
a. max { d(u,v): u belongs to v, u does not equal to v : where d(u,v) is the
distance
between u&v}
b. min { d(u,v): u belongs to v, u does not equal to v }
c. Both A and B
d. None of these
Show/Hide
Answer Answer =
A
Explanation: The eccentricity E(v) of a vertex V in the graph is the distance from v to
the vertex farthest from v in G.
9) Radius of a graph, denoted by rad(G) is defined by....?
a. max {e(v): v belongs to V }
b. min { e(v): v belongs to V}
c. max { d(u,v): u belongs to v, u does not equal to v }
d. min { d(u,v): u belongs to v, u does not equal to v }
Show/Hide Answer
Answer = A
Explanation: The diameter or radius of a graph G is largest distance between two
vertices in the graph G.
10) The complete graph K, has....different spanning trees?
a. n
n-2
b. n*n
c. n
n
d. n
2
Show/Hide Answer
Answer = A

1 A graph is a collection of....?


A Row and columnsB Vertices and edgesC EquationsD None of these

Answer: Vertices and edges


2 The degree of any vertex of graph is......?
A The number of edges incident with vertexB Number of vertex in a graphC Number
of vertices adjacent to that vertexD Number of edges in a graph

Answer: The number of edges incident with vertex


Advertisement

3 A graph with no edges is known as empty graph. Empty graph is also


known as ?
A Trivial graphB Regular graphC Bipartite graphD None of these

Answer: Trivial graph


4 If the origin and terminus of a walk are same, the walk is known as. . .?
A OpenB ClosedC PathD None of these

Answer: Closed
5 Radius of a graph, denoted by rad(G) is defined by....?
A max {e(v): v belongs to V }B min { e(v): v belongs to V}C max { d(u,v): u belongs
to v, u does not equal to v }D min { d(u,v): u belongs to v, u does not equal to v }

Answer: max {e(v): v belongs to V }


6 A graph G is called a......if it is a connected acyclic graph
A Cyclic graphB Regular graphC TreeD Not a graph

Answer: Tree
7 A graph is a collection of
A Row and columnsB Vertices and edgesC EquationsD None of these

Answer: Vertices and edges


8 How many relations are there on a set with n elements that are symmetric
and a set with n elements that are reflexive and symmetric ?
A 2n(n+1)/2 and 2n.3n(n–1)/2B 3n(n–1)/2 and 2n(n–1)C 2n(n+1)/2 and 3n(n–1)/2D
2n(n+1)/2 and 2n(n–1)/2

Answer: 2n(n+1)/2 and 2n(n–1)/2 9


In a graph if e=(u, v) means
A u is adjacent to v but v is not adjacent to uB e begins at u and ends at vC u is
processor and v is successorD both b and c

Answer: both b and c


10 A minimal spanning tree of a graph G is
A A spanning sub graphB A treeC Minimum weightsD All of above

Answer: All of above


11 A partial ordered relation is transitive, reflexive and
A AntisymmetricB BisymmetricC Anti reflexiveD Asymmetric

Answer: Antisymmetric
12 A graph with n vertices will definitely have a parallel edge or self loop if
the total number of edges are
A greater than n–1B less than n(n–1)C greater than n(n–1)/2D less than n2/2

Answer: greater than n–1


13 A vertex of a graph is called even or odd depending upon
A Total number of edges in a graph is even or oddB Total number of vertices in a graph
is even or oddC Its degree is even or oddD None of these

Answer: Its degree is even or odd


14 The expression a+a c is equivalent to
A aB a+cC cD 1

Answer: a+c
15 A graph with no edges is known as empty graph. Empty graph is also
known as
A Trivial graphB Regular graphC Bipartite graphD None of these

Answer: Trivial graph


16 A continuous non intersecting curve in the plane whose origin and
terminus coincide
A PlanerB JordanC HamiltonianD All of these

Answer: Jordan
17 A graph with n vertices will definitely have a parallel edge or self loop of
the total number of edges are
A more than nB more than n+1C more than (n+1)/2D more than n(n-1)/2

Answer: more than n(n-1)/2


18 Which of the following pair is not congruent modulo 7?
A 10, 24B 25, 56C -31, 11D -64, -15

Answer: 25, 56
19 The maximum degree of any vertex in a simple graph with n vertices
is A n–1B n+1C 2n–1D n

Answer: n–1
20 Consider a weighted undirected graph with positive edge weights and let (u,
v) be an edge in the graph. It is known that the shortest path from source vertex s to u
has weight 53 and shortest path from s to v has weight 65. Which statement is always
true ?
A Weight (u, v) <= 12B Weight (u, v) = 12C Weight (u, v) >= 12D Weight (u, v) > 12

Answer: Weight (u, v) >= 12

21 How many onto (or surjective) functions are there from an n-element (n =>
2) set to a 2-element set?
A 2nB 2n - 1C 2n - 2D 2(2n – 2)

Answer: 2n - 2
22 Hasse diagram are drawn
A Partially ordered setsB LatticesC Boolean algebraD None of these

Answer: Partially ordered sets


Advertisement

23 In how many ways can 5 balls be chosen so that 2 are red and 3 are black
A 910B 990C 970D None of these

Answer: 990
24 Circle has
A No verticesB Only 1 vertexC 8 verticesD None of these

Answer: No vertices
25 The proposition ~qvp is equivalent to
A p?qB q?pC p?qD p?q

Answer: q?p
26 If B is a Boolean Algebra, then which of the following is true
A B is a finite but not complemented latticeB B is a finite, complemented and
distributive latticeC B is a finite, distributive but not complemented latticeD B is not
distributive lattice

Answer: B is a finite, complemented and distributive lattice


27 If R is a relation “Less Than” from A = {1,2,3,4} to B = {1,3,5} then RoR-1
is A {(3,3), (3,4), (3,5)}B {(3,1), (5,1), (3,2), (5,2), (5,3), (5,4)}C {(3,3), (3,5),
(5,3),
(5,5)}D {(1,3), (1,5), (2,3), (2,5), (3,5), (4,5)}

Answer: {(3,3), (3,5), (5,3), (5,5)}


28 The number of distinguishable permutations of the letters in the
word BANANA are,
A 60B 36C 20D 10

Answer: 60
29 Let G be a simple undirected planar graph on 10 vertices with 15 edges. If
G is a connected graph, then the number of bounded faces in any embedding
of G on the plane is equal to
A 3B 4C 5D 6

Answer: 6
30 A graph is tree if and only if
A Is planarB Contains a circuitC Is minimallyD Is completely connected

Answer: Is minimally
31 How many different words can be formed out of the letters of the
word VARANASI?
A 64B 120C 40320D 720

Answer: 720
32 Suppose v is an isolated vertex in a graph, then the degree of v is

Answer: 0
A 0B 1C 2D 3

Answer: 0
33 The complete graph with four vertices has k edges where k is
A 3B 4C 5D 6

Answer: 6
34 Which one of the following statements is incorrect ?
A The number of regions corresponds to the cyclomatic complexityB Cyclometric
complexity for a flow graph G is V(G) = N–E+2, where E is the number of edges and N
is the number of nodes in the flow graphC Cyclometric complexity for a flow graph G
is V(G) = E–N+2, where E is the number of edges & N is the number of nodes in the
flow graphD Cyclometric complexity for a flow graph G is V(G) = P + 1, where P is the
number of predicate nodes contained in the flow graph G

Answer: Cyclometric complexity for a flow graph G is V(G) = N–E+2, where E is the
number of edges and N is the number of nodes in the flow graph
35 Choose the most appropriate definition of plane graph
A A graph drawn in a plane in such a way that any pair of edges meet only at their end
verticesB A graph drawn in a plane in such a way that if the vertex set of graph can be
partitioned into two non - empty disjoint subset X and Y in such a way that each edge of
G has one end in X and one end in YC A simple graph which is Isomorphic to
Hamiltonian graphD None of these

Answer: A graph drawn in a plane in such a way that any pair of edges meet only at
their end vertices
36 Length of the walk of a graph is
A The number of vertices in walk WB The number of edges in walk WC Total
number of edges in a graphD Total number of vertices in a graph

Answer: The number of edges in walk W 37 A


graph with one vertex and no edges is
A multigraphB digraphC isolated graphD trivial graph

Answer: trivial graph


38 In any undirected graph the sum of degrees of all the nodes
A Must be evenB Are twice the number of edgesC Must be oddD Need not be even

Answer: Are twice the number of edges


39 In a graph if e=[u, v], Then u and v are called
A Endpoints of eB Adjacent nodesC NeighborsD All of above

Answer: All of above


40 The number of leaf nodes in a complete binary tree of depth d is
A 2dB 2d–1+1C 2d+1+1D 2d+1

Answer: 2d
41 An undirected graph possesses an eulerian circuit if and only if it
is connected and its vertices are
A all of even degreeB all of odd degreeC of any degreeD even in number

Answer: all of even degree


42 The relation { (1,2), (1,3), (3,1), (1,1), (3,3), (3,2), (1,4), (4,2), (3,4)} is
A ReflexiveB TransitiveC SymmetricD None of these

Answer: Transitive
Advertisement

43 In an undirected graph the number of nodes with odd degree must be


A ZeroB OddC PrimeD Even

Answer: Even
44 What is the probability of choosing correctly an unknown integer between 0
and 9 with 3 chances ?
A 963/1000B 966/1000C 968/1000D None of these

Answer: 963/1000
45 The complete graph K, has... different spanning
trees? A nn-2B n*nC nnD n2

Answer: nn-2
46 Eccentricity of a vertex denoted by e(v) is defined by....?
A max { d(u,v): u belongs to v, u does not equal to v : where d(u,v) is the distance
between u&v}B min { d(u,v): u belongs to v, u does not equal to v }C Both A and BD
None of these

Answer: max { d(u,v): u belongs to v, u does not equal to v : where d(u,v) is the
distance between u&v}
47 A graph G is called a......if it is a connected acyclic graph ?
A Cyclic graphB Regular graphC TreeD Not a graph

Answer: Tree
48 Length of the walk of a graph is......?
A The number of vertices in walk WB The number of edges in walk WC Total
number of edges in a graphD Total number of vertices in a graph

Answer: The number of edges in walk W


49 If for some positive integer k, degree of vertex d(v)=k for every vertex v of
the graph G, then G is called... ?
A K graphB K-regular graphC Empty graphD All of above

Answer: K-regular graph


50 The number of colours required to properly colour the vertices of
every planer graph is
A 2B 3C 4D 5

Answer: 5
51 In how many ways can a president and vice president be chosen from a set
of 30 candidates?
A 820B 850C 880D 870

Answer: 870
52 Consider an undirected random graph of eight vertices. The probability
that there is an edge between a pair of vertices is ½. What is the expected
number of unordered cycles of length three?
A 1/8B 1C 7D 3

Answer: 7
53 In how many ways can a hungry student choose 3 toppings for his prize
from a list of 10 delicious possibilities?
A 100B 120C 110D 150

Answer: 120
54 The number of colours required to properly color vertices of every
planar graph is
A 2B 3C 4D 5

Answer: 2

formulas

Set Identities
Identity laws
A∪ ∅ =AA∩U=A
Domination laws
A∪ U=UA∩∅=∅
Idempotent laws
A∪ A=AA∩A=A
Complementation law
(A) = A
Complement laws
A∩A=∅A∪ A=U
Richard Mayr (University of Edinburgh, UK) Discrete Mathematics.
Chapters 2 and 9 22 / 74
Set Identities (cont.)
Commutative laws
A∪ B=B∪ AA∩B=B∩A
Associative laws
A ∪ (B ∪ C) = (A ∪ B) ∪ C
A ∩ (B ∩ C) = (A ∩ B) ∩ C
Distributive laws
A ∩ (B ∪ C) = (A ∩ B) ∪ (A ∩ C)
A ∪ (B ∩ C) = (A ∪ B) ∩ (A ∪ C)
Absorption laws
A ∪ (A ∩ B) = A A ∩ (A ∪ B) = A
De Morgan’s laws
A∪ B=A∩BA∩B=A∪ B

Properties of Binary Relations


A binary relation R ⊆ A Χ A is called
Reflexive iff ∀x (x, x) ∈ R
Symmetric iff ∀x, y ((x, y) ∈ R → (y, x) ∈ R) Antisymmetric
iff ∀x, y ((x, y) ∈ R ∧ (y, x) ∈ R → x = y) Transitive iff ∀x, y, z
((x, y) ∈ R ∧ (y, z) ∈ R → (x, z) ∈ R).

TABLE 6 Logical Equivalences.


Equivalence Name
p ∧ T ≡ p Identity laws
p ∨ F ≡ p p ∨ T ≡ T Domination laws p
∧ F ≡ F p ∨ p ≡ p Idempotent laws
p ∧ p ≡ p ¬(¬p) ≡ p Double negation law p ∨
q ≡ q ∨ p Commutative laws
p∧ q≡q∧ p
(p ∨ q) ∨ r ≡ p ∨ (q ∨ r) Associative laws (p
∧ q) ∧ r ≡ p ∧ (q ∧ r)
p ∨ (q ∧ r) ≡ (p ∨ q) ∧ (p ∨ r) Distributive laws
p ∧ (q ∨ r) ≡ (p ∧ q) ∨ (p ∧ r)
¬(p ∧ q) ≡ ¬p ∨ ¬q De Morgan’s laws
¬(p ∨ q) ≡ ¬p ∧ ¬q p ∨ (p ∧ q) ≡ p Absorption laws
p ∧ (p ∨ q) ≡ p p ∨ ¬p ≡ T Negation laws
p ∧ ¬p ≡ F

TABLE 7 Logical Equivalences


Involving Conditional
Statements.
p → q ≡ ¬p ∨ q p → q ≡ ¬q → ¬p p ∨ q ≡ ¬p → q p ∧ q ≡ ¬(p → ¬q)
¬(p → q) ≡ p ∧ ¬q
(p → q) ∧ (p → r) ≡ p → (q ∧ r)
(p → r) ∧ (q → r) ≡ (p ∨ q) → r
(p → q) ∨ (p → r) ≡ p → (q ∨ r)
(p → r) ∨ (q → r) ≡ (p ∧ q)
→ r TABLE 8 Logical
Equivalences Involving
Biconditional Statements.
p ↔ q ≡ (p → q) ∧ (q → p)
p ↔ q ≡ ¬p ↔ ¬q p ↔ q ≡ (p ∧ q) ∨ (¬p ∧ ¬q)
¬(p ↔ q) ≡ p ↔ ¬q
1. A __________ is an ordered collection of objects.
a) Relation
b) Function
c) Set
d) Proposition

Answer: c
2. The least number of computers required to connect 10 computers to 5 routers to guarantee 5 computers can directly access 5
routers is ______
a) 74
b) 104
c) 30
d) 67

Answer: c
3. In a group of 267 people how many friends are there who have an identical number of friends in that group?
a) 266
b) 2
c) 138
d) 202

Answer: b
4. When four coins are tossed simultaneously, in _______ number of the outcomes at most two of the coins will turn up as heads.
a)17
b)28
c)11
d)43 43
Answer:c

5. How many numbers must be selected from the set {1, 2, 3, 4} to guarantee that at least one pair of these numbers add up to 7?
a)14
b)5
c)9
d)24
Answer:b
6. During a month with 30 days, a cricket team plays at least one game a day, but no more than 45 games. There must be a period of
some number of consecutive days during which the team must play exactly ______ number of games.
a)17
b)46
c)124
d)24
Answer:d
7. In how many ways can 8 different dolls be packed in 5 identical gift boxes such that no box is empty if any of the boxes hold all of the
toys?
a)2351
b)365
c)2740
d)1260
Answer:d
8. A group of 20 girls plucked a total of 200 oranges. How many oranges can be plucked one of them?
a)24
b)10
c)32
d)7
Answer:a
9. In a get-together party, every person present shakes the hand of every other person. If there were 90 handshakes in all, how many
persons were present at the party?
a)15
b)14
c)16
d)17
Answer:b
10. A bag contains 25 balls such as 10 balls are red, 7 are white and 8 are blue. What is the minimum number of balls that must be
picked up from the bag blindfolded (without replacing any of it) to be assured of picking at least one ball of each colour?
a)10
b)18
c)63
d)35 Answer:b
1. How many even 4 digit whole numbers are there?
a)1358
b)7250
c)4500
d)3600

Answer:c

2. In a multiple-choice question paper of 15 questions, the answers can be A, B, C or D. The number of different ways of answering the
question paper are ________
a)65536x47
b)194536x45
c)23650x49
d)11287435
Answer:a

3. How many words with seven letters are there that start with a vowel and end with an A? Note that they don’t have to be real words
and letters can be repeated.
a)45087902
b)64387659
c)12765800
d) 59406880
Answer:d

4. Neela has twelve different skirts, ten different tops, eight different pairs of shoes, three different necklaces and five different bracelets.
In how many ways can Neela dress up?
a)50057
b)14400
c)34870
d)56732
Answer: b.
5. How many five-digit numbers can be made from the digits 1 to 7 if repetition is allowed?
a)16807
b)54629
c)23467
d)32354
View Answer
Answer:a

6. For her English literature course, Ruchika has to choose one novel to study from a list of ten, one poem from a list of fifteen and one
short story from a list of seven. How many different choices does Rachel have?
a)34900
b)26500
c)12000
d)10500
View Answer
Answer:d

7. There are two different Geography books, five different Natural Sciences books, three different History books and four different
Mathematics books on a shelf. In how many different ways can they be arranged if all the books of the same subjects stand together?
a)353450
b)638364
c)829440
d)768700

Answer: c.
8. The code for a safe is of the form PPPQQQQ where P is any number from 0 to 9 and Q represents the letters of the alphabet. How
many codes are possible for each of the following cases? Note that the digits and letters of the alphabet can be repeated.
a)874261140
b)537856330
c)549872700
d)456976000

Answer:d

9. Amit must choose a seven-digit PIN number and each digit can be chosen from 0 to 9. How many different possible PIN numbers can
Amit choose?
a) 10000000
b) 9900000
c) 67285000
d) 39654900

Answer:a

10. A head boy, two deputy head boys, a head girl and 3 deputy head girls must be chosen out of a student council consisting of 14 girls
and 16 boys. In how many ways can they are chosen?
a)98072
b)27384
c)36428
d)44389

Answer: b

1. The shaded area of figure is best described by?

a)A∩B
b)AUB
c)A
d)B

Answer:a

2. The shaded area of figure is best described by?

a)A‘(ComplementofA)
b)AUB-B
c)A∩B
d)B

Answer:b

3. If n(A)=20 and n(B)=30 and n(A U B) = 40 then n(A ∩ B) is?


a)20
b)30
c)40
d)10

Answer:d
Explanation: n(A U B) = n(A) + n(B) – n(A ∩ B).
4. The shaded area of figure is best described by?
a)A‘(ComplementofA)
b)B–(A∩B)–(C∩B)
c)A∩C∩B
d)B’(ComplementofB)

Answer:b
Explanation: The region is difference B with A and C.
5. The relation between sets A, B, C as shown by venn diagram is __________
a)A is subset of B and B is subset of C
b)C is not a subset of A and A is subset of B
c)C is subset of B and B is subset of A
d)Noneofthementioned

Answer:c
Explanation: As set C is totally inside set B, set B is totally inside set A.
6.Let A: All badminton player are good sportsperson.
B: All person who plays cricket are good sportsperson.
Let X denotes set of all badminton players, Y of all cricket players, Z of all good sportsperson. Then which of the following statements is
correct?
a)Z contains both X and Y
b)Z contains X and Y is outside
c)X contains Y and Z
d)None of the mentioned

Answer:a
Explanation: X and Y are subset of Z.
7. If n(A)=10, n(B)=30,n(C)=50 and if set A, B, C are pairwise disjoint then which of the following is correct?
a)n(AUB)=0
b)n(BUC)=0
c)n(AUBUC)=90
d)Allofthementioned

Answer:d
Explanation: All the statements are true based on definition.
8. In the given figure the if n(A)=20,n(U)=50,n(C)=10 and n(A∩B)=5 then n(B)=?

a)35
b)20
c)30
d)10

Answer:a
Explanation: Here n(B)= n(U) – n(A) + n(A∩B).
9. Let the students who likes table tennis be 12, the ones who like lawn tennis 10, those who like only table tennis are 6, then number of
students who likes only lawn tennis are, assuming there are total of 16 students.
a)16
b)8
c)4
d)10
View Answer
Answer:c
Explanation: The students who only plays lawn tennis will be a total lawn tennis player – those who play both the sports.
10. The shaded area of figure is best described by?

a)A‘(ComplementofA)
b)AUB–(A∩B)
c)A–B
d)B

Answer:b
Explanation: The region is complement of( A intersection B).

1. If a set contains 3 elements then the number of subsets is?


a)6
b)3
c)12
d)8

Answer:d
Explanation: For elements with n elements the number of subsets are 2n.
2. The set containing all the collection of subsets is known as _________
a)Subset
b)Powerset
c)Unionset
d)Noneofthementioned

Answer:b
Explanation: Power set contains all the subsets as its elements.
3. If a set is empty then number of subsets will be _________
a)1
b)2
c)0
d)4

Answer:a
Explanation: The set has zero elements so 2o = 1.
4. If the number of subsets of a set are 4 then the number of elements in that sets are _________
a)1
b)2
c)3
d)4

Answer:b
Explanation: The number of elements be x then x2 = 4 thus x=2.
5. The number of subsets of a set is 5.
a)True
b)False

Answer:b
Explanation: The number of subsets will always be a power of 2.
6. The number of subsets of a set can be odd or even.
a)True
b)False

Answer:a
Explanation: The number of subsets will be odd in case of empty set otherwise even.
7. Let a set be A={1, 2, 3} then the number of subsets containing two elements will be _________
a)4
b)3
c)5
d)8

Answer:b
Explanation: The subsets will be {1, 2}, {2, 3}, {1, 3}.
8. Let the set be A= {a, b, c, {a,b}} then which of the following is false?
a){a,b}ЄA
b)aЄA
c){a}ЄA
d)b,cЄA

Answer:c
Explanation: Only elements belongs to a set, {a} is a subset of this set.
9. If A={1, 2, 3, 4}, then the number of the subsets of A that contain the element 2 but not 3, is?
a)16
b)4
c)8
d)24

Answer:b
Explanation: The subsets would be {1, 2, 4},{1, 2}, {2, 3}, {2}.
10. Let A(1), A(2), A(3),…….., A(100) be 100 sets such that number of elements in A(i)=i+1 and A(1) is subset of A(2), A(2)is subset of
A(3),….., A(99) is subset of A(100). The number of elements in union of the all the sets are: n(A(1) U A(2) U A(3) …..U A(100)).
a)99
b)100
c)101
d)102

Answer:c
Explanation: Since all sets are subsets of A(100) therefore in union only elements of A(100)will come.A(100) contains 101 elements.
This set of Discrete Mathematics Multiple Choice Questions & Answers (MCQs) focuses on “Functions”.

1. A function is said to be ______________ if and only if f(a) = f(b) implies that a = b for all a and b in the domain of f.
a)One-to-many
b)One-to-one
c)Many-to-many
d)Many-to-one

Answer:b

2. The function f(x)=x+1 from the set of integers to itself is onto. Is it True or False?
a)True
b)False

Answer:a

3. The value of ⌊1/2.⌊5/2⌋ ⌋ is ______________


a)1
b)2
c)3
d)0.5
View Answer
Answer:a

4. Which of the following function f:ZXZ→Z is not onto?


a)f(a,b)=a+b
b)f(a,b)=a
c)f(a,b)=|b|
d)f(a,b)=a–b

Answer:c
Explanation: The function is not onto as f(a)≠b.
5. The domain of the function that assign to each pair of integers the maximum of these two integers is ___________
a)N
b)Z
c)Z +
d)Z+ XZ+

Answer:d
Explanation: The domain of the integers is Z+ X Z+.
6. Let f and g be the function from the set of integers to itself, defined by f(x) = 2x + 1 and g(x) = 3x + 4. Then the composition of f and g
is ____________
a)6x+9
b)6x+7
c)6x+6
d)6x+8

Answer:a
Explanation: The composition of f and g is given by f(g(x)) which is equal to 2(3x + 4) + 1.
7. __________ bytes are required to encode 2000 bits of data.
a)1
b)2
c)3
d)8

Answer:b
Explanation: Two bytes are required to encode 2000 (actually with 2 bytes you can encode up to and including 65,535.
8. The inverse of function f(x) = x3 + 2 is ____________
a)f -1 (y)=(y–2) 1/2
b)f -1 (y)=(y–2) 1/3
c)f -1 (y)=(y) 1/3
d)f -1 (y)=(y–2)
Answer:b
Explanation: To find the inverse of the function equate f(x) then find the value of x in terms of y such that f -1 (y) = x.
9. The function f(x) = x 3 is bijection from R to R. Is it True or False?
a)True
b)False

Answer:a

10. The g -1({0}) for the function g(x)= ⌊x⌋ is ___________


a){x|0≤x<}
b){x|0<x≤1}
c){x|0<x<1}
d){x|0≤x≤1}
View Answer
Answer:d

1. For an inverse to exist it is necessary that a function should be __________


a)injection
b)bijection
c)surjection
d)noneofthementioned

Answer:b
Explanation: Inverse exist only for those functions which are one one and onto.
2.If f(x) = y then f-1(y) is equal to __________
a)y
b)x
c)x2
d)noneofthementioned

Answer:b

3. A function f(x) is defined from A to B then f -1 is defined __________


a)fromAtoB
b)fromBtoA
c)dependsontheinverseoffunction
d)noneofthementioned

Answer:b

4. If f is a function defined from R to R, is given by f(x) = 3x – 5 then f –1(x) is given by __________


a)1/(3x-5)
b)(x+5)/3
c)doesnotexistsinceitisnotabijection
d)noneofthementioned
Answer:b
Explanation: y = 3x-5, x = (y+5)/3, f -1(x) = (x+5)/3.
5. For some bijective function inverse of that function is not bijective.
a)True
b)False

Answer:b.
6. f(x) is a bijection than f -1(x) is a mirror image of f(x) around y = x.
a)True
b)False

Answer:a

7. If f is a function defined from R to R, is given by f(x) = x 2 then f -1(x) is given by?


a)1/(3x-5)
b)(x+5)/3
c)doesnotexistsinceitisnotabijection
d)noneofthementioned

Answer:c.
8. For any function fof -1(x) is equal to?
a)x
b)1
c)x2
d)noneofthementioned
View Answer
Answer:a

9.The solution to f(x) = f -1(x) are __________


a)nosolutionsinanycase
b)sameassolutiontof(x)=x
c)infinitenumberofsolutionforeverycase
d)noneofthementioned
View Answer
Answer:b

10. Let f(x) = x then number of solution to f(x) = f -1(x) is zero.


a)True
b)False

Answer:b

1.A floor function map a real number to ___________


a)smallestpreviousinteger
b)greatestpreviousinteger
c)smallestfollowinginteger
d)noneofthementioned

Answer:b
Explanation: Floor function f(x) is the largest integer not greater than x.
2.A ceil function map a real number to __________
a)smallestpreviousinteger
b)greatestpreviousinteger
c)smallestfollowinginteger
d)noneofthementioned

Answer:c
Explanation: Ceil function f(x) is the smallest integer not less than x.
3. A function f(x) is defined as f(x) = x – [x], where [.] represents GIF then __________
a)f(x)willbeintergralpartofx
b)f(xwillbefractionalpartofx
c)f(x)willalwaysbe0
d)noneofthementioned

Answer:b
Explanation: A integral part of a number is subtracted from that number we are left with the fractional part of that number.
4.Floor(2.4)+Ceil(2.9)isequalto__________
a)4
b)6
c)5
d)noneofthementioned

Answer:c
Explanation: Floor(2.4) = 2, Ceil(2.9) = 3, 2 + 3 = 5.
5. For some integer n such that x < n < x + 1, ceil(x)<n.
a)True
b)False

Answer:b
Explanation: If x < n < x + 1 then ceil(x) = n.
6.For some number x, Floor(x)<=x<=Ceil(x).
a)True
b)False

Answer:a
Explanation: Floor function f(x) is the largest integer not greater than x and ceil function f(x) is the smallest integer not less than x.
7. If x, and y are positive numbers both are less than one, then maximum value of floor(x + y) is?
a)0
b)1
c)2
d)-1
Answer:b.
8. If x, and y are positive numbers both are less than one, then maximum value of ceil(x + y) is?
a)0
b)1
c)2
d)-1
r
Answer:c

9.If X = Floor(X) = Ceil(X) then __________


a)Xisafractionalnumber
b)XisInteger
c)Xislessthan1
d)noneofthementioned

Answer:b
Explanation: Only in case of integers X = Floor(X) = Ceil(X) holds good.
10. Let n be some integer greater than 1,then floor((n-1)/n) is 1.
a)True
b)False

Answer:b

S-ar putea să vă placă și